Criminal Law

Réussis tes devoirs et examens dès maintenant avec Quizwiz!

An ACT is defined as:

"An effect produced through conscious exertion of will".

Number of Arguments that Argue Allowing Abandonment or Renunciation as a Defense

(1) It has little deterrent effect (2) It fails to show whether the defendant will prove dangerous in the future (3) The accused has already completed the crime of attempt

To be Liable as an Accomplice

(1) Knows what the criminal is trying to do (2) Intentionally aid or encourage another person to commit a crime (3) Believe that the aid or encouragement would make the criminal's success likely

personal crime

(Offenses against persons) Also called violent crimes. A crime committed against a person, including (according to the FBI's UCR Program) murder, rape, aggravated assault, and robbery. Also called violent crime. Personal crimes include criminal homicides, kidnapping and false imprisonment, various forms of assault, and rape

D&C

(dominion and control) proves possession when someone is not present at house There is a difference between POSSESSION and OWNERSHIP

Lawful possession of a concealable firearm

...

Dan White

1978 -twinkie defense -got diminished capacity abolished in california

PC 22210: Striking weapons: Sandbag or Sandclub

A bag or device filled with sand

fault exception

A clause, found in many state statutes that codify the defense of duress, when the actor recklessly placed himself in a situation in which it was probable that he would be subjected to duress.

Syndrome

A complex of signs and symptoms presenting a clinical picture of a disease or disorder.

PC 22210: Striking weapons: Sap

A device used to bludgeon

outrageous government conduct

A kind of entrapment defense based on an objective criterion involving "the belief that the methods employed on behalf of the Government to bring about conviction cannot be countenanced."

Double Jeopardy

A second prosecution or a second punishment for the same offense. Double jeopardy is prohibited by the Fifth Amendment of the U.S. Constitution (1) re-prosecuted after acquittal (2) Re-prosecuted after conviction (3) Separate punishment for the same offense

Felony

A serious crime, generally one punishable by death or by incarceration in a state or federal prison facility as opposed to a jail

Specific Intent

A thoughtful, conscious intention to perform a specific act in order to achieve a particular result

Self Defense Requires:

Actual Belief (subjective Stn) Reasonable Belief( Objective Stn) In an Imminent(immediate) Danger

crimen falsi

Any class of crime that involves falsification

12020 P.C.

Dangerous weapon manufacture sale possession. MERE POSSESSION of any of the following is a FELONY.

28. DURESS (MPC & COMMON LAW)

DURESS (JUSTIFICATION OR EXCUSE) Common Law (Justification): A defendant is coerced into committing a crime but has made the right choice in light of the circumstances. Duress is never a defense to murder or when the defendant was at fault for incurring the risk. A defendant must show: 1. Reasonable apprehension of deadly force or grievous bodily injury from a third party. 2. Against him or her (or a close relative). 3. Imminence of the harm. 4. That there was no legal alternative. 5. Reasonable expectation that the threat would be avoided by carrying out the compelled offense. MPC (Excuse): The defendant committed a crime, but it was one in which a reasonable person in his situation would not have been able to resist. Duress is not a defense when the defendant was reckless in placing himself in a position in which it was probable he would be subjected to duress, or when he was negligent in doing so and negligence establishes culpability. A defendant must show: 1. Reasonable apprehension of unlawful force. 2. Against him or any other person. 3. That a reasonable person in his circumstances would have been unable to resist. Limitations to Above: a. Cannot be used as a defense for any offense when D acted recklessly in placing himself situation where it was probable that he would be subject to duress. b. Cannot be used as a defense for any offense when D acted negligently in placing himself in the situation when negligence establishes culpability for the offense; duress still available for R, P, K. Differences: 1. MPC does not limit threat to only deadly force, but rather requires threat of unlawful physical force. 2. MPC does not limit third persons to just close relatives, but rather extends to any strangers. 3. MPC does not retain imminence/legal alternative requirements, but merely uses them as factors for evaluating reasonable firmness. 4. MPC allows duress to operate as a defense against murder. United States v. Haney: Defendant possessed escape equipment to help his friend avoid being murdered in prison. - Rule: Duress consists of (1) threat of imminent infliction upon the defendant or a third person of death or bodily harm; (2) the defendant's well-grounded fear of that threat; and (3) the defendant's and third person's lack of reasonable opportunity to avert the threatened harm. o Court turned the duress defense into a necessity defense in which the defendant was only required to weigh the evils. - Rationale for Relaxing Imminence: Only justifying a minor offense as opposed to murder; prisoners are physically restrained in their movements (somewhat sexist) New Jersey v. Toscano: A mobster threatened the defendant's family to get him to falsify medical reports. - Rule: A per se rule barring duress defense for "non-immediate" harms exclude valid cases where duress should be applied. Instead, consider imminence as only a factor. o When evaluating "reasonable firmness," consider (1) immediacy, (2) gravity of harm threatened, (3) seriousness of crime committed, (4) identity of person endangered, and (5) possibilities of escape, resistance, or rescue. - Rationale: There is no deterrent effect if a state establishes a standard that not even a person of reasonable firmness can meet.

Penumbra

Implied protection

At Sentencing

Insane and mentally retarded convicts may not be executed but may otherwise be punished

Common Law Tradition

Legal tradition, having unified set of laws that were common to all the English

1. Legality (must be a law)

Offensive and harmful behavior is not illegal unless it has been prohibited by law before it was committed

felony

Serious crime-punished by death, state prison, or county jail for a felony term (1 yr)

...

Throwing stars

...

Wallet gun; inside a case resembling a wallet.

What is a Bench Trial?

When a case is tried to a judge

What is Subornation Of Perjury (P.C. 127)?

Willfully procures another person to commit perjury (felony)

Compounding

accepting a reward not to prosecute, a crime in itself.

Ex post facto

after the fact

Taboos

based on trial prhibitions and customs

Bench warrant

issued by judge for contempt or failure to appear.

one who institutes a lawsuit is

plaintiff

Define: False Personation of another in private or official capacity(PC529)

putting yourself in someone elses shoes and committing a crime or civil wrong while pretending to be the person you are impersonating making them liable for the offense (felony)

Common Law

resulting from custom and court decisions

What Does Proving an Accessory's Guilt Require?

(1) A crime has been completed (2) The defendant knew that the crime had been committed (3) The defendant knew that the crime was committed by the individual who was being assisted (4) The assistance was personally given by the individual who committed the crime

How to Consider the Defendant's (Accomplice's) Actions to be Intentional

(1) Commit the acts that in fact gave aid or encouragement (2) Bring about the other party's commission of the offense by committing these acts

Criminal Fault is Imputed to Corporation Based on Two Legal Theories, What are They?

(1) The principle of vicarious liability (2) The identification doctrine

Great Bodily Injury

*Significant or substantial injury * Similar to serious bodily injury * Question for the jury to decide * Does not have to be one injury can be a combo

First Degree murder

-requires perpetrator engaged knowingly and in premeditated manner in killing of another -lesser level or intent: 2nd degree murder or involuntary manslaughter

Insanity Defense Attitudes-Revised Scale (IDA-R)

-scale to assess attitudes of potential jurors and general public toward insanity defense -Skeem, Louden, & Evans (2004) -22 items making statement relevant to insanity defense. "i believe people should be held responsible for actions no mater what their mental condition". agreement-disagreement on a 7-point scale -has been strongly predictive of a community sample of jurors' verdicts in mock insanity cases than host of other measures

Clinical Psychologists

-study and treat various forms of psychological dysfunction and mental illness -crucial to legal process in such cases.

Postpartum mental illness

-what Andrea Yates had -depressed: deepened following birth of each child -anxiety -out of touch with reality

People v. Moran

...

Postpartum Psychosis

00.2% of women experience it -hallucinations, delusions, clinical depressions, and disorganized thought. -onset first 90 days after childbirth women with it 25X more likely to be admitted to psychiatric hospital.

Elements of The Crime of Conspiracy

1) An agreement between two or more people 2) The intention to carry out an act that is unlawful or one that is lawful but is to be accomplished by unlawful means 3) A culpable intent on the part of the defendants

Protections:

1) The Fourth Amendment protection from unreasonable searches and seizures. 2) The Fourth Amendment requirement that no warrant for a search or an arrest be issued without probable cause. 3) The Fifth Amendment requirement that no one be deprived of "life, liberty, or property without due process of law." 4) The Fifth Amendment prohibition against double jeopardy (trying someone twice for the same criminal offense). 5) The Fifth Amendment requirement that no person be required to be a witness against (incriminate) himself or herself. 6) The Sixth Amendment guarantees of a speedy trial, a trial by jury, a public trial, the right to confront witnesses, and the right to a lawyer at various stages in some proceedings. 7) The Eighth Amendment prohibitions against excessive bail and fines and against cruel and unusual punishment.

After the Act: Conviction

1. Burden: Government must prove every element beyond a reasonable doubt 2. Corpus delicti must be proven 3. Guilt must be determined by courts. Bills of attainder by legislatures are prohibited

After the Act: Punishment

1. Ex post facto: No law may increase criminal liability of punishment after the act 2. Double jeopardy: Individuals may not be tried twice or have punishments increased after first jurisdiction 3. Other constitutional limitations exist, such as the Eighth Amendment's prohibition of cruel and unusual punishment

The Act

1. Government may not coerce or excessively encourage the act (entrapment) 2. Government's ability to prevent and monitor conduct is limited by the Constitution and by statutes

criminal negligence

1. Have a legal duty toward the person 2. know of the legal duty 3. be able to perform legal duty 4. fail to perform the duty 5. appear that the negligent act caused the injury.

How do you successfully prosecute a person for the crime of accessory?

1. Principal's crime was a felony. 2. Accessory had actual knowledge that the principal had committed a felony or been charged, or been convicted. 3. accessory either harbored, concealed, or aided 4. with intent of assisting the principal in avoiding arrest, trial, conviction, or punishment.

Before an Act as Criminal

1. Principles of legality (due process): Government must clearly define the act as criminal and set forth its punishment 2. Constitutional limitations: Some actions/inactions may not be criminalized

Condition That are Met for a Company to Be Held Criminally Liable

1. The offense is a minor offense, and the conduct was performed by an agent acting on the corporation's behalf within the scope of the agent's employment. 2. The offense is defined by another statute and is made applicable to corporations. 3. The offense consists of a failure to perform a specific duty imposed on the corporation by law, such as failing to file a tax return. 4. The criminal acts were approved, authorized, permitted, or recklessly tolerated by the board of directors or a high management official acting on behalf of the corporation and within the scope of his or her employment.

Assault

1. UNLAWFUL ATTEMPT 2. PRESENT ABILITY 3. TO DO VIOLENT INJURY* ON THE PERSON OF ANOTHER 4. NOT ACTING IN SELF-DEFENSE OR DEFENSE OF ANOTHER PC 240 MISDEMEANOR PC 241 (C) ASSUALT ON POLICE OFFICER, FIREFIGHTER OR ER PERSONNEL

Assault with a deadly weapon

1. Unlawful ATTEMPT to commit a violent injury 2. By use of a deadly weapon

Bill of Rights

10 amendments to the United States Constitution.

What circuit is California in?

9

Inference

A conclusion drawn from other facts. Juries often infer intent from a defendant's behavior

Strict Liability Crime "absolute liability offenses"

A violation of law for which one may incur criminal liability without fault or intention. Strict liability offenses do NOT require mens rea.

What is a writ of replevin?

A writ which enables a person to get back personal property taken wrongfully or unlawfully and get compensation for resulting losses.

M'Naghten Test

Adopted by Supreme Court in 1864 Overturned in 1978 Reinstated under prop 8 in 1982 Burden of proof is on the defendant to prove with a preponderance of evidence that they were insane at the time the act was committed

The statute of limitations for a misdemeanor is generally: a. 3 years b. 1 year c. 6 years d. 6 months

B) 1 year

...

Ballistic Knife

Short barreled shotgun

Barrel length of less than 18 inches overall length of less than 26 inches any device that can be readily restored to meet onw of the above descriptions any part designed to convert a device into meeting one of the above descriptions

...

Belt Buckle Knife

...

Blackjack

Corpus Delicti

Body of the Crime

Delusional Disorders

Characterized by a dominant delusion (for example, the belief that the person is being persecuted). Delusions may also be associated with schizophrenia

Wheel Conspiracy

Conspirators deal only with a ringleader who serves as a central hub, and not with one another

mala in se

Crimes that are wrong by their very nature.

Which is NOT TRUE with regards to Proposition 115 a. It specifies that a defendant indicted by a Grand Jury need not also receive a preliminary hearing b. It limited Voir Dire to the attorneys c. It permits qualified officers to testify to hearsay at preliminary hearing d. Grants unlimited discovery rights to the defense and none to the prosecution

D) Grants unlimited discovery rights to the defense and none to the prosecution

Jury Instructions

Directions given by a judge to a jury concerning the law of the case

...

Dirk of Dagger: Knife sharpened on both sides (MUST BE CONCEALED).

...

Disguised Firearm; Not immediately recognizable.

Corpus Delicti

Facts that show that a crime has occured. Literary, the "body of crime"

Legal Defense

Factual guilt is immaterial for assertion • Justifications • Excuses • Constitutional or Other Law Violated by Government Outcomes Include • Acquittal • Reduction in Punishment • Exclusion of Evidence • Other

Crimes and public offenses include:

Felony wobblers Misdemeanor Infractions

2. Actus reus (Human conduct)

For a crime to occur there must be an act of either commission or omission by the accused. can't be convicted of a crime due to your status

Greatest impact to criminal law?

Fourth Amendment (searches, seizures, warrants), Fifth Amendment (self-incrimination).

_____ is a federal regulation that requires health care professionals to protect the privacy and confidentiality of patients' health information.

HIPAA

In the health care field the acronym HIPAA stands for.

Health Insurance Portability And Accountability Act

What two things are never needed to establish Corpus Delicti?

Identity of the perpetrator and Motive

Ignorance of The Fact vs. Mistakes of Fact

Ignorance of fact refers to a lack of knowledge of some fact relating to the matter at hand, while mistake of fact refers to a misinterpretation or misunderstanding of the facts at hand. Both can be defenses to a criminal charge. As a defense, both ignorance of fact and mistake of fact may negate the mens rea required for a specific offense.

vicarious liability

In California, where two or more people agree to kill as part of their plan to carry out the crimes and one of them does, all are guilty of murder.

Conduct

In the criminal law, behavior and its accompanying mental state

Intent

Is the purpose or resolve to do an act.

What are the three fundamental principles of our legal system?

Jurisdiction Hierarchical Structure of Courts Stare Decisis

implied malice

Killing shows an "abandoned and malignant heart" (i.e., cold-hearted or a disregard for other's safety)

What is Mandatory Authority?

Law from within the governing jurisdiction (primary authority). Binding.

Comparison between Insanity Tests

M'NAUGHTEN Defendant was suffering from mental disease that caused defendant to not know nature and quality of act or that act was wrong. IRRESISTIBLE IMPULSE Expanded M'Naughten to cover individuals who knew their acts were wrong but could not control their behavior because of disease of mind. MODEL PENAL CODE Incorporated both M'Naughten and irresistible impulse and expanded them by broadening to include individuals with substantial (as opposed to total) impairment and individuals who "appreciated" (as opposed to "knew") that their actions were wrong.

Nature of the offense

Malum Prohibita: "wrong [as or because] prohibited" Malum in se:wrong or evil in itself.

16. MISTAKE OF LAW (COMMON LAW & MPC)

MISTAKE OF LAW The defendant misperceives the definition or meaning of the law. Distinguishing in Unclear Cases (between Mistakes of Facts or Mistakes of Law): Determine all the things the defendant thought he believed/knew. Then ask if a fact-finder could discover, in that set of things, enough evidence to infer that the actus reus (conduct, circumstance, result) has been satisfied? - If the actus reus has been satisfied, then it is a mistake of law. EX. a. D possesses oregano thinking its Marijuana: Mistake of Fact. b. D possesses Marijuana knowing that its Marijuana but not knowing it's a controlled substance: Mistake of Law. General Rule: Mistake of law is no defense. Rationale: Criminal law's goal is deterrence. Denying the defense encourages conformance to common social duty. Uncertainty of the law's boundaries incentivizes people to be good and discourages people learning the law to game the system. EXCEPTIONS 1. Ad Hoc Statutory Exception (All Jurisdictions): Mistake of law is a defense when a statute includes knowledge or awareness of the law as an element of the offense (e.g., larceny) 2. Mistake of Non-Criminal Law (Most Jurisdictions): Defendant has a mistake of law defense if he violates a law that informs the meaning of the criminal statute at hand, but is not the statute under which he is charged. Distinction between specific intent offenses (defense allowed) and general intent/strict liability offenses (defense not allowed) is important. 3. Reliance (MPC Jurisdictions): If D reasonably relies on an OFFICIAL interpretation of the law (statute, judicial decision, official interpretation by a relevant public officer, etc.) while acting, supported by a preponderance of the evidence, D has a mistake of law defense. 4. LAMBERT Notice Exception (MPC jurisdictions, never applied): If the statute defining the offense is not known or made reasonably available prior to the conduct alleged, and D had no reason whatsoever to suspect that his conduct was criminal, there is a mistake of law defense. 5. Reasonable Mistake of Law (NJ, minority rule, extends to MPC): If D makes mistake of law reasonably and in good faith where discover the meaning of a statute through all available means in an effort to do the right thing, there is a mistake of law defense (e.g. ensuring voting rights but preventing voter fraud). Idaho v. Fox: Defendant was convicted of possession of a controlled substance but claimed he didn't know it was a controlled substance. - Rule: Where knowledge of the illegality of a substance is not an element of the offense, there is no mistake of law defense. New York v. Marrero: Defendant was charged with possession of a gun in a social club but claimed he believed he was exempt under the statute. - Rule: Where the mistake is unreasonable, there is no mistake of law defense. Hopkins v. State of Maryland: Defendant was convicted of soliciting marriages but claimed the state's attorney advised him they were legal. - Rule: The advice of counsel, even though followed in good faith, is no excuse to violate the law because advice of a public official is not an official interpretation of the law. o Would permit that advice to become law. Iowa v. Striggles: Defendant installed a candy machine in his store which the municipal court did not consider to be a gambling machine. The Supreme Court of Iowa later deemed the machine a gambling device. - Rule: Defendants are permitted to rely on a court's interpretation of the law, but that interpretation must come from the highest court. LAMBERT v. California: Defendant was convicted for failing to register as a felon in L.A., in violation of a city ordinance. - Notice Exception RULE: When a defendant did not know of a duty to register and when there was no proof in the probability of such knowledge, a defendant's conviction does not comport with due process. o Distinction between passive and active conduct is important. (Mistake of "Non-Criminal" Law) Long v. State of Delaware: Defendant consulted with his attorney over his out-of-state divorce so he could remarry in the state. He was convicted of bigamy (general intent offense). - Rule: Mistake of law is generally no defense for general intent crimes because of deterrence justifications. However, mistake of non-criminal law when a defendant has exerted good faith effort and diligence to abide with the law is an excuse for bigamy because it is a mistake of marriage law, which is confusing (specific intent crimes). Vermont v. Woods: Defendant witnessed fiancée's divorce proceedings and then married him. They then returned to Vermont and were charged with adultery. - Rule: Rejection of mistake of non-criminal law defense for general intent crimes because otherwise the other laws would lose effect if they could be circumvented.

Carrying a loaded firearm

Misdemeanor may be a felony if prior conviction for felony, crime against property, active gang member, is not the registered owner through DOJ of a concealable firearm

Mistake of Fact

Misinterpretation, misunderstanding, or forgetfulness of a fact relating to the situation at hand; belief in the existence of a thing or condition that does not exist.

Reasonable Mistake

Misinterpretation, misunderstanding, or forgetfulness of a fact relating to the situation at hand; belief in the existence of a thing or condition that does not exist.

9. MISTAKE OF FACT (COMMON LAW)

Mistake or ignorance of a certain fact may show a lack of mens rea Specific Intent: (1) good faith mistake General Intent: (1) material mistake was reasonable and (2) mistake was made in good faith.

misdemeanor

Moderately serious crime-punished by county jail and/or fine.

Tear Gas/OC Spray

Must be at least 16 years of age or older.

No breaking and entering in California law

No breaking and entering in California law

Is the elements of the crime and corpus deliciti the same thing?

No. Not the same.

...

Nunchaku; EXCEPTION (being sold or used in a school which is licensed to use such weapons as self defense)

Battery on a peace officer with injury

PC 243 (c) Wobbler Defendant must reasonably know that the individual is a peace officer

what is resisting a police officer (148(A)(1))

R.D.O. - resisting, delaying or obstructing - resisting delaying or Obstructing a Peace officer or EMT from performing his or her duty (misdemeanor)

SPECIFIC INTENT

Requires a specific state of mind. This state of mind must be proved with the other elements of the crime.

...

Sawed-off Handgun; Barrel less than 18 inches and overall less than 26 inches. EXCEPTION (Used in motion picture set or teaching institution)

...

Special ammo, teflon bullets.

Who creates Executive Orders?

The President

transferred intent

The defendant can be liable where he intends the harm caused but harms a different victim.

A rout is simply defined as the preparatory stage of a riot

True

Deadly Weapon

Whenever an object has the capacity for being used to inflict death or great bodily injury

mala prohibita

Wrong because the crime is prohibited by statute

The withdrawal of a physician from the care of a patient without reasonable notice of such discharge from the case by the patient is

abandonment

Public Policy

best for majority, custom

Abandonment of attempt when a defense

if person intends to commit crime but BEFORE any act freely abandons the idea.

Sovereign

power to act, king.

What Does Inchoate Crimes Include

(1) Attempts (2) Solicitation (3) Conspiracies

Refined Elements That Comprise The Conceptual Essence of Criminality

(1) causation (2) a resulting harm (3) the principle of legality (4) necessary attendant circumstances

When A Jury Returns a Finding of Guilty but Mentally Ill

(1) every element necessary for a conviction has been proven beyond a reasonable doubt, (2) the defendant is found to have been mentally ill at the time the crime was committed, and (3) the defendant was not found to have been legally insane at the time the crime was committed. The difference between mental illness and legal insanity is a crucial one.

Two Most Common Inducements to Entrapment Activities

(1) false representation by enforcement agents that are calculated to induce the belief that the illegal behavior is not prohibited and (2) the use of inducements to crime that are so strong that a person of average will and good intent could not resist

Conspiracy

*2 or more persons *Specific intent to AGREE to commit a crime *Overt Act - Committed by any one of the parties to further the object of the conspiracy

Attempt

*Act done by a perpetrator *Specific intent of committing a crime *Direct & ineffectual act toward the commission of a crime

Accomplice testimony

*Conviction cannot be made based solely on accomplice testimony *Must have corroborated evidence

Overt Act

*Not necessary that any one defendant be proven guilty for overt act. As long as he/she was a conspirator at the time the overt act was committed *Only one overt act need be proven *Object of conspiracy does not need to be complete as long as the overt act has been completed *Any act that goes beyond planning or agreeing to commit the crime

Accomplice (PC 1111)

*Principle of a crime *A person who knowingly and voluntarily with common intent with the principle offender unites in the commission of a crime *One who is liable to prosecution for identical offense charged to the defendant in a trial in which cause for testimony of the accomplice is given

- Preventing future criminal behavior

-Deterrence of crime

Malingering (Insanity)

-Possible to fake/exaggerate psychological problems so that trier might find defendant insane. ex: Kenneth Bianchi

- Penalizing criminal behavior

-Punishment. Least amount of punishment to achieve compliance.

Affirmative Defense

-When defendant bears burden of proof for a defense at a trial

Totality of circumstances

1. APPEARANCE (UNDER INFLUENCE, DESCRIPTION) 2. ACTIONS (FLIGHT ALONE NOT ENOUGH) 3. TRAFFIC VIOLATIONS 4. PRIOR KNOWLEDGE OF THE PERSON 5. SUSPECT DEMEANOR DURING THE DETENTION 6. TIME OF THE DAY 7. AREA OF THE STOP 8. POLICE TRAINING AND EXPERIENCE 9. SOURCES OF INFORMATION

9. MENS REA (COMMON LAW)

1. DEFAULT: General Intent (OBJECTIVE) a) Objective standard that requires a showing of criminal negligence or a blameworthy state of mind; D only needs to intend his actions, but not the resulting offense (e.g. assault). b) Leonard: "Given the circumstances that the defendant was aware of, what risks should the defendant have known or been aware of - i.e. what should s/he have foreseen - and how should s/he have acted?" c) Holmes: "Criminal liability arises out of such conduct as would be blameworthy in the 'man of ordinary intelligence and reasonable prudence.'" d) Consequences: (1) Do not need to show the actual state of mind possessed by the defendant (2) Concerned about the risks of which "reasonable" person should have been aware (3) Equivalence — "Criminal negligence" ***General intent is the default mens rea level under common law, unless the statute has clear specific intent language in it (e.g., "deliberate premeditation" or "knowingly"); e) If there is specific mens rea language, then you're likely dealing with specific intent f) If an offense has a strong tradition in the Common Law of requiring certain mens rea, then go with the traditional rule unless the statute says otherwise. g) Lambert Rule: Individuals must be alerted that their conduct crosses the line into criminal conduct before D can be convicted. h) Morissette Rule: Common law background and other established mens rea doctrines can help guide classifications of offenses (see Jackson's interpretive method). f) Smith Rule (Some Jurisdictions, Including Mass.): Even language suggesting specific intent (e.g., "knowingly") may still be read objectively under the natural and probable consequence rule. MORE OBJECTIVE TO MORE SUBJECTIVE: 1. "Wicked" Faulkner Prosecutor 2. Holmes Crim Neg 3. Cunningham Risk Consciousness 4. Morisette Intent

Assault by force likely to produce great bodily injury

1. Unlawful ATTEMPT to commit a violent injury upon another 2. with force likely to produce great bodily injury 3. No one needs to have been injured *No strike unless GBI

Exhibiting a deadly weapon

1. drew or displayed in the immediate presence of another 2. did so in a rude, angry or threatening manner 3. or used in fight or quarrel 4. loaded or unloaded *misdemeanor

One aids or abets a crime when he/she:

1. has knowledge of the unlawful purpose of the perpetrator 2. Has the intent or purpose of encouraging or facilitating the commission of a crime 3. By act, aids, encourages, promotes or instigates the commission of a crime

Definition of insanity:

1. knowing the nature and quality of his act; oro 2. understanding the nature and qualityof his act; or 3. distinguishing what is legal right form what is wrong or; 4. Distinguishing what is morally right from what is morally wrong The defendant has legal burden....

Sexual battery

1. person touched an immediate part of alleged victim; 2. the touching was against 3. the touching was done with the SPECIFIC intent to cause sexual arousal, g,ratification, or abuse THIS IS SPECIFIC INTENT

How many circuits are numbered?

11

How many circuits are there in the US?

13

Possession of a deadly weapon with intent to assault another person

17500 PC Classification Misdemeanor Felony if intent to commit a Felony 25800(a) PC Every person; Having upon him or her person any deadly weapon with intent to assault another

Insanity Defense Reform Act (IDRA)

1984 -ALi standard abandoned. -required there be presumption of sanity and that defendants prove "by clear and convincing evidence" that they were insane at the time of the crime (affirmative defense)

Discharging a firearm at an inhabited or occupied dwelling, vehicle, or aircraft

246 PC Felony any person who discharges a firearm willfully or maliciously at inhabited dwelling house inhabited house car inhabited camper occupied building occupied motor vehicle or aircraft

Unlawful discharge of a firearm

246.3 PC Felony Any person who except as otherwise authorised by law willfully discharges a firearm in a grossly negligent manner which could result in injury or death to a person

Carrying a firearm in a public place or within a vehicle

25850(a) PC Classification Misdemeanor Felony if prior, gang, stolen, no lawful pos, crime against property or person, not listed with DOJ as RO Any person who; Carries a loaded firearm; on person or in vehicle

PC 20510: Unrecognizable blade weapons: Cane sword

A cane, swagger stick, stick, staff, rod, pole, umbrella, or similar device with a concealed blade that may be used as a sword or stiletto

PC 22210: Striking weapons: Blackjack

A hard device usually consisting of leather-enclosed metal, with a strap or spring shaft for a handle

voluntary manslaughter

A homicide in which the intent to kill was present in the mind of the offender, but malice was lacking. Most commonly used to describe homicides in which the offender was provoked or otherwise acted in the heat of passion.

mistake of law

A misunderstanding or misinterpretation of the law relevant to the situation at hand. mistake of law may be a valid defense to a criminal charge if made in good faith under circumstances involving a bona fide attempt to ascertain the meaning of the law through reliance on a public official who is in a position to interpret the statute or through the use of appropriate legal counsel. In some jurisdictions, a defendant may effectively raise such a defense where "before engaging in the conduct, the defendant made a bona fide, diligent effort, adopting a course and resorting to sources and means at least as appropriate as any afforded under our legal system, to ascertain and abide by the law, and where he acted in good faith reliance upon the results of such effort."

Pax Romana

A peace imposed by the military might of Rome

PC 20710: Blade weapons: Shobi-zue

A staff, crutch, stick, rod, or pole, concealing within it a knife or blade which may be exposed by a flip of the wrist or by a mechanical action

PC 22210: Striking weapons: Leaded cane

A staff, crutch, stick, rod, pole, or similar device unnaturally weighted with lead

Specific Intent

A state of mind that: denotes design determination to break the law specific intent that is written in the statute * must prove intent

irresistible impulse test

A test for insanity that evaluates defense claims that at the time the crime was committed, a mental disease or disorder prevented the defendant from controlling his or her behavior in keeping with the requirements of the law.

Social-Orders (victimless crimes)

A type of criminal law violation in which parties to the crime willfully (even joyfully) participate and in which the element of harm seems remote. Example: drug use, illegal forms of gambling, prostitution, pornography Areas afflicted with chronic prostitution, drug use, sexual deviance, and illegal gambling usually also experience property values falling, family life disintegrating, and other more traditional crimes increasing as money is sough to support the "victimless" activities

guilty but mentally ill (GBMI)

A verdict, equivalent to a finding of guilty, that establishes that "the defendant, although mentally ill, was sufficiently in possession of his faculties to be morally blameworthy for his acts."

Infractions

A violation of a state statute or local ordinance punishable by a fine or other penalty, but not by incarceration. Also called Summary Offense

Which one of the following offenses is incapable of being completed and thus cannot be attempted?

A) Assault with a Deadly Weapon through a window unknowingly using an unloaded gun. B) Theft from an empty pocket believed to contain money. C) Shooting through a wall to kill one who is no longer there. D) Mayhem using red pepper. Answer: (A) Assault with a Deadly Weapon through a window unknowingly using an unloaded gun.

There are several components to PC 538d simulating or impersonating an officer, the components are:

A) arresting, detaining or threatening to arrest or detain. B) intimidating others. C) searches any person, building or property of another. D) obtains money, property or anything of value. E) all of the above are correct. Answer: (E) All of the above are correct.

Asking, giving, or accepting anything of value to a specific class of people, such as police, with the specific intent to influence any decision, etcetera is:

A) embezzlement. B) fraud. C) perjury. D) bribery. Answer: (D) Bribery

Incomplete crimes, or attempts, are also referred to as _________crimes.

A) inchoate B) unfinished C) aborted D) abandoned Answer: (A) Inchoate

Which statements are true? Receiving or agreeing to accept something of value to conceal or withhold evidence, is referred to as:

A) misprision of treason. B) abrogation. C) subornation. D) compounding a crime. Answer: (D) Compounding a crime

If "A" and "B" plan a burglary and "A" purchases tools to be used in the crime:

A) only "A" is guilty of conspiracy. B) both are guilty of attempted burglary. C) both are guilty of conspiracy at this point. D)neither are guilty of any crime at this point. Answer: (C) both are guilty of conspiracy at this point.

13. NEGLIGENCE AND THE GRADING OF HOMICIDE: COMMON LAW MANSLAUGHTER

A. Voluntary Manslaughter - Homicides in which the defendant intended to kill another but did so in the heat of passion caused by adequate provocation B. Involuntary Manslaughter - Homicides in which the defendant was reckless or criminally negligent with respect to the other's death. (1) Actus reus - cause death (2) Mens reus - recklessness or criminal negligence (e.g. gross deviation from the standard of care of a reasonable person) (a) Recklessness and criminal negligence are synonyms in the common law; separate in the MPC (b) Objective as opposed to subjective standard C. Exception: Juries may impute homicidal intent when the defendant is aware of certain facts which lead to the natural and probable consequence of death (1) Merely need to show that the defendant was aware of certain facts

Reckless (reckless behavior) Mens Rea

Activity that increases the risk of harm. It exists more in the form of probability than certainty. Recklessly means, with respect to a result or to a circumstances described by a statute defining an offense, that a person is aware and consciously disregards a substantial and unjustifiable risk that the result will occur or that the circumstances exists Did not know for sure, but considered chance and acted anyway. Example: Maybe if I had not been going 20 mph over the speed limit I would have seen the stop sign or drinking while driving

Mala Prohibita

Acts that are considered "wrongs" only because there is a law against them, Without a statute specifically proscribing them, mala prohibita offense might not be regarded as "wrong" by a large number of people. Mala porhibita offenses often include the category "victimless crimes," such as prostitution, drug use, and gambling, in which a clear-cut victim is difficult to identify and whose commission rarely leads to complaints from the parties directly involved in the offense

Possession crime

Actual possession: defendant knowingly exercises direct physical control over an object.

PC 24610: Undetectable firearms

After removal of grips, stocks, and magazines, and the weapon is not detectable by the security exemplar by walk through metal detector • any major component of the weapon when subjected to the inspection by the type of x-ray machines commonly used at airports, does not generate an image that does not accurately depict the shape of the component

Mood Disorders

Also known as affective disorders. Characterized by emotional extremes. Major depression and manic-depressive (bipolar) disorders are included

Nullum Crimen Sine Lege

Also known as the principle of legality. "There is no crime if there is no statute"

Mail and Wire Fraud

Among the most potent weapons against white-collar criminals are the federal laws that prohibit mail fraud and wire fraud. These laws make it a federal crime to devise any scheme that uses U.S. mail, commercial carriers (FedEx, UPS), or wire (telegraph, telephone, television, the Internet, e-mail) with the intent to defraud the public. These laws are often applied when persons send out advertisements or e-mails with the intent to fraudulently obtain cash or property by false pretenses. The maximum penalty under these statutes is substantial. Persons convicted of mail, wire, and Internet fraud may be imprisoned for up to twenty years and/ or fined. If the violation affects a financial institution or involves fraud in connection with emergency disaster-relief funds, the violator may be fined up to $1 million, imprisoned for up to thirty years, or both.

affirmative defense

An answer to a criminal charge in which a defendant takes the offensive and responds to the allegations with his or her own assertions based on legal principles. Affirmative defenses must be raised and supported by the defendant independently of any claims made by the prosecutor. Affirmative defenses include justifications and excuses Since most criminal cases are complex, it is difficult to be certain whether a specific defense will be accepted by a judge or jury in a given instance. Moreover, those charged with crimes may be particularly inventive in their efforts to "bend" traditional defenses in order to apply them to their cases. Other defendants may offer creative excuses that have not previously been heard in American courts. Still others may attempt to apply traditionally accepted defenses under novel circumstances. All this makes it difficult to generalize about the applicability of any particular defense to a given set of circumstances If defendants meet the burden of production, they also meet burden of persuasion, meaning they have to prove defenses by a preponderance of the evidence, defined more than 50%

Felony

An offense punishable by death or by imprisonment in a state or federal prison for more than one year

Inchoate Crime

An unfinished crime that generally leads to another crime. Also, a crime that consists of actions that are steps toward another offense. Also called anticipatory offense

Administrative Regulations

Another source of law. Legitimately promulgated regulations have the full authority of legislation. Administrative regulations can be penal and can result in incarceration, fines, and other punishments

Switch Blade Knife 653k P.C.

Any Blade OVER 2 INCHES is a MISDEMEANOR

Crime

Any act or omission in violation of penal law, committed without defense or justification, and made punishable by the state in a judicial proceeding

provocative act

Applies when someone commits an act that provokes someone into killing someone else.

Anxiety Disorders

Characterized by apprehension, dread, and fear and sometimes associated with particular situations or types of situations but may also exist independently of such situations. Phobias, generalized anxiety disorder, panic attacks, and obsessive-compulsive disorders are subtypes

Psychosexual Disorders

Characterized by sexual arousal via unusual objects (fetishism) or situations or by sexual dysfunctions

What are the intermediate courts of appeal called in the Federal Court System?

Circuit Courts of Appeals

What is Secondary Authority?

Commentary about the law and is not binding on anyone. Secondary Authority can provide an overview of an area of law or a critique of the law.

Result Crimes

Consist of criminal conduct that causes a criminal harm. Those in which the actus reus is defined in terms of prohibited consequences irrespective of these are brought about, i.e. causing death (murder). This differs from conduct crime. Includes actus reus, mens rea, concurrence, causation, harm

5. CONDUCT REQUIREMENT (ACTUS REUS)

Crime requires some conduct (affirmative act, possession, and omission). Limiting Principle: "Thoughts alone" not sufficient. Note: Line between "no conduct" or "thoughts alone" and "conduct" can be very unclear and vary based on the nature of the crime. Doe v. City of Lafayette, 2003: pedo in the park. Holding: thoughts alone are not enough. D must have done some conduct before the state can punish.

White Collar Crime

Crimes occurring in the business context are popularly referred to as white-collar crimes, although this is not an official legal term. Ordinarily, white-collar crime involves an illegal act or series of acts committed by an individual or business entity using some nonviolent means to obtain a personal or business advantage. Usually, this kind of crime takes place in the course of a legitimate business occupation. Corporate crimes fall into this category. Certain property crimes, such as larceny and forgery, may also be white-collar crimes if they occur within the business context. The crimes discussed next normally occur only in the business context.

Employment Fraud

Cyber criminals also look for victims at online job-posting sites. Claiming to be an employment officer in a well-known company the criminal sends bogus e-mail messages to job seekers. The messages ask the unsuspecting job seekers to reveal enough information to allow for identity theft. As the unemployment rate has remained high, cyber criminals have found many opportunities for employment fraud.

Crimes in California can be divided into all but one of the following categories: a. Felonies b. Misdemeanors c. Infractions d. Indictments

D) Indictments

Public Laws

Dealt with the organization of the Roman state, its senate, and governmental offices

general intent

Defendant is aware that she is acting in a certain way: "doing the act" (e.g. shoving the person in front of them in line). Intent is inferred merely from doing the act.

Criminal Procedure

Defines the process that may be used by law enforcement, prosecutors, victims, and courts to investigative and adjudicate criminal cases. As an academic field, criminal procedure also includes the study of the Constitution's role in the process

Duress

Duress exists when the wrongful threat of one person induces another person to perform an act that he or she would not otherwise have performed. In such a situation, duress is said to negate the mental state necessary to commit a crime because the defendant was forced or compelled to commit the act. Duress can be used as a defense to most crimes except murder. Both the definition of duress and the types of crimes that it can excuse vary among the states, however. Generally, to successfully assert duress as a defense, the defendant must reasonably have believed that he or she was in immediate danger, and the jury (or judge) must conclude that the defendant's belief was reasonable.

accessory

Every person who, after a felony had been committed, harbors, conceals, or aids a principle in such felony, with the intent that said principle may avoid or escape from arrest, trial, conviction or punishment, having knowledge that said principle has committed such felony, KEY WORDS in this penal code AFTER, FELONY, INTNET, KNOWLEDGE This only applies to FELONIES

Accessory

Every person who, after a felony has been committed, harbors, conceals, or aids a principal in such felony, with the intent that said principal may avoid or escape from arrest, trial, conviction, or punishment, having knowledge that said principal has committed such felony or has been charged with such felony or convicted thereof, is an accessory to such felony.

148.3(a) PC

Falsely reporting a crime, emergency. misdemeanor; unless likely to cause death of GBI.

148.5 PC

Falsely reporting a crime. misdemeanor.

rape on high seas is what type of case?

Federal

What laws does the US Federal Government sovereign jurisdiction enact?

Federal Law

3. Causation (human conduct must cause harm)

For a crime to have been committed, there must be a casual relationship between and act and the harm suffered

What are the 4 types of criminal intent?

General Specific Transferred Negligence

Self Defense for Homicide: HOMICIDE

Great bodily injury or Death killing in self defense is OK and not unlawful when person actually believes they are in imminent danger AND it was necessary to kill under circumstance fear of death or harm is not sufficient, to justify the circumstances must be that a reasonable person would act the same in that situation, it must not be under fear alone that they are acting. danger must be apparent present immediate

A physician must have the patient's permission in writing to reveal any confidential information except for which one of the following?

Gunshot wound

He fondled the females vagina, not the dudes parts that proves intent. He wasn't fondling the pole THIS IS HIS SPECIFIC INTENT Battery is a general intent crime

He fondled the females vagina, not the dudes parts that proves intent. He wasn't fondling the pole THIS IS HIS SPECIFIC INTENT Battery is a general intent crime

4 reasons why southerners created segregation (jim crow Laws) after civil war

History of slavery, science (whites were supiror), economics( kept black wages low), politics

Canon of Presumptive Constitutionally

Holds that if two interpretations of a statute are possible, one constitutional and another not, the statute is to be interpreted as constitutional

Consensual Encounter

IF A PERSON FEELS THAT THEY SHOULD COOPERATE, THEN IT IS A CONSENSUAL ENCOUNTER. IF A PERSON FEELS THEY MUST COOPERATE THEN................

Carrying a loaded firearm Felony exceptions

If the person carrying the firearm: • has a prior felony conviction PC 25850(c)(1), or of any crime made punishable by a prevision listed in PC 16580 • knew or reasonably should have known the firearm was stolen PC 25850(c)(2) • is an active participant in a criminal street gang PC 25850(c)(3) • is not in lawful possession of the firearm PC 25850(c)(4) • has been convicted of a crime against a person or property, or of a narcotics violation PC 25850(c)(5) • is not listed with the Department of Justice as the registered owner of the pistol, revolver, or firearm capable of being concealed upon the person PC 25850(c)(6)

Defenses to Attempt

Impossibility and abandonment

The Collective Knowledge Doctrine

Imputes to a corporation the knowledge of all employees, thereby imposing liability on companies even when no single employee has sufficient knowledge to be liable

Problems with Constitutionality

In 2005, the United States Supreme Court held that certain provisions of the federal sentencing guidelines were unconstitutional. Essentially, the Court's ruling changed the federal sentencing guidelines from mandatory to advisory. Depending on the circumstances of the case, a federal trial judge may now depart from the guidelines if she or he believes that it is reasonable to do so.

In CAL attempt is specific intent (he had that) and DIRECT ineffectual act to commit crime (he had that) according to CAL LAW yes its attempt and abandonment not possible it was too late for that

In CAL attempt is specific intent (he had that) and DIRECT ineffectual act to commit crime (he had that) according to CAL LAW yes its attempt and abandonment not possible it was too late for that

Local Laws

In addition to state and federal crimes, municipalities are often empowered by state law to define and punish crimes, usually misdemeanors and infractions. Local laws are commonly known as ordinances

Burglary 459 for commercial establishment:

Intenet to steal something and have entered the establishment (that's all that needed for crime to be committed)

involuntary intoxication

Intoxication that is not willful. Involuntary intoxication may serve as a defense if it either creates in the defendant an incapacity to appreciate the criminality of his or her conduct or creates an incapacity to conform his or her behavior to the requirements of the law.

A violation of a person's right not to have his or her name, photograph, or private affairs exposed of made public without giving consent is

Invasion of privacy

Complicity

Involvement in crime either as a principal or as an accomplice. The term also refers to activities of conspirators and may therefore be taken to mean conduct that is intended to encourage or aid another person to commit a crime, to escape, or to avoid prosecution

Tort

Is a "private wrong", such as failure to comply with the terms of a legal contract, etc. The civil action for a tort is designed to obtain redress(payment or reimbursement).

infraction

Least serious crime-punished by fine; no right to trial by jury or public defender at public expense.

examples of Omission

Legal duty to act: PC 273a: endanger the health of child. (your child) PC 150: refusing to assist a peace officer PC 12035: criminal storage of a firearm Failing to file a tax return Failing to act in a morally obligation is not a crime. There is a difference between a moral duty and legal duty.

The Branches of The Government

Legislative Executive Judicial

What are the Three Branches of the Government?

Legislative Branch Executive Branch Judicial Branch

Misdemeanors

Less serious offenses, generally thought of as punishable by less than a year's incarceration

Strict Liability

Liability without fault or intention. Routine traffic offenses are generally considered strict liability offenses that do not require intent and may even be committed by drivers who are not consciously aware of what they are doing. Statutory rape provides another example of the concept of strict liability, with one person under the age of legal content. Another example of strict liability is selling alcohol to minors

State Crimes

Many states have also enacted laws that punish behavior as criminal without the need to show criminal intent.

MENTAL INCOMPETENCY

Mental competency is an important concept in the law, particularly criminal law. A defendant's mental state is assessed using different legal tests at various points in the criminal justice process. The two most significant points are at trial and at the time of the commission of the crime.

Merely preparing to commit crime (abandonment is still possible) direct but ineffectual act done towards crime (abandonment is not possible) GUILTY OF ATTEMPT

Merely preparing to commit crime (abandonment is still possible) direct but ineffectual act done towards crime (abandonment is not possible) GUILTY OF ATTEMPT

PC 29655: Firearm Possession by a minor (Felony)

Minors may not possess a concealable firearm or live ammunition unless they are: actively engaged in, or going to or from, a lawful, recreational sport the nature of which involves the use of a firearm at least 16 years of age with written consent from a parent or legal guardian accompanied by a parent, legal guardian, or responsible adult with written consent from the parent while in possession of the firearm or ammunition

Possession of a deadly weapon with intent to assault

Misdemeanor possession of a deadly weapon with the intent to assault another person

Tear gas weapon or tear gas weapon

Misdemeanor any person, firm, or corporation who knowingly possesses sells offers for sale transports any prohibited tear gas weapons

7. Punishment

Must be a provision in the law calling for punishment of those found guilty of violating the law

25. NECESSITY (JUSTIFICATION)

NECESSITY (JUSTIFICATION) A necessity defense requires the Court to determine what the proper legislative policy is in a particular case when it is evident that it was not a situation the legislature contemplated. This requires the court to weigh the costs and benefits of allowing an exception to a statute. Necessity differs from civil disobedience in that civil disobedience involves explicitly opposing a statute to dramatize its immorality or effect. Civil disobedience is no defense to the criminal law. Common Law Rule: D must show the following: 1. The evil or harm one tried to prevent was objectively greater than the evil or harm that the relevant criminal statute sought to prevent. 2. The evil or harm was imminent. 3. The defendant reasonably expected the action to be effective in preventing the harm. 4. There was no legal alternative available to prevent the harm. 5. The defendant was not at fault for bringing about the emergency. MPC Rule: D must show the following: 1. The evil or harm one tried to prevent was objectively greater than the evil or harm that the relevant criminal statute sought to prevent. 2. The defendant subjectively believed that his conduct was necessary to avert the harm. 3. The defendant was non-negligent or non-reckless, depending on the offense's underlying culpability level, in appraising the necessity of his conduct. 4. No statute deals with the specific circumstance. 5. A legislative purpose to exclude the justification claimed does not otherwise plainly appear. Differences between MPC, Common Law: 1. The MPC does not require imminence, effectiveness, or the lack of a legal alternative, but these factors can be considered in evaluating whether the actor's conduct was actually necessary. 2. MPC also has an objective quality - if the defendant negligently or recklessly decides his conduct is necessary, then justification is not available for crimes that have negligence or recklessness as culpability levels, respectively. 3. The MPC incentivizes individuals to do the right thing where the defendant was negligent or reckless in creating the emergency. Common law completely removes the defendant's defense if he were negligent at all in creating the risk. Pennsylvania v. Markum: Defendants were arrested after unlawfully occupying and damaging an abortion clinic. - Rule: Justification is not available in situations where the legislature or court has legalized conduct that some people believe is a "public disaster" (i.e. if it's legal, it can't be immoral). o Too broad a generalization regarding whether something can never be an evil just because it is legal. New York v. Archer: Defendants were arrested after occupying an abortion clinic to prevent abortions. - Rule: Courts should consider the harm the legislature was seeking to prevent when it passed the statute when determining scope of necessity defense. This requires the court to imaginatively reconstruct what the legislature would have done when presented with the situation in question (especially the costs v. benefits). o Civil disobedience is no defense to the criminal law. Washington v. Diana: Defendant smoked pot to relieve his multiple sclerosis. Washington prohibited marijuana use but allowed exceptions for medical reasons and provided administrative means to add more exceptions. MS was not listed as an exception. - Rule: Medical necessity exists if fact-finder can determine that (1) the defendant reasonably believed criminal conduct was necessary to minimize the disease's effects; (2) the benefits derived from criminal conduct outweighed harm prevented by drug law; and (3) no other drug was as effective. Washington v. Moe: Defendants stole groceries to avoid starvation. - Rule: Economic necessity is never a defense because property rights are fundamental to an ordered society and other institutions exist to relieve the problem. The Queen v. Dudley & Stephens: Sailors lost at sea ate their companion, who had ingested seawater and was likely to die within days. - Rule (Minority Rule): There is no necessity to preserve one's own life. To allow otherwise would require courts to weigh the value of lives and create an excuse for unbridled passion and atrocious crime. Conjoined Twins Case: Twins shared an aorta. One of the twins would eventually kill her sister if they remained attached, but the other twin would die almost immediately if the twins were separated. - Rule: If (1) it is impossible to save X without killing Y; (2) Y will inevitably kill X and Y in a short amount of time; and (3) X is capable of living an independent life but for Y's existence, then killing Y to save X is justified. o Potentially less justifiable when the harm takes longer to manifest itself or the parties involved aren't infants.

The Hammurabic Code (ancient laws)

Named after the Babylonian king Hammurabi (1792-175 BC), specified a number of property rights, crimes, and associated punishments. Hammurabi's law spoke to issues of ownership, theft, sexual relationships, and interpersonal violence. Its major contribution was that it routinized the practice of justice in Babylonian society by lending predictability to punishment. It had, however, little impact on the development of Western legal traditions

Obtaining Goods by False Pretenses

Obtaining goods by means of false pretenses is a form of theft that involves trickery or fraud, such as using someone else's credit-card number without permission to purchase an iPad. Statutes dealing with such illegal activities vary widely from state to state. They often apply not only to property, but also to services and cash. Sometimes, a statute consolidates the crime of obtaining goods by false pretenses with other property offenses, such as larceny and embezzlement, into a single crime called simply "theft." Under such a statute, it is not necessary for a defendant to be charged specifically with larceny, embezzlement, or obtaining goods by false pretenses. Petty theft is the theft of a small quantity of cash or low-value goods. Grand theft is the theft of a larger amount of cash or higher-value property. In the following case, the sales manager of a sports vehicle dealership was charged under a state statute with grand theft.

Plea-Bargaining

Often, a grant of immunity from prosecution for a serious crime is part of the plea bargaining between the defending and prosecuting attorneys. The defendant may be convicted of a lesser offense, while the state uses the defendant's testimony to prosecute accomplices for serious crimes carrying heavy penalties.

knowingly

Only the knowledge that the facts exist; does not require any knowledge of unlawfulness. (This state of mind does not require Intent).

The Computer Fraud and Abuse Act

Perhaps the most significant federal statute specifically addressing cyber crime is the Counterfeit Access Device and Computer Fraud and Abuse Act. This act is commonly known as the Computer Fraud and Abuse Act (CFAA). Among other things, the CFAA provides that a person who accesses a computer online, without authority, to obtain classified, restricted, or protected data (or attempts to do so) is subject to criminal prosecution. Such data could include financial and credit records, medical records, legal files, military and national security files, and other confidential information. The data can be located in government or private computers. The crime has two elements: accessing a computer without authority and taking the data. This theft is a felony if it is committed for a commercial purpose or for private financial gain, or if the value of the stolen data (or computer time) exceeds $5,000. Penalties include fines and imprisonment for up to twenty years. A victim of computer theft can also bring a civil suit against the violator to obtain damages, an injunction, and other relief.

Mere Possession

Possession in which one may or may not be aware of what he or she possesses Example: If you agree to carry your friend's briefcase you do not know is filled with stolen money, you have got mere possession of the money

Knowing Possession

Possession with awareness of what one possesses Example: If you buy crystal meth. Knowing does not mean you have to know it is a crime to possess crystal meth, only that you know it is crystal myth

Legal Impossibility

Precludes prosecution in most jurisdiction. The defense of legal impossibility claims that the attempted offense is really no offense at all, either because there is no law against the imagined crime or because the actions in questions do not meet the requirements of the law for a crime to have occurred Example: Receiving sugar when you wanted and knew that it was going to be cocaine

31 CVC

Presenting false Id to a police officer if in a car.

wobbler 17B4

Reduction to felony to misdemonr by DA if it's a wobbler 17B4

What do Regulations do?

Regulations accompany Statutes to help implement the Statute.

What type of law does the Executive Branch produce?

Regulations and Executive Orders

wobbler crimes

Some crimes can be punished as either a felony or a misdemeanor.

M'naghten and Other State Rules

Some states use the M'Naghten test. Under this test, a person is not responsible if, at the time of the offense, he or she did not know the nature and quality of the act or did not know that the act was wrong. Other states use the irresistible-impulse test. A person operating under an irresistible impulse may know an act is wrong but cannot refrain from doing it. Under any of these tests, proving insanity is extremely difficult. For this reason, the insanity defense is rarely used and usually is not successful. Four states have abolished the insanity defense.

Scienter

Sometimes used to signify a defendant's knowledge or "guilty knowledge"

Necesity

Sometimes, criminal defendants can be relieved of liability by showing necessity—that a criminal act was necessary to prevent an even greater harm.

Online Retail Fraud

Somewhat similar to online auction fraud is online retail fraud, in which consumers pay directly (without bidding) for items that are never delivered. As with other forms of online fraud, it is difficult to determine the actual extent of online sales fraud, but anecdotal evidence suggests that it is a substantial problem.

31. RAPE: COERCION, EXTORTION AND FRAUD

THREAT OF NONPHYSICAL FORCE - EXTORTION OR COERCION It is difficult to draw the line between what constitutes coercion and what constitutes normal sexual bargaining. MPC Rule: A defendant has committed gross sexual imposition if he compels a victim to submit by any threat that would prevent resistance by a female of ordinary resolution. - MPC attempts to broaden scope of sexual offenses, but with two important limitations: (1) "woman of ordinary resolution" and (2) "compels to submit" prevent conviction for bargaining. - Essentially the same as duress doctrine. Pennsylvania v. Mlinarich: Victim's guardian compelled her to have sex with him by threatening to send her back to the juvenile detention center. The victim was 14 and the defendant was in his 60s. - Rule: If the defendant has not made threats of forcible compulsion that would prevent resistance by a person of reasonable resolution, then he is not guilty of rape. A choice, even an illegitimate one, makes the act voluntary. o Essentially a maximum resistance requirement. Tennessee v. Hilton: Stepfather bribed his stepdaughter to have sex with him in exchange for clothing and additional freedom. Intercourse began when the victim was 13 and continued until she was 17. - Rule: Where a defendant never makes a threat of injury against his victim and all evidence suggests that the two mutually agreed to the intercourse, it is a sexual bargain and not rape. o Court should have looked to the abuse of authority and power dynamic to find that the victim couldn't really freely consent to the conduct. FRAUDULENTLY OBTAINED CONSENT Fraudulently obtained consent substitutes for the force element of rape. Majority Rule: Most jurisdictions differentiate between fraud in the fact and fraud in the inducement. Fraud in the fact is rape because the victim had no opportunity to consent to the act. Fraud in the inducement is just misrepresentation, and the court is hesitant to impute guilt. Fraud in the fact would be a medical case or a wife believing she is having sex with her husband (consented to lawful intercourse, not unlawful intercourse). Minority Rule: Rape is satisfied by substantial fraud, which blurs the line between fact and inducement. Boro v. Superior Court (Inducement): Defendant convinced his victim he was a doctor and that she had a disease that could only be cured by having sex with an anonymous donor. - Rule: Where a victim knows that she is consenting to sex, even under false pretenses, it is fraud in the inducement and therefore not rape. Tennessee v. Tizard (Inducement or Fact): Victim went to doctor to obtain steroids. Doctor proceeded to sexually abuse the victim in exchange for the steroids. The statute was broad and covered both fraud in fact and fraud in the inducement. - Rule: If a physician (1) intends to gain sexual access for nonmedical purposes; (2) abuses his position as treating physician for that purpose; and (3) the patient allows access because of reasonable belief that it is for medical purposes, the physician is guilty of sexual offense. o Largely based on the breach of duty. The facts could be either fraud in fact or in the inducement, but the court chose to hold the doctor responsible because of his position of authority.

infamous crimes

Term is associated with an act of vileness or moral depravity, such as sodomy, buggery, or bestiality.

What is Perjury (P.C.118(a))?

Testifying under oath, knowingly presenting false evidence which must be of a material nature (felony) - Elements: - any person having taken an oath - to testify,depose,declare or certify truthfully - before any competent tribunal, oficer, or person - willfully states as true - any material matter which he knows to be false

Hierarchy in American Law

The Constitution of the U.S. is the highest form of law. All other law, including state constitutions, must be consistent with it. After the U.S. Constitution, the U.S. code is the highest form of federal law, with administrative regulations following. At the state level, state constitutions fall below the federal constitution, with state codes and then administrative regulation following, in that order. The common law, except interpretations of state and federal constitutions, is a lower form of law than statutes. As such, statutes prevail when in conflict with the common law

Constructive Possession

The ability to exercise control over property or objects, even though they are not in one's physical custody For example, if one's car is sitting in one's driveway, one has physical possession of the car. However, any person with the key has constructive possession, as they may take physical possession at any time without further consent from one.

GENERAL INTENT

The accused Merely intended to do the outlawed act, even if he or she did not intend or even know there he or she was violating the law

Indispensable Element Test

The accused becomes guilty when he or she obtains an indispensable element of the crime - i.e. murder weapon (murder weapon purchased) Defendant is culpable once he or she has committed an indispensable act

The Union of Actus Reas and Mens Reas equals:

The body of a crime or Corpus Delicti

Case Law

The body of previous decisions, or precedents, that has accumulated over time and to which attorneys refer when arguing cases and that judges use in deciding the merits of new cases

Criminal Law

The body of rules and regulations that defines and specifies punishments for offenses of a public nature or for wrongs committed against the state or society. Also called penal law

causation:

The causal relationship between conduct & result; this can lead to criminal liability. The relationship exits when a defendant's act is the Proximate cause of the injury. A defendant is liable for direct or proximate causes. A defendant is NOT liable for indirect or remote consequences.

The Criminal Law Legal Tradition vs. The Common Law Legal Tradition

The civil law legal tradition * Where: France, Italy, Portugal, Spain, Franco-Africa, Latin America, Louisiana (limited) * Judges play minimal role in development of law * Periodic, sometimes abrupt, legislative change * Rational and forward thinking The common law legal tradition * Where: England, United States, Australia, New Zealand, Belize, Canada, Anglo-Africa * Judges play major role in development of law * More evolutionary change through judicial decisions with occasional legislative change * More likely to change in response to current conditions

Adversarial System

The court system that pits the prosecution against the defense in the belief that truth can best be realized through effective debate over the merits of the opposing sides

Bribery

The crime of bribery involves offering to give something of value to a person in an attempt to influence that person, who is usually, but not always, a public official, to act in a way that serves a private interest. Three types of bribery are considered crimes: bribery of public officials, commercial bribery, and bribery of foreign officials. As an element of the crime of bribery, intent must be present and proved. The bribe itself can be anything the recipient considers to be valuable. Realize that the crime of bribery occurs when the bribe is offered—it is not required that the bribe be accepted. Accepting a bribe is a separate crime. Commercial bribery involves corrupt dealings between private persons or businesses. Typically, people make commercial bribes to obtain proprietary information, cover up an inferior product, or secure new business. Industrial espionage sometimes involves commercial bribes.

Vicarious Liability

The criminal liability of one party for the criminal acts of another party

Factual Defense

The defendant simply asserts that he or she didn't do it Examples: • Alibi: when a defendant claims to not have been present at the location of the crime • Proof Beyond Reasonable Doubt Doesn't Exist • Greater Crime Unproven, Although Lesser Is Proven • Other Outcomes Include • Acquittal • Lesser Punishment

Diminished Defensed vs. Defense of Insanity

The diminished capacity defense is similar to the defense of insanity in that it depends on showing that the defendant's mental state was impaired at the time of the crime. As a defense, diminished capacity is most useful when it can be shown that, because of some defect of reason or mental shortcoming, the defendant's capacity to form the mens rea required by a specific crime was impaired. Unlike an insanity defense, however, which can result in a finding of not guilty, a diminished capacity defense is built on the recognition that "[m]ental condition, though insufficient to exonerate, may be relevant to specific mental elements of certain crimes or degrees of crime."

Attendant Circumstances

The facts surrounding an event such as time of occurrence, location of occurrence, victim's mental capacity, victim's age, victim's exposure to narcotics or other drugs, victim's ability to escape, victim's capacity to resist Example: Use of weapon + bodily injury = higher level crime

culpable ignorance

The failure to exercise ordinary care to acquire knowledge of the law or of facts that may result in criminal liability.

Federal Crimes

The federal criminal code now lists more than four thousand criminal offenses, many of which do not require a specific mental state. There are also at least ten thousand federal rules that can be enforced through criminal sanctions, and many of these rules do not require intent.

FLIGHT

The flight of a person after the commission of a crime is not sufficient in itself to establish his guilt, but is a fact which may be considered by you in the light of all other proved facts in deciding whether a defendant is guilty. The weight to which this circumstance is entitled is a matter for you to decide.

Treason

The highest crime an individual could commit, and the only crime mentioned in the constitution

Criminal

The human conduct that violated the criminal law (only)

5. Concurrence (State of Mind and Human Conduct)

The intent and the act must be present at the same time.

General Intent

The intent to do that of which the law prohibits * doesn't matter if there was intention or knowledge of violating the law * No specific state of mind is required

premeditated murder

The intentional killing of another human being with malice afterthought. An act of killing that has been planned out beforehand.

Canon Law

The law of the Catholic Church, was influential on the development of the civil law tradition

Stare Decisis

The legal principle that requires that courts be bound by their own earlier decisions and by those of higher courts having jurisdiction over them regarding subsequent cases on similar issues of law and fact. The term literally means "standing by decided matters." or " let the decision stand". Embodies the idea that cases with like fact should have the same law applied. The earlier case is commonly known as precedent

Burden of Proof

The mandate, operative in American criminal courts, that an accused person is presumed innocent until proven guilty and that the prosecution must prove the defendant's guilt beyond a reasonable doubt

Due Process of Law

The procedure that effectively guarantee individual rights in the face of criminal prosecution; the due course of legal proceedings according to the rules and forms that have been established for the protection of private rights; formal adherence to fundamental rules for fair and orderly legal proceedings. Due process of law is a constitutional guarantee

There is no requirement that theft take place in burglary

There is no requirement that theft take place in burglary

Concurrence

There must be unity of Act and Intent or Criminal Negligence to constitute a crime. Aka: joint operation and intent

Extortion and Blackmail

Threat made for the purpose of illegally acquiring money or other things of value

Distinguish

To argue or to find that a rule established by an earlier appellate court decision does not apply to a case currently under consideration even though an apparent similarity exists between the cases

Rackateering

To curb the entry of organized crime into the legitimate business world, Congress enacted the Racketeer Influenced and Corrupt Organizations Act (RICO). The statute makes it a federal crime to: Use income obtained from racketeering activity to purchase any interest in an enterprise. Acquire or maintain an interest in an enterprise through racketeering activity. Conduct or participate in the affairs of an enterprise through racketeering activity. Conspire to do any of the preceding activities.

Criminalize

To make criminal; to declare an act or omission to be criminal or in violation of a law making it so

What is "pro se"?

To prosecute one's case by oneself. Meaning "for him- or herself" in Latin.

Begging money in a public place is no longer a crime in California

True

Larceny

Under the common law, the crime of larceny involved the unlawful taking and carrying away of someone else's personal property with the intent to permanently deprive the owner of possession. Put simply, larceny is stealing, or theft. Whereas robbery involves force or fear, larceny does not. Therefore, picking pockets is larceny, not robbery. Similarly, taking company products and supplies home for personal use without permission is larceny. (Note that a person who commits larceny generally can also be sued under tort law because the act of taking possession of another's property involves the tort of trespass to personal property—see Chapter 12.) Most states have expanded the definition of property that is subject to larceny statutes. Stealing computer programs may constitute larceny even though the "property" is not physical (see the discussion of computer crime later in this chapter). So, too, can the theft of natural gas or Internet and television cable service.

Infancy to Seven Years Old

Usually can't be charged, even as a minor. There is a possible defense of infancy (Automatic). The violators in this case are usually not referred to as anything (i.e. offenders or criminals) and are not convicted

Deviant Behavior

Violations of both mores and norms are forms of deviance called "deviant behavior"

Actus Reus (Guilty Action), Mens Rea (Guilty Mind), Concurrence, Harmful Result and Causation

What are the four essential elements of a crime?

Conduct which is not the product of the actor's determination. Physical acts resulting from seizures or convulsion. Acts while the defendant was asleep or unconscious.

What are the three parameters that define involuntary acts in regards to Actus Reus?

What are Remedies?

What courts can do to and for litigants

Guilty physical action or unlawful omission by the defendant.

What is Actus Reus?

A guilty state of mind or intent at the time of the act.

What is Mens Rea?

The guilty act and the guilty mind must occur at the exact same time.

What is concurrence?

The defendant carried out the act.

What is factual harm?

Motive is the underlying reason for why we do what we do.

What is motive?

The defendant used power of suggestion, manipulation, or subliminal messages to cause another to act in a harmful manner.

What is proximate harm?

Immunity

When the state wishes to obtain information from a person accused of a crime, the state can grant immunity from prosecution. Alternatively, the state can agree to prosecute the accused for a less serious offense in exchange for the information. Once immunity is given, the person has an absolute privilege against self-incrimination and therefore can no longer refuse to testify on Fifth Amendment grounds.

double jeopardy

Wherever possible, all related offenses from a single criminal episode will be charged in a single accusatory pleading and be simultaneously prosecuted.

Strict Liability vs. Vicarious Liability

While strict liability and vicarious liability are both forms of liability without fault, they dispense with the fault requirement in entirely different ways. Strict liability is imposed in situations in which the mens rea requirement has been eliminated. Vicarious liability, on the other hand, may be imposed in situations in which the act requirement has been eliminated

Organized Crime

White-collar crime takes place within the confines of the legitimate business world. Organized crime, in contrast, operates illegitimately by, among other things, providing illegal goods and services. Traditionally, the preferred markets for organized crime have been gambling, prostitution, illegal narcotics, and loan sharking (lending funds at higher-than-legal interest rates), along with more recent ventures into counterfeiting and credit-card scams.

voluntary intoxication

Willful intoxication; intoxication that is the result of personal choice.Voluntary intoxication includes the voluntary ingestion, injection, or taking by any other means of any intoxicating liquor,

Deliberate and premeditated murder example:

Willful, Deliberate and premeditated killing with express malice aforethought is murder of the 1st The word willful as used in this instruction means intentional The word deliberate means formed or arrived at or determined upon These are mental states

• Defense of others:

Yes. If you reasonably believe that bodily injury is about to be inflicted upon another person, you can use reasonable force to protect that person from attack. In doing so, you may use all the force and means which you believe to be reasonably necessary and which would appear to a reasonable person, in the same or similar circumstances, to be necessary to prevent the injury which appears to be imminent.

Criminal Law vs Civil Law

You can civilly sue for damages -Criminal action and civil remedies are not mutually exclusive, they can go both at same time or different times. - think OJ Simpson Burden of proof, civil is anything above 50%, criminal is Criminal law need to prove beyond a responsible doubt

Appellate Court

a court of appeals or Supreme Court

Dirk or dager

a knife or other instrument with or without a hand guard capable of ready use as a stabbing weapon that may inflict great bodily injury Carried concealed upon his person (not in a sheath if open, not capable for ready use, has to be unscrewed etc.

Under the peer review improvement act of 1982, peer organizations are responsible for the review for appropriateness and necessity of putting a patient into the hospital by a process known as ______ review.

admissions

Large capacity magazine

any ammunition feeding device which has the capacity to accept more than ten rounds

An unlawful threat or attempt to do bodily injury to another is

assault

consent to treatment based on a full understanding of all possible risks of unpreventable results of that treatment is called

assumption of risk

836.6(a) PC

attempting to escape custody from a judges orders. misdemeanor.

Precedent

binding court decisions

Burglary is punishable as follows: (a) Burglary in the first degree:

by imprisonment in the state prison for two, four, or six years.

Abridge

circumvent, avoid

Clear and Convincing Evidence Standard

clear and convincing proof requires that truth of issue be highly probable, and this standard is greater than preponderance of evidence and less than proof byone a reasonable doubt

Autopsy

dissection or testing of a body to determine cause of death.

State Law

each state has their own penal code

A person who is no longer under the care, custody, or supervision of a parent is called a(n)

emancipated minor

The ability to see things from another person's point of view is

empathy

Right and wrong conduct is known as

ethics

Except for juveniles, the testimony of an accomplice must be corroborated in all cases.

false

Feigned accomplices are guilty as principals providing they are over fourteen years of age

false

In those instances where required, an accomplice's testimony must be corroborated to a greater degree than merely showing that a crime was committed.

false

Holding or detaining a person against his will is

false imprisonment

148.1(a) PC

falsely reporting a destructive device. felony.

Jurisprudence

field of law

Zip gun

home made fires a projectile by the force of an explosion or other from of combustion

______ are defined as what are right and wrong

laws

Substantive law

laws defining crimes and their punishments

4 reasons why protestant morality came under assult in the first decades of 20th cent

loss of biblical authority, decline of universal morality, psychology, consumerism

Negligence by a professional person is called

malpractice

Incestuous

marriage or sexual relations between close blood relatives

what is posse comitatus?

means power of the county, came from the old west when an officer can deputize you on the spot when he is need of help, not really done now, but can be done

Conduct, courtesy, and manners that are customarily used in a medical office by medical professionals are known as

medical etiquette

Switchblade knife

on one's person or within the passenger compartment of a vehicle with a blade two or more inches in length

That which in natural and continuous sequence, unbroken by any new independent cause, produces an event, and without which the injury would not have occurred is called_____.

proximate cause

latin for "he who acts through another acts for himself" is called

qui facit per alium facit perse

Moral turpitude

relating to ethics, honesty, morality

define: taking a firearm from a peace officer. (PC148c)

removes or takes a firearm from the immediate presence of the officer or public officer while in the performance of his or her duties

1489(c) PC

removing or taking an officers gun. felony

latin for "things done; deeds"; the facts and circumstances attendant to the act in question would be called

res gestae

Latin for "things decided'- that is, a matter already decided by judicial authority is called.

res judicate

what is obstructing an excutive officer? (PC69)

resisting delaying or Obstructing an executive officer by threat or violence that deters them from the performace of their duties or who knowingly resists by force or violence such officer in the performance of his duties (felony)

define: disarming a peace officer by removing a weapon other than a firearm (PC 148(b)

resisting or delaying a peace officer by removing a non firearm from the immediate presence of the officer (misdemenaor)

Responsibility of an employer for the acts of an employee is

respondeat superior

Stare decisis

rigidly adhere to precedence

Doctrines

rules

Felonies

serious crimes punishable by death or by imprisonment for more than one year. Many states also define different degrees of felony offenses and vary the punishment according to the degree. For instance, most jurisdictions punish a burglary that involves forced entry into a home at night more harshly than a burglary that involves breaking into a nonresidential building during the day.

attempt two elements needed

specific intent and direct but ineffectual act done towards its commission

148(b) PC

taking any other weapon from a peace officer

Aiders and Abettors

those who help the perpetrator commit the crime

71 PC

threatening a public officer. felony

140(a) PC

threats of retaliation. felony

Harbor Defined?

to "harbor" a person means to receive that person secretly or covertly and without lawful authority for the purpose of concealing him so that another having the right to lawful custody of such person shall be deprived of same

a wrong committed against another person or the person's property ia a

tort

Statutes

written laws.

Mala prohibita

wrong because prohibited by law

Mala in Se

wrong by its very nature

Malum in se:

wrong or evil in itself.... rape, murder, ect (moral condemnation)

What is violating a court order? (PC 166(A)(4))

violating a court order, which is anything ordered to you by the court, must have been notified of the order, this crime is a (misdemeanor) unless there is a prior court order violation involving violence or force.(felony)

166(a)(4) PC

violating a court order. misdemeanor.

Infamous crime

violating common principles of humanity

Mere preparation:

which may consist of planning the offencse or of devising obtaining or arraging the means for its commission, is not sufficient to constitute an attempt

a legal statement of how an individuals property is to be distributed after death is

will

Define: Threats of retaliation (P.C. 140).

willfully threatens to use force or violence upon a victim or witness. Or to damage or destroy any property of a witness. Because somebody helped Law Enforcement (felony)

Inherent

within, basic to, built-in

Capricious

without good (legal) reason

non-compos mentis

without mastery of mind (Roman empire) shouldn't be held blameworthy of their crimes. "mens rea" = guilty mind. to be guilty, criminal act not enough (actus reus), one must also possess a guilty mind.

Inchiate

"partial" or "unfinished". Applied to crimes like conspiracy to commit a criminal act, solicitation of others to engage in criminal acts, and attempts to commit crimes

Malum Prohibita:

"wrong [as or because] prohibited"...Parking tickets, traffic tickets ect (against the law because the law says so)

California's legal defense of necessity

("A person is not guilty of a crime when [he] [she] engages in an act, otherwise criminal, through necessity. The defendant has the burden of proving by a preponderance of the evidence all of the facts necessary to establish the elements of this defense, namely: [1] The act charged as criminal was done to prevent a significant and imminent evil, namely, [a threat of bodily harm to oneself or another person] [or] [ ]; [2] There was no reasonable legal alternative to the commission of the act; [3] The reasonably foreseeable harm likely to be caused by the act was not disproportionate to the harm avoided; [4] The defendant entertained a good-faith belief that [his] [her] act was necessary to prevent the greater harm; [5] That belief was objectively reasonable under all the circumstances; [and] [6] The defendant did not substantially contribute to the creation of the emergency[.] [; and [7] The defendant reported to the proper authorities immediately after attaining a position of safety from the peril.]")

Additional Principles that are Necessary to Fully Appreciate Contemporary Understandings of Crime

(1) Causation (2) A resulting harm (3) The principle of legality (4) Necessary attendant circumstances All these act as limitations on the authority of government to prosecute

11. MISTAKE OF FACT (MPC 2.04)

(1) Ignorance or mistake as to a matter of fact or law is a defense if: (a) the ignorance or mistake negatives the purpose, knowledge, belief, recklessness or negligence required to establish a material element of the offense; or (b) the law provides that the state of mind established by such ignorance or mistake constitutes a defense. (2) Although ignorance or mistake would otherwise afford a defense to the offense charged, the defense is not available if the defendant would be guilty of another offense had the situation been as he supposed. In such case, however, the ignorance or mistake of the defendant shall reduce the grade and degree of the offense of which he may be convicted to those of the offense of which he would be guilty had the situation had been as he supposed.

Number of Arguments in Favor of Allowing Abandonment or Renunciation as a Defense

(1) It encourages desistance (2) It indicates a lack of dangerousness on the part of the offender (3) It shows a lack of intent on the part of the defendant to carry the crime to completion

Common Punishments for Criminal Acts

(1) Monetary Fines: Applied to felonies, misdemeanors, or infractions (2) Short-Term Incarceration: Applied to felonies or misdemeanors (3) Long-Term Incarceration: Applied to felonies (4) Death Penalty: Applied to some felonies

Why is it Important to Establish When an Individual Conspirator Withdrew?

(1) Prosecution Under a Statute of Limitations (2) Defense challenges to declarations made by another conspirators after withdrawal (3) Defense claims of nonliability for crimes committed by other after the date of a defendant's withdrawal

11. MENS REA (MPC) (2.02)

(1) Minimum Requirements of Culpability: Except as provided in Section 2.05, a person is not guilty of an offense unless he acted purposely, knowingly, recklessly or negligently, as the law may require, with respect to each material element of the offense. a. Must attach to each part of the statute. Structure of MPC MR 2.02 (2) a. Break down Actus Reus into material elements of (1) conduct, (2) circumstance, or (3) result; b. Attach a culpability level to each material element: i. Purposely (P); 1. Conduct/Result: Desire or hope to act or cause results; 2. Circumstance: Awareness of circumstances or belief/hope that they exist; ii. Knowingly (K); 1. Conduct/Circumstances: Awareness that D's conduct is of the nature or that such circumstances exist; 2. Result: Awareness of a certain risk that conduct will cause such a result. iii. Recklessly (R) (default MPC culpability level); 1. D consciously disregards a substantial and unjustifiable risk constituting a gross deviation from the standard of conduct that a law abiding person would observe in D's position; iv. Negligently (N); 1. D "should have" perceived a risk, where a reasonable person in D's position would have been aware of a substantial and unjustifiable risk. 1. About half of jurisdictions adopt the MPC mens rea standards. MPC is considered a great improvement on common law, which suffers from serious problems of ambiguity and vagueness but may punish some crimes too leniently compared to the common law. a) No distinction between general and specific intent b) Concerned about subjective states of mind c) Every element has a culpability requirement; must meet the culpability requirement for every material element d) Recklessness is the default culpability level if not specified. e) Any single culpability term applies to all elements, unless a culpability term comes later in the statute, suggesting that the term modifies only elements that follow. f) Willful Blindness: When D lacks statutory knowledge, D is treated as though he had knowledge anyway when D is aware of a high probability of a particular fact's existence that is an element of an offense but chooses to ignore it. g) 2.02 defines mental states and shows where mistakes are okay h) 2.04 is superfluous i) Culpability level is sufficient for offense in a few circumstances (1) Culpability level applies to all elements (2) Culpability level applies to one element

The Purpose of Law

(1) Regulate the flow of human interaction (2) Law makes for predictability in human events by using the authority of government to ensure that society agreed-on-standards of behavior will be followed and enforced. They allow people to plan their lives by guaranteeing a relative degree of safety to well-intentioned individuals, while constraining the behavior of those who could unfairly victimize others (3) Laws provide a stable foundation for individuals wishing to join together in a legitimate undertaking by enforcing rights over the control and ownership of property (4) Laws also provide for individual freedoms and personal safety by sanctioning the conduct of anyone who violates the legitimate expectations of others. Hence the first and most significant purpose of the law can be simply states: Laws support social order (5) To many people, a society without laws is unthinkable. Where sucha society exists, however, it would be doubtless rules by individuals and groups powerful enough to unsurp control over others. The personal whims of the powerful would rule, and the less powerful would live in constant fear of attack

Four Possible Verdicts When the GMBI INsanity Defense is Raised

(1) guilty, (2) not guilty, (3) not guilty by reason of insanity, and (4) guilty but mentally ill A finding of GBMI is equivalent to a finding of guilty, and the court will sentence the defendant just as a person found guilty of the crime in question would be sentenced. A GBMI verdict establishes that "the defendant, although mentally ill, was sufficiently in possession of his faculties to be morally blameworthy for his acts."107 As one author says, "The most obvious and important function of the GBMI verdict is to permit juries to make an unambiguous statement about the factual guilt, mental condition, and moral responsibility of a defendant."108 Once sentenced, GBMI offenders are evaluated to determine whether hospitalization or psychiatric treatment is needed. If the offender is hospitalized, he or she will join the "regular" prison population on release from the treatment facility. Time spent in the hospital and in prison both count toward sentence completion, and the offender will be released from custody after the sentence has been served, even if he or she still suffers from mental disease.

Situations defense of property, also called protection of property, can be applied

(1) protection of personal property, (2) defense of home or habitation, (3) defense of another's property, and (4) use of a mechanical device to protect property

Reasons/Importance of Corpus Delciti

(1) reduce the risk of convicting a defendant based on his confession for a crime that did not occur (2) A crime will generally be prosecuted in the location where it was commited - that is, where the corpus delicti of the crime exists

Essential Aspects of All Crimes

(1) the criminal act (which, in legal parlance, is termed the actus reus) (2) a culpable mental state (mens rea) (3) a concurrence of the two

The Justinian Code's (Corpus Juris Civilis (CJC) )Lengthy Legal Documents

(1) the institutes, (2) the Digest, and (3) the Code itself. Justinian's code distinguished between two major legal categories: public laws and private laws

vandalism

(a) Every person who maliciously commits any of the following acts with respect to any real or personal property not his or her own, in cases other than those specified by state law, is guilty of vandalism: (1) Defaces with graffiti or other inscribed material. (2) Damages. (3) Destroys.

Postpartum Depression

- 15% of women experience it following childbirth -Loss of pleasure in most activities -depressed mood -sleep difficulties -weight gain/loss -loss of energy -fatigue -extreme guilt -suicidal thoughts.

General principles .. Elements for criminal liability

- Actus reus- bodily movement plus the (free will)will to do so. o If someone has seizure and harms a person its not an assult because there is no free will in the movement o Pg 88 case people vs decina CASE BREIF here - Mens rea- mental state provides whats going through a persons mind when commiting a crime - BOTH mens rea and Actus rea MUST BE PRESENT for a crime to occur - CONCURRENCE o How do you prove intent: actions, behavior - Causation- - Resulting harm: ie muder, mayhem

Burden of proof for crim/ civil

- Civil- anything above 50% preponderance of the evidence - Criminal- beyond a reasonable doubt

-Goals crim/ civil are different

- Civil- want to make things whole again - Criminal- punishment

- Preventing future criminal behavior

- Deterrence of crime

Sources of criminal law

- Federal - State - Local

- Classifying and grading criminal behavior

- Felony, wolber, misdemonor ect

Cal classification of offenses

- Felony- stae prison is possible - Wobblers- could be felony or misdemonor - Misdemeanor- up to a year in county jail - Infraction- fine only (in califonia its infraction, not a *** like the book says, ) ---------------------`Write infreaction on the test!

2 parts to criminal law

- General: applies to all crimes - Specific: defines specific crime (elements)

- General: applies to all crimes

- Law of attempt

- Specific: defines specific crime (elements)

- Murder, rape, robbery ect

Regina v. Oxford (1840)

- because of a "diseased mind" defendant was "quite unaware of the nature, character, and consequences of the act he was committing"

M'Naghten Rule

-15 high court judges establish new standard of legal insanity b/c of M'naghten case. Queen victoria ordered it 3 components 1) presumption that defendants are sane and responsible for their crime 2) Requirement that, at moment of crime, accused must have been laboring "under a defect of reason or from disease of the mind 3) requirement that defendant didn't know nature and quality of act he was doing or if he did, that he didn't know what he was doing was wrong. -Sometimes referred to as cognitive test of insanity b/c emphasizes knowing and understanding.

- Excusing criminal behavior.

-Behavior that would otherwise be criminal

- Classifying and grading criminal behavior

-Felony, wolber, misdemonor ect

Post-Hinkley developments in insanity law

-Insanity Defense Reform Act (IDRA) -volitional prong dropped from definition -Ultimate issue/opinion testimony

The Durham Case: Reshaping definition of insanity

-appeal prompted judge David Bazelon to reexamine M'Naghten rule. -decided old definition of insanity obsolete and misguided. -called for retrial with new standard of insanity rule used: Durham Standard or the Product Test -was borrowed from New Hampshire state case that took place in 1870s

The Hinkley Case: The consequences

-case seemed to epitomize all that was wrong with insanity defense -Public view: Had presence of mind to stalk president, buy handgun, and plan murder attempt.avoided being held accountable because wealthy parents bought him a good lawyer and psych experts -most prominent factor in acquittal was burden of proof, which was placed on prosecution to prove that he WAS sane.

Guilty but Mentally Ill Verdict

-permitted in 13 states -alternative verdict to three more standard options of guilty, not guilty, and NRGI/ -Ppl found guilty of their crime and sentenced to prison for a period consistent with that verdict. -Supposed to receive treatment for their mental health issues while in prison or transferred to secure psychiatric facility. -offers no guarantee that offenders receive EFFECTIVE treatment for mental disorders, so scholars question usefulnesss

American Law Institute (ALI) standard

-person isn't responsible for criminal conduct if at the time of such conduct, as a result of mental disease or defect, he lacks substantial capacity either to appreciate the criminality (wrongfulness) of his conduct OR to conform his conduct to the requirements of the law

Mens Rea Defenses

-plead diminished capacity if lacks this - only available for crimes that require specific mental state on the part of the defendant -couldn't get first degree murder because lacked capacity to form specific intent to kill another person (ex if person thought was alien not human) -must be proven beyond a reasonable doubt

Norman Finkel (2007)

-presented groups of mock jurors with range of insanity instructions (M'naughten, irresistible impulse test, durham test, ALI test, and no test) -findings agreed: content of insanity instructions didn't matter -didn't blame on jurors. -argued jurors use preexisting commonsense notions of insanity to inform and interpret their judgements of a defendant's responsibility and intentions.

Criticisms of R-Cras

-quantifies factors that are qualitative, so implies scientific precision in insanity ratings that mostly derive from examiners subjective opinions (Melton et el., 2007)

Problems with developing tests

1) insanity involves retrospective evaluation of individuals mental state at time of crime. -Have to figure out how defendant was functioning in the past. -lots has changed from past 2) legal elements of insanity hazy -hard to quantify terms used -hard to make test to examine whether someone has ability to control their actions and knows difference between right and wrong 3) Considerable variations in states insanity standards -psych test designed to assess ALI-based definition not work on M'Naghten

1. D intends to kill A. While hiding in wait for A to walk down a road which he usually traverses, D shoots at a person whom he believes to be A, but is actually B. The bullet misses B and hits C, wounding him. D is charged with the crime of assault with intent to murder. Which of the following persons should be specified by the prosecutor as the victim of that crime? (A) A only. (B) B only. (C) C only. (D) Both A and B.

1. (B) The correct answer is (B). For the crime of assault with intent to kill, there must be both an assault and an intent to kill. In firing at B, D assaulted him, and he intended to kill the person he assaulted, who is in fact B. (A) is incorrect because, no matter what D's intention was with respect to A, D did not assault A. (C) is incorrect because D did not intend to assault C, and did not attempt to assault the person who turned out to be C. The doctrine of transferred intent is inapplicable to this crime. (D) is incorrect because, again, D did not assault A.

Battery w/serious bodily injury

1. A serious bodily injury is a serious impairment of physical condition that can include: * loss of consciousness * Bone fracture * Concussion * Wound requiring stitching

Mayhem

1. Unlawful and maliciously 2. Removed a part of someone's body or disabled or made unless a part of someone's body and the disability was more than slight or temporary 3. Permanently disfigured 4. Cut or disabled tongue 5. Slit someone's nose, ear or lip 6. Puts out an eye or disables it *General intent crime *Felony *A disfiguring injury may be permanent even if it can be repaired by medical procedures

Battery

1. Willful & unlawful 2. Touching 3. In a harmful or offensive manner 4. Not acting in self-defense or in defense of others PC 242 Misdemeanor and general intent crime * there need not be direct touching of the victims person to constitute a battery

11. Alpha and Beta participated in the armed robbery of the First National Bank. When they left the bank, Alpha drove away in the getaway car; but Beta missed connections, and ran into the cellar of an abandoned building next door to the bank. A bank employee saw Beta entered the cellar, and followed him. When the employee encountered Beta in the cellar, a scuffle ensued, and Beta's gun discharged accidentally, killing the employee. If Alpha is charged with felony murder in the death of the employee, his best defense is: (A) The act was committed by Beta. (B) The act was committed by accident. (C) The killing of an employee was not part of the agreement. (D) The killing of the employee was not part of the armed robbery.

11. (D) On these facts even the best answer is not a very good one. For felony murder, the killing must occur in the commission of a felony - not after the felony has been completed. The distinction here is that a killing during flight from the scene of the crime is not within the res gestae if the felon has reached a point of momentary safety. Arguably, this happened when Beta took refuge in the cellar, although a bank employee followed him there. (D) is the correct answer because it would be Alpha's best defense to argue that his conspiracy with Beta and the felony they committed were complete when Beta reached the safety of the cellar. Accordingly, any subsequent killing, intentional or not, was not within the perpetration or flight from the felony. (A) is incorrect because Alpha and Beta were conspirators and there is no evidence of Alpha's withdrawal. Under principles of accomplice liability, principals and accessories before the fact are equally liable for a killing committed during the commission of the underlying felony. (B) is incorrect because even an accidental death, committed during the course of a felony can be felony murder. (C) is incorrect because Alpha and Beta did agree to commit the dangerous felony of armed robbery and within the scope of the risk created it was possible that someone would be killed.

14. In which of the following situations is Defendant most likely to be convicted of manslaughter? (A) Vic approached Defendant carrying a cap pistol and said in a joking manner, "Stick 'em up.'' Defendant stabbed Vic with his knife and killed him. (B) Defendant was engaged in the rape of Victoria when she began to cry. For the purpose of keeping her quiet, Defendant put a gag in her mouth. Victoria attempted to swallow the gag and choked to death. (C) Vic approached Defendant with a leather strap in his hand and said, "I'm going to hit you with this for what you said about my wife." Defendant shot Vic with W's gun and killed him. (D) Defendant, intending only to wound Vic, shot him in the leg with his gun. The bullet severed an artery and Vic bled to death.

14. (C) The best answer, (C), is a case of voluntary manslaughter on a theory of imperfect self-defense (an overreaction) with deadly force, to a non-deadly assault. A leather strap is an instrument unlikely to inflict death or great bodily harm, but it would clearly be battery to use it on Defendant. If Defendant shot Vic with his gun and killed him, this would be the better example of the imperfect self defense to reduce a murder charge to manslaughter. (A), the next best answer, is much weaker, on the facts. The defendant's right of self-defense is imperfectly exercised because a cap pistol probably looks like a toy and Vic said "Stick 'em up" in a joking manner. In (A), unlike (C), it is unlikely Defendant could have reasonably believed an assault was taking place, even a non-deadly one. Note that neither (A) nor (C) on the given facts is voluntary manslaughter on a "heat of passion" theory. (B) is incorrect because the crime is not manslaughter but is felony-murder, i.e., a killing (although an unintentional one) in the course of a felony rape. (D) is incorrect because the crime is murder. The intent to inflict great bodily harm such as when one intends to shoot someone in the leg, is sufficient to establish the element of malice. To answer Questions #15-#18 it is necessary to fit the fact pattern of each question to the most suitable precedent, (A)-(D), by extracting their principles. (A) holds that deadly force is justified in making an arrest for a felony. (B) holds that deadly force is not justified in making an arrest for a misdemeanor. (C) holds that, where non-deadly force is justifiably used to make an arrest, there is no liability for criminal homicide if death unexpectedly results.

Transporting Firearms in Vehicles

18 years or older, Legal to carry or transport firearm (pistol, revolver, firearm) ONLY if locked in trunk, locked in container.

The Hinkley Case: The Trial

1983 -4 psych experts testified Hinkley suffered from severe psychological disturbance, most likely paranoid schizophrenia. -psychiatrists for prosecution disputed claims -Found not guilty by reason of insanity.

Altering identifying marks on a firearm

23900 PC Classification Felony Any person who; Changes, alters, removes, or obliterates the Name, model, manufactures number, other mark or identification on any firearm without written permission from DOJ to make change

Basis for a stop & detention

A LAWFUL STOP AND DETENTION IS WARRANTED IF AN OFFICER: IS IN A PLACE WHERE HE OR SHE HAS A RIGHT TO BE, AND CAN ARTICULATE SPECIFIC FACTS AS TO WHY HE/SHE HAS REASONABLE SUSPICION TO BELIEVE THAT: SOME ACTIVITY RELATED TO CRIME HAS OCCURRED, IS OCCURRING OR IS ABOUT TO OCCUR, AND THE PERSON DETAINED IS INVOLVED IN THAT ACTIVITY CANNOT BE BASED ON A MERE HUNCH BUT MUST BE BASED ON SPECIFIC AND ARTICULABLE FACTS DESCRIBING SUSPICIOUS BEHAVIOR. REASONABLE SUSPICION

Detention

A TEMPORARY SEIZURE OF A PERSON A TEMPORARY "DETENTION" OR "STOP" OCCURS WHEN AN OFFICER USES HIS/HER POLICE AUTHORITY TO COMPEL A PERSON TO HALT OR TO REMAIN IN ONE PLACE OR TO PERFORM SOME ACT IT IS A SEIZURE UNDER THE 4TH AMENDMENT. IT IS NOT AN ARREST OF THE PERSON

Recklessness

A court can also find that the required mental state is present when a defendant's acts are reckless or criminally negligent. A defendant is criminally reckless if he or she consciously disregards a substantial and unjustifiable risk.

PC 22210: Striking weapons: Slungshot

A device consisting of a small mass of metal or stone fixed on a metal handle or strap

PC 32900: Multiburst trigger activator

A device designed or redesigned to be attached to a semiautomatic firearm that allows the firearm to discharge two or more shots in a burst, or a manual or power-driven, trigger-activating device constructed and designed so that when attached to a semi-automatic firearm it increases the rate of fire of that firearm.

Apparent Danger

A form of imminent danger that is said to exist when the conduct or activity of an attacker makes the threat of danger obvious

Accessory Before the Fact

A person who aids and abets in preparation for crime commission, but who was not present at the crime scene

a CRIME is an offense against the state (all the people) & carries a punishment with it.

A public wrong

Durham rule

A rule for determining insanity that holds that an accused is not criminally responsible if his or her unlawful act was the product of mental disease or mental defect. Also called product rule.

Retreat Rule

A rule in many jurisdictions that requires that a person being attacked retreat in order to avoid the necessity of using force against the attacker if retreat can be accomplished with "complete safety"

Wharton's Rule

A rule of applicable to conspiracy cases that holds that, when the targeted crime by its very nature takes more than one person to commit, there can be no conspiracy when no more than the number of people required to commit the offense participate in it

alter ego rule

A rule of law that, in some jurisdictions, holds that a person can only defend a third party under circumstances and only to the degree that the third party could act on his or her own behalf.

Canon of Construction

A rule that guides courts in interpreting constitutions, statutes, and other law

substantial capacity test

A test developed by the American Law Institute and embodied in the Model Penal Code that holds that "a person is not responsible for criminal conduct if at the time of such conduct as a result of mental disease or defect he lacks substantial capacity either to appreciate the criminality [wrongfulness] of his conduct or to conform his conduct to the requirements of the law."

What is a writ of execution?

A writ which will authorize the sheriff to seize any property belonging to the defendant, sell it (typically through an auction), and give the resulting money to the plaintiff (up to the amount of the judgment).

It is a crime to solicit all but one of the following. Select the incorrect answer.

A) Burglary B) Robbery C) Mayhem D) Assault with deadly weapon Answer: (C) Mayhem

The statement " A physicaian shall respect the law" is a part of the.

AMA code of ethics

Reasonable Suspicion

AN OFFICER MUST OBJECTIVELY ENTERTAIN SUCH SUSPICION AND THE FACTS MUST BE SUCH THAT ANY REASONABLE PEACE OFFICER IN A SIMILAR SITUATION WOULD COME TO THE SAME CONCLUSION

19. ATTEMPT - MENS REA

ATTEMPT - MENS REA Common Law Rule (Majority Rule): Attempt crimes require showing D's specific intent to perform all material elements of the offense. Even when offense is one of general intent or strict liability, common law mens rea attempt shifts culpability to specific intent. Holmes Rule: Only intent to commit the underlying offense is necessary because the criminal law is concerned with assessing the seriousness of a risk and deterring it. Actual intent is required for incomplete conduct because it establishes the dangerousness of the individual. MPC Rule: Purpose is required for conduct and purpose or knowledge for result elements, but only the required mens rea for the underlying offense for circumstance elements. a. Completed conduct with a Circumstance element: Purpose with respect to conduct. b. Completed conduct with a Result element: Purpose or knowledge with respect to the result c. Incomplete conduct with a Result element: Purpose where the act must be a substantial step toward the commission of a crime that strongly corroborates criminal purpose. Thomas (Colo.) Rule: Defendant must have purpose to commit the underlying offense with respect to the result. - Criticism: No logical stopping point - why not carry through to attempted negligent homicide? Most jurisdictions follow the MPC or supplement the common law approach with reckless/negligent endangerment statutes to achieve the same level of deterrence. Thacker v. Virginia: Defendant expressed intent to shoot out a lantern but nearly hit a woman in the head with his shot. - Common Law Rule: To establish guilt for attempt, the defendant must have acted with specific intent with respect to all elements of the crime. Colorado v. Thomas: Defendant fired warning shots at man who raped his ex-girlfriend. - Rule: A defendant must demonstrate the culpability for the underlying offense with respect to the result to be guilty of attempted reckless manslaughter.

Who creates Regulations?

Agencies within the Executive Branch

All in STATE prison for burglary

All in STATE prison for burglary 2 yrs (low term) 4 yrs (mid term) 6 yrs (upper term) Cant do 5 yrs. You can determine the amount of time due to the damages and character of perp He can go to jail if he gets probation but still filled under a felony

public-order offense

An act that is willfully committed and that disturbs public peace or tranquillity. Included are offenses like fighting, breach of peace, disorderly conduct, vagrancy, loitering, unlawful assembly, public intoxication, obstructing public passage, and (illegally) carrying weapons. Public-order offenses or crimes against the public order, include offenses such as fighting, breach of peace, disorderly conduct , vagrancy, loitering, unlawful assembly, public intoxication, obstructing public passage, and (illegally) carrying weapons

Causation in Fact

An actual link between an actor's conduct and a result. Even so cause in fact cannot be said to be the sole cause, or even the primary cause, of a particular event. If a person fires a gun, for example, and the bullet strikes the building, causing it to ricochet and hit a person standing next to the shooter, it can be said that the person who fired the weapon shot the bystander or, at the very least, caused him to be shot

What Does the Essence of Criminality Consist of?

An actus reus A mens rea The concurrence of an illegal act and a culpable mental state

Insanity

An affirmative defense to a criminal charge; a social and legal term (rather than a medical one) that refers to "a condition which renders the affected person unfit to enjoy liberty of action because of the unreliability of his behavior with concomitant danger to himself and others."v Also, a finding by a court of law

What is Jurisdiction?

An area of authority over which a governing body has control.

Probable Desistance Test

An attempt has been made when the accused is unlikely to stop voluntarily. Here the accused has obtained the victim and little stands in the way of the crime's completion (victim lured into vehicle) Defendant is culpable if completion of the crime is within grasp, although the defendant's act need not be the penultimate act

PC 21810: Striking weapons: Metal knuckles

Any device or instrument: • made wholly or partially of metal • which is held in or worn on the hand • for offensive or defensive purposes Metal knuckles: • protect the wearer's hand while striking a blow • increase the force of impact from the blow or injury to the individual receiving the blow The metal contained in the device may: • help support the hand or fist • provide a shield to protect the user's hand or fist - consist of projections or studs which would contact the individual receiving the blow

PC 22210: Striking wepons: Billy

Any device that is intended to be used as a striking device (i.e., heavy club)

PC 29800(b): Juvenile who has been convicted of a violent offense (Felony)

Any juvenile who has been convicted of a violent offense is prohibited from: owning, possessing, having custody or control of any firearm until the age of 30 Violent offenses include, but are not limited to: murder, attempted murder, arson, armed robbery, rape, kidnapping, assault by means of force or with a firearm, carjacking

Deadly weapon

Any object or instrument or weapon that is inherently deadly or dangerous or one that is used in such a way that it is capable of causing and likely to cause death or injury

P.C. 31 defines a principle as:

Any person involved in planning, preparation or carrying out the act of a crime. *One who directly & actively commits or attempts the act constituting a crime *One who aids or abets the act or attempted act of a crime

PC 417.4: Imitation firearm (Misdemeanor)

Any person, except in self-defense, draws or exhibits an imitation firearm in a threatening manner against another person in such a way as to cause a reasonable person apprehension or fear of bodily harm

Criminal Process

As mentioned earlier in this chapter, a criminal prosecution differs significantly from a civil case in several respects. These differences reflect the desire to safe-guard the rights of the individual against the state. Exhibit 7-3 below summarizes the major steps in processing a criminal case. We now discuss three phases of the criminal process—arrest, indictment or information, and trial—in more detail.

Solicitation

Asking someone to commit a crime, with intent that crime be committed 1. solicit another to commit crime 2. at the time, person have specific intent for that crime to be committed. 3. the soliciting message was received by the intended recipient Crime is in the asking (not agreeing or completion)

Steps in Criminal Case: Trial

At a criminal trial, the accused person does not have to prove anything. The entire burden of proof is on the prosecutor (the state). As mentioned earlier, the prosecution must show that, based on all the evidence, the defendant's guilt is established beyond a reasonable doubt. If there is reasonable doubt as to whether a criminal defendant committed the crime with which she or he has been charged, then the verdict must be "not guilty." A verdict of "not guilty" is not the same as stating that the defendant is innocent. It merely means that not enough evidence was properly presented to the court to prove guilt beyond a reasonable doubt. Courts have complex rules about what types of evidence may be presented and how the evidence may be brought out in criminal cases, especially in jury trials. These rules are designed to ensure that evidence presented at trials is relevant, reliable, and not prejudicial toward the defendant.

Prop 115 was passed in what year? a. 1982 b. 1990 c. 1999 d. 2004

B) 1990

Other Syndromes

BWS was one of the first syndromes to be raised as a defense enhancement in American courtrooms. Many others have since followed. Among them are these48: adopted child syndrome, false memory syndrome, premenstrual syndrome (PMS), holocaust survival syndrome (see, for example, Werner v. State49), attention deficit disorder (raised in defense of Michael Fay, the teenager "caned" in Singapore in 1994), black rage defense (created by William Kunstler and offered in defense of Colin Ferguson, the Long Island Rail Road shooter), elder abuse syndrome, fetal alcohol syndrome (a variation of which was used in defense of Eric Smith, the 14-year-old who sodomized and killed a fouryear- old neighborhood boy), Gulf War syndrome, Munchausen by proxy syndrome (wherein a caregiver injures his or her children to gain attention), nicotine withdrawal syndrome, repressed memory syndrome (used by California prosecutors to convict George Franklin, Sr., of the rape-murder of his eight-year-old daughter's friend 20 years after the killing), rotten social background syndrome, parental abuse syndrome (employed by Erik and Lyle Menendez), post-traumatic stress disorder, rape trauma syndrome (see, for example, State v. Marks50), ritual abuse syndrome (abuse at the hands of Satanic and other cults), UFO survivor syndrome, urban survival syndrome (used to defend Texas killer Daimian Osby), and Vietnam syndrome (see, for example, State v. Kenneth J. Sharp, Jr.51). One especially recent syndrome to be discussed in the medical literature is Internet addiction disorder (IAD). Specialists studying this disorder say that IAD is as real as alcoholism. People with IAD tend to lose control over their daily activities and crave the use of the Internet. They even have withdrawal symptoms when forced to forgo access to the Internet for an extended period of time.

Standards of Proof Compared

Beyond a Reasonable Doubt: High or moral certainty, but not 100% confidence. Used for convictions in criminal cases Clear and convincing evidence: Greater that preponderance of evidence but less than beyond a reasonable doubt. Used to decide specific issues in criminal and quasi-criminal cases The level of factual proof used in civil cases involving issues of personal liberty. The standard requires greater certainty than "more probable than not" but is not as demanding as "no reasonable doubt" Preponderance of Evidence: Greater than 50% probability. Used for verdicts in civil cases. A standard for determining legal liability, which requires a probability of just over 50 percent that the defendant did what is claimed

Malware

Botnets are one of the latest forms of malware, a term that refers to any program that is harmful to a computer or, by extension, a computer user. A worm, for example, is a software program that is capable of reproducing itself as it spreads from one computer to the next. --> Virus: A virus, another form of malware, is also able to reproduce itself, but must be attached to an "infested" host file to travel from one computer network to another. For instance, hackers are now capable of corrupting banner ads that use Adobe's Flash Player. When an Internet user clicks on the banner ad, a virus is installed. Worms and viruses can be programmed to perform a number of functions, such as prompting host computers to continually "crash" and reboot, or otherwise infect the system. (For a discussion of how malware is now affecting smartphones, see this chapter's Insight into the Global Environment feature.)

What is "Judgment-Proof"?

Broke AF! When a litigant cannot pay damages because he has no money to give.

residential burglary

Burglary of an Inhabited Dwelling (residential) with the Action of actually taking something.

What is the maximum sentence one can serve for a misdemeanor crime? a. 3 years in state prison b. 6 months in jail c. 1 years in county jail d. 9 months in county jail

C) one year in county jail

PC 30210: Ammunition

Certain types of specialty ammunition are also prohibited by law. These include: ammunition containing or consisting of a flechette dart (a dart capable of being fired from a firearm) which: - measures approximately 1 inch in length - with tailfins which take up 5/16 of an inch of the body • bullets which contain or carry explosive agents

- Penalties of crim/civil are different in nature

Civil- mostly monetary damages Criminal- jail time, restitution (monetary payment) can be included too

Factual Impossibility

Claims that the defendant could not have committed the envisioned offense even if her or she had been able to carry through the attempt to do so. i.e., alibi, killing a dead person

Statutes (Public Laws)

Codification of the common law. Also known as the laws of legislature. In addition of the Congress of the United Sates, the legislatures of each state has the authority to define crimes. Statutes are commonly organized by topics, such as the Code of Criminal Laws. The vast majority of crimes are defined by state crimes, and more than 90 percent of felonies and serious misdemeanors are prosecuted in state courts

12025(a)(1) P.C.

Concealed firearm in VEHICLE. The driver may be arrested because a firearm is being carried concealed in a vehicle that is under his control or direction.

Chain Conspiracy

Conspirators follow in a sequence and deal only with the individual who comes before them

Prosecuting Cyber Crime

Cyber crime has raised new issues in the investigation of crimes and the prosecution of offenders. Determining the "location" of a cyber crime and identifying a criminal in cyberspace present significant challenges for law enforcement.

Cyberterrorism

Cyberterrorists, as well as hackers, may target businesses. The goals of a hacking operation might include a wholesale theft of data, such as a merchant's customer files, or the monitoring of a computer to discover a business firm's plans and transactions. A cyberterrorist might also want to insert false codes or data. For instance, the processing control system of a food manufacturer could be changed to alter the levels of ingredients so that consumers of the food would become ill. A cyberterrorist attack on a major financial institution, such as the New York Stock Exchange or a large bank, could leave securities or money markets in flux and seriously affect the daily lives of millions of citizens. Similarly, any prolonged disruption of computer, cable, satellite, or telecommunications systems due to the actions of expert hackers would have serious repercussions on business operations—and national security—on a global level.

Purposeful Mens Rea

Equivalent of specific intent at the common law; a desire to cause the outcome that resulted (requires more than an awareness than"knowing" that the result is likely; it requires the actor to want it to happen Example: She needed to be taught a lesson for roaming carelessly in the street. If it cost her her life, then so be it

Morals

Ethical principles, or principles meant to guide human conduct and behavior; principles or standards of right and wrong

4 reasons for the rise in immigration in 1880-1900

European pop growth, urban crowding in Europe, antisemitism, economic oppertunity

Defense

Evidence and arguments offered by a defendant and his or her attorney(s) to show why that person should not be held liable for a criminal charge. Legal defense categories: Alibi, justification, excuse, procedural

Punishment for misdemeanor; punishment not otherwise described.

Except in cases where a different punishment is prescribed by any law of this state, every offense declared to be a misdemeanor is punishable by imprisonment in the county jail not exceeding six months, or by fine not exceeding one thousand dollars ($1,000), or by both.

6. Mens Rea (State of Mind; "guilty mind")

Exceptions are strict liability offenses involving health and safety, in which it is not necessary to show intent. Legislatures have criminalized certain kinds of offenses in order to protect the public. Courts limit application of such laws to situations in which recklessness or indifference is present.

Direct evidence

Eyewitness testimony Statements to the police Words spoken by suspect

Discharging Firearms at vehicle

FELONY (12034 P.C.) Misdemeanor if the DRIVER ALLOWS the PASSENGER with firearm to enter the vehicle.

Criminal Negligence

Failure to exercise a degree of care which a person of ordinary prudence would exercise under the same circumstances. * negligence substitutes for the "act"

CRIMINAL NEGLIGENCE

Failure to use the degree of care required to avoid criminal consequences. Uses the "reasonable person"

If no one is home at the time, a subject cannot be guilty of window-peeking under PC 647(i)

False

When police detain a suspect, they are required to Mirandize him/her prior to asking any questions which may incriminate him/her T/F

False

Bankruptcy Fraud

Federal bankruptcy law allows individuals and businesses to be relieved of oppressive debt through bankruptcy proceedings. Numerous white-collar crimes may be committed during the many phases of a bankruptcy action. A creditor may file a false claim against the debtor, which is a crime. Also, a debtor may fraudulently transfer assets to favored parties before or after the petition for bankruptcy is filed. For instance, a company-owned automobile may be "sold" at a bargain price to a trusted friend or relative. Closely related to the crime of fraudulent transfer of property is the crime of fraudulent concealment of property, such as the hiding of gold coins.

Altering identification marks on a firearm

Felony Any person who changes, alters, remoces or obliterates the name of the maker, model, manufacturer's number, other mark of identification including any distinguishing number or mark assigned by DOJ

Prohibited weapon PC 16590

Felony Any person who: Manufactures or causes to be manufactured or imports into the state or sells, offers for sale, exposes for sale, keeps for sale gives, lends, or possesses ANY WEAPON THAT HAS BEEN SPECIFIED WITHIN THIS PC SECTION AS PROHIBITED

Aggravated Mayhem

Felony Specific intent Intended to permanently disable or disfigure the other person

Brandishing a weapon 417 P.C.

Firearm unloaded in a threatening manner. MUST have 245 P.C. ADW

What Does The Rule of Law Include According to The American Bar Assoicaiton (ABA)?

Freedom form private lawlessness provided by the legal system of a politically organized society A relatively high degree of objectivity in the formulation of legal norms and a like degree of even handedness in their application Legal ideas and justice devices for the attainment of individual and group objectives within the bounds of ordered liberty Substantive and procedural limitations on governmental power in the interest of the individual for the enforcement of which there are appropriate legal institutions and machinery

Insanity case is bifurcation (split in 2)

Guilt phase: is he guilty of the crime? If they are not guilty no need to go to next phase Sanity phase: was he insane during the crime? Insanity BY legal definition Evidence of mental state before during and after the crime

The Principle of Vicarious Liability

Has mostly been used to hold a corporate body accountable for violations of regulatory offenses

Brief Fact Summary. The defendant, Christian Porter's (the "defendant"), home was destroyed by a fire on July 20, 1992. The defendant was subsequently charged with and convicted of two counts of arson. Synopsis of Rule of Law. Scientific evidence must be reliable and relevant before it will be admissible. Facts. The defendant's home was destroyed by a fire on July 20, 1992. He was subsequently charged with and convicted of two counts of arson. The defendant retained Leighton Hammond ("Mr. Hammond"), a polygraph examiner, to conduct a polygraph examination to determine whether the defendant was telling the truth when he claimed that he had no knowledge of, and had not participated in, the burning of his home. The examination results were in the defendant's favor, and he moved that the trial court admit the results of the polygraph examination. The trial court denied the motion, stating that it was not the place of a trial court to reconsider Connecticut's traditional per se ban on the admissibility of polygraph evidence. Issue. Whether Connecticut should adopt the Daubert v. Merrell Dow Pharmaceuticals, Inc. standard for the admissibility of scientific evidence? Whether Connecticut should abandon its traditional per se rule that polygraph evidence is not admissible at trial?

Held. Justice Borden issued the opinion of the Supreme Court of Connecticut which did adopt the Daubert standard. The per se rule against admitting polygraph evidence is affirmed because any evidentiary value of a polygraph is outweighed by its prejudicial effects. Discussion. The Court extensively examined the elements of a polygraph examination and noted that the accuracy of the test is open to debate. Further, the value of the test as indicative of guilt or innocent is questionable. The test is too unreliable. Thus, even if the polygraph satisfies the Daubert standard for admission, the trial judge should continue to exclude evidence if its probative value is outweighed by its prejudicial effect

Bailey v. United States Brief Fact Summary. Defendant was convicted of robbery. Defendant argues that he was merely present and did not have a role in the crime. Synopsis of Rule of Law. Mere presence at the scene of the crime is insufficient; a culpable presence is essential. Facts. Defendant was convicted of robbery. Defendant was seen together with an unknown man believed to be the actual robber prior to the robbery. Defendant walked away from the actual robber prior to the robbery, but remained a few feet away. After the robbery was accomplished, both Defendant and the robber ran in the same direction. Issue. Was Defendant's proximity to the crime sufficient to support a conviction for robbery?

Held. Reversed and remanded for a judgment of acquittal. In order to aid and abet another to commit a crime, it is necessary that a Defendant in some sort associate himself with the venture and that he participate in it as in something that he wishes to bring about. An inference of criminal participation cannot be drawn merely from presence, a culpable purpose is essential. Present is equated with aiding and abetting when it is shown that it encourages the perpetrator, facilitates the unlawful deed, or when it stimulates others to offer assistance in the commission of the crime. Dissent. The dissent focused on the fact that Defendant was closely associated with the perpetrator and was with him immediately before the robbery. This left in the dissenting judges' eyes no other permissible explanation but that Defendant was involved with the crime.

Public Order Crime

Historically, societies have always outlawed activities that are considered contrary to public values and morals. Today, the most common public order crimes include public drunkenness, prostitution, gambling, and illegal drug use. These crimes are sometimes referred to as victimless crimes because they normally harm only the offender. From a broader perspective, however, they are deemed detrimental to society as a whole because they may create an environment that gives rise to property and violent crimes.

Fact: ∆ and man met each other and ∆ thought that man stole his watch. Then there was a plan to rob a drug store and the ∆ boosted the man in and then he called the police and helped them catch the man. He said that he participated in the burglary to get him caught by the cops so that he could get him back for stealing the watch. The trial court said that the decoy defense would not work—a decoy has to stop short of lending assistance to the criminal. Issue: Was ∆ entitled to a defense that he was acting as a decoy or a spy?

Holding: Yes. Reasoning: In order to find an aider and abettor guilty, it is necessary that he have the same criminal intent as the principal The jury should have been allowed to determine whether ∆ had acted with a felonious intent There was evidence to support the ∆'s claim that he was acting as a decoy

Direct versus Circumstantial Evidence

If John testifies that he saw Tom raise a gun and fire it at Ann and that Ann then fell to the ground, John's testimony is direct evidence that Tom shot Ann. If the jury believes John's testimony, then it must conclude that Tom did in fact shoot Ann. If, however, John testifies that he saw Tom and Ann go into another room and that he heard Tom say to Ann that he was going to shoot her, heard a shot, and saw Tom leave the room with a smoking gun, then John's testimony is circumstantial evidence from which it can be inferred that Tom shot Ann. The jury must determine whether John's testimony is credible.

felony murder rule

If a homicide occurs during the commission or attempt to commit certain serious felonies( i.e. rape, robbery, etc), it is 1st degree murder regardless of whether the killing was intentional or unintentional.

Burden of Proof

In a civil case, the plaintiff usually must prove his or her case by a preponderance of the evidence. Under this standard, the plaintiff must convince the court that based on the evidence presented by both parties, it is more likely than not that the plaintiff's allegation is true. In a criminal case, in contrast, the state must prove its case beyond a reasonable doubt. If the jury views the evidence in the case as reasonably permitting either a guilty or a not guilty verdict, then the jury's verdict must be not guilty. In other words, the government (prosecutor) must prove beyond a reasonable doubt that the defendant has committed every essential element of the offense with which she or he is charged. If the jurors are not convinced of the defendant's guilt beyond a reasonable doubt, they must find the defendant not guilty. Note also that in a criminal case, the jury's verdict normally must be unanimous—agreed to by all members of the jury—to convict the defendant.Footnote (In a civil trial by jury, in contrast, typically only three-fourths of the jurors need to agree.)

Cyber Theft

In cyberspace, thieves are not subject to the physical limitations of the "real" world. A thief can steal data stored in a networked computer with Internet access from anywhere on the globe. Only the speed of the connection and the thief's computer equipment limit the quantity of data that can be stolen.

Reasonable Doubt

In legal proceedings, an actual and substantial doubt arising from the evidence, form the facts or circumstances show by the evidence, or form the lack of evidence. Also, that state of the case, which after the entire comparison and consideration of all the evidence, leaves the minds of the jurors in such a condition that they cannot say they feel an abiding conviction of the truth of the charge

Civil Liability

In the event of a RICO violation, the government can seek civil penalties. The government can seek the divestiture of a defendant's interest in a business or the dissolution of the business. (Divestiture refers to the taking of possession—or forfeiture—of the defendant's interest and its subsequent sale.) Moreover, in some cases, the statute allows private individuals to sue violators and potentially recover three times their actual losses (treble damages), plus attorneys' fees, for business injuries caused by a RICO violation. This is perhaps the most controversial aspect of RICO and one that continues to cause debate in the nation's federal courts. The prospect of receiving treble damages in civil RICO lawsuits has given plaintiffs a financial incentive to pursue businesses and employers for violations.

You have a legal duty to act and do not act. The reasonableness test.

In what two ways can omission be a crime?

Inquisitorial System v. Adversarial System

Inquisitorial System * One continues investigation/trial overseen by judge *Judges play active role throughout process * Truth is sought through investigation * Process is less formal than in an adversarial system Adversarial System * Investigation overseen by police and separate judicial phase initiated by prosecutor with minimal judicial involvement in early stages * Judges are more passive, and attorneys play more active role * Truth is sought through competition between prosecution and defense * Process is highly formal and technical, especially once charges are filed Adversarial System: Two parties against each other in search of truth, promotes argument/debate, the united states of america inquisitorial system: judge takes the prominent role, Not Attorneys, focus on fact finding, juries are the exception

Rex v. Arnold (1724)

Insanity had become less of a moral failing (good v evil) and more of a cognitive failing (mental deficiency) -jurors instructed to acquit the defendant (wounded british lord in assasination attempt) if found him to be "totally deprived of his understanding and memry and doesn't know what he's doing, no more than brute or WILD BEAST. -test now called wild beast test of insanity.

motive

Is the desire or inducement which incites or stimulates a person to do an act.

Why Is Common Law Referred to As Judge-Made Law?

It depended heavily on judicial interpretation

Why is Stare Decisis Preferred?

It promotes the overhanded, predictable, and consistent development of legal principles, fosters reliance on judicial decisions, and contributes to the actual and perceived integrity of the judicial process Stare decisis is not an inexorable command; rather, it "is a principle of policy and not a mechanical formula of adherence to the latest decision."

The Hinkley Case: John Hinckley Jr Background

John Hinckley Jr. -1976 dropped out of tx tech to pursue music in hollywood -became obsessed with jodi foster. -went to Yale where foster was a student. -tried to assassinate President Ronald Reagan to win her love (reenacting scene from Taxi Driver) -he shot and wounded 4 people, including the president -Trial in 1983

Knowing Mens Rea

Knowing (knowing behavior):Action undertaken with awareness that the outcome is practically certain didn't hope she'd die but knew with substantial certainty that she would. Example: She needed to be taught a lesson for roaming carelessly in the street. If it cost her life, then so be it

Ignorance of Fact

Lack of knowledge of some fact relating to the situation at hand.

What is Primary Authority?

Law from any branch of government. Primary Authority IS the law.

What is Persuasive Authority?

Law from other jurisdictions. A court may rely on or follow case law from another jurisdiction (but is not required to do so) if the reasoning from that case law is persuasive and consistent with the law from the court's jurisdiction.

Statutory Law

Law in the form of statutes or formal written codes made by a legislature or governing body with the power to make law

Common Law

Law originating from use and custom rather than form written statutes. The term refers to nonstatutory customs, traditions, and precedents that help guide judicial decision making THE MAJOR SOURCE OF MODERN CRIMINAL LAW

Positive Law

Law that is legitimately created and enforced by governments (made by people)

Midemeanors

Less serious crimes punishable by a fine or confinement for up to a year.

Criminal intent

Manifested by the circumstances connected with the offense. Law states that "all person are capable of committing crimes" except those listed in the penal code (pc26)

The Durham Case: Background information

Monte Durham. -1945 released from US navy b/c psych eval found him unfit to continue -attempted suicide 2 years later, put in psych hospital for two months -mental condition deteriorated further when imprisoned for car theft and bad checks -1951 arrested for breaking and entering -even w/ past diagnoses, trial judge refused to let him plead insanity. Found guilty -1954 US court of appeals for DC overturned conviction.

2. PRINCIPLE OF LEGALITY (COMMON LAW & MPC)

Must be a statute before crime occurs in order to prosecute crime. MPC: No conduct constitutes an offense unless it is crime or violation under MPC or another statute of this State. Goff v. Tenn: Pool Hall on a Sunday, $10 civil penalty turns into criminal conviction: RULE: Conduct is not criminal unless made criminal by statute or by common law. The King v. Manley: different approach, common law courts have authority to define criminal offenses (rejected)

Firearms within home or business

Not a 2nd amendment issue and MUST be a citizen. (depends on the state)

What turns a consensual encounter into a detention or arrest:

ORDERS OR COMMANDS BLOCKING THE PERSON'S PATH ACCUSATORY QUESTIONING PATDOWN SEARCHES MANNER OF APPROACH USING AN EMERGENCY LIGHT RATHER THAN A SPOTLIGHT DEMANDING OR KEEPING A PERSON'S I.D. RATHER THAN REQUESTING IT AND RETURNING IT WHEN FINISHED MIRANDA ADMONITION REQUEST FOR CONSENT TO SEARCH F.I. CARD-BE CAREFUL DISPLAY WEAPON DURING CONTACT STATIONHOUSE INTERVIEWS TONE OF VOICE LENGTH OF TIME NUMBER OF OFFICERS

Accessory

One who knowingly gives assistance to a person who has committed a felony for a purpose of helping that individual avoid apprehension or detection. An accessory is liable for separate, lesser offenses following a crime (1) Crime Committed by someone else (2) Personally aided the offender after the crime (3) Knew crime was committed (4) Acted for the purpose of Hindering prosecution

Online Auction Fraud

Online auction fraud, in its most basic form, is a simple process. A person puts up an expensive item for auction, on either a legitimate or a fake auction site, and then refuses to send the product after receiving payment. Or, as a variation, the wrongdoer may send the purchaser an item that is worth less than the one offered in the auction. The larger online auction sites, such as eBay, try to protect consumers against such schemes by providing warnings about deceptive sellers or offering various forms of insurance. It is nearly impossible to completely block fraudulent auction activity on the Internet, however. Because users can assume multiple identities, it is very difficult to pinpoint fraudulent sellers—they will simply change their screen names with each auction.

Justifiable use of Force

Other situations, however, also justify the use of force: the defense of one's dwelling, the defense of other property, and the prevention of a crime. In all of these situations, it is important to distinguish between deadly and nondeadly force. Deadly force is likely to result in death or serious bodily harm. Nondeadly force is force that reasonably appears necessary to prevent the imminent use of criminal force. Generally speaking, people can use the amount of nondeadly force that seems necessary to protect themselves, their dwellings, or other property, or to prevent the commission of a crime. Deadly force can be used in self-defense only when the defender reasonably believes that imminent death or grievous bodily harm will otherwise result. In addition, normally the attacker must be using unlawful force, and the defender must not have initiated or provoked the attack. Many states are expanding the situations in which the use of deadly force can be justified. Florida, for instance, allows the use of deadly force to prevent the commission of a "forcible felony," including robbery, carjacking, and sexual battery.

Civil Suits

Parties to a civil suit are referred to as plaintiff and the defendant, and the names of civil suits take the form Named Plaintiff v. Named Defendant. Unlike criminal cases, in which the state routinely prosecutes wrongdoers, most civil suits are brought by individuals. No civil action is undertaken that is not initiated by the injured party!

initiative measures

Petitions signed by a certain minimum number of registered voters can force a public vote on a proposed statute or constitutional amendment.

Somatoform Disorders

Physical symptoms, such as paralysis, without biological explanation. Hypochondriasis, conversion disorder, and somatization disorder are subtypes

What are Executive Orders?

Policy directives that implement or interpret a statute, a constitutional provision, or a treaty. Executive Orders play a less active role in governing peoples' day-to-day lives.

Actual Possession

Possession in which one has direct physical control over the object or objects in question Example: a person wearing a watch has actual possession of the watch. Likewise, if you have your wallet in your jacket pocket, you have actual possession of your wallet. This type of possession, however, is by necessity very limited.

When must Corpus Delicti be established?

Prior to arrest

In order to arrest the arresting officer must have:

Probable Cause

Psychoactive Substance Use Disorders

Produced by excessive and long-term use of mind-altering substances, such as stimulants, barbiturates, cocaine, heroin, and alcohol

Elements of Corpus Delicti?

Prohibited Act Actus Reus Accomplished by a criminal human agency State of mind (Mens Rea) General or specific intent Criminal negligence Distinguish crime from an accident or mistake or fact

Overcriminalization

Proponents of strict liability criminal laws argue that they are necessary to protect the public and the environment. Critics say laws that criminalize conduct without any required intent have led to overcriminalization, or the use of criminal law as the main tool to solve social problems, such as illegal drug use. They argue that when the requirement of intent is removed, people are more likely to commit crimes unknowingly—and perhaps even innocently. When an honest mistake can lead to a criminal conviction, the idea that crimes are a wrong against society is undermined.

Functions of Criminal Law

Protects members if the public from harm Preserve and maintain social order Support fundamental social values Distinguish criminal wrongs from civil wrongs Express communal condemnation of criminal behavior Deter people form criminal activity Stipulate the degree of seriousness of criminal conduct Establish criteria for the clear determination of guilt or innocence Punish those who commit crimes Rehabilitate offenders Assuage victims of crime

148.9 PC

Providing false ID to a police officer. misdemeanor.

Organic Mental Disorders

Psychological or behavioral abnormalities associated with temporary or permanent dysfunction of the brain. Causes include aging, disease, injury, and drugs. Delirium and dementia are subcategories.

moral turpitude

Relates to the deemed "shamefully immoral" and suggests a lack of honesty, modesty, integrity, and good morals.

...

Rifle, Barrel less than 16 inches and overall less than 26 inches.

19. ATTEMPT - MENS REA CHART

SEE CHART

23. D's (Accomplice's) Mens Rea wrt other Actor's CHART

SEE CHART P. 48

Factors that increase criminal penalties

Sentencing guidelines still exist and provide for enhanced punishment for certain types of crimes. Penalties can be enhanced for white-collar crimes, violations of the Sarbanes-Oxley Act (, and violations of securities laws. The sentencing judge must take into account the various sentencing factors that apply to an individual defendant before concluding that a particular sentence is reasonable. When the defendant is a business firm, these factors include the company's history of past violations, management's cooperation with federal investigators, and the extent to which the firm has undertaken specific programs and procedures to prevent criminal activities by its employees.

Duress, justification or an excuse?

Some legal scholars say that duress may qualify as either a justification or an excuse. The main difference, they point out, is that necessity(justification for unlawful behavior) is brought about by acts of nature or natural events,whereas duress is imposed by one human being on another. Whenever a person is forced to act in violation of the law, whether by reason of human or natural "pressure," he or she is probably convinced that such action is necessary.

Postsentence

Some states provide for mandatory psychological assessment of specific offenders (e.g., sexual predators) for dangerousness. If dangerous, civil commitment is used to continue detention

Civil Liability for Criminal

Some torts, such as assault and battery, provide a basis for a criminal prosecution as well as a civil action in tort.

What type of crime can intoxication be used as a defense?

Specific Intent Crimes - to show defendant was incapable of forming intent

10. STRICT LIABILITY (PUBLIC WELFARE OR INDEPENDENT MORAL WRONGS)

Strict Liability can apply to elements of a statute (Public Welfare or Independent Moral Wrongs) An offense where at least one element does not require mens rea. GENERAL PRINCIPLES: 1. No Mens Rea defense. Never ask what the Defendant was thinking. 2. No Mistake of Fact defenses allowed for Strict Liability elements. 3. Offenses traditionally attached to SL under common law usually carry over into whatever statute has been adopted to punish those offenses. RULE: When at least one material element of the offense clearly rejects a mens rea requirement, then SL. A. Public Welfare Offenses (PWO): Utilitarian. 1. RULE: If a statute is a PWO, then it most likely (1) is regulatory and reasonable in nature, (2) has relatively light penalties with little social stigmatization, (3) is an offense that makes mens rea very difficult to prove, (4) is not derived from common law, and (5) is designed to prevent simultaneous injury to many people. 2. Rationale: a. Superincentives for public safety; b. Difficulty in identifying the party at fault in a complicated chain of actors; c. Societal gains outweigh light public welfare offense penalties. B. Independent Moral Wrongs (IMW): moral conduct. 1. RULE: When D knows that he engages in wrongful conduct, then D does not need to be aware of further facts that render the conduct criminal. a. Rationale: D does not deserve to use mistake of fact as a defense when D knew the conduct was wrong. b. Unlike PWO, SL for IMW often results in high stigmatization and severe punishments. 2. HIGHER GRADE OFFENSE RULE: If D knows enough to satisfy mens rea for one criminal offense, that knowledge will be sufficient for conviction of a higher grade of that offense if the actual facts turn out to fit the higher grade, even if D is unaware of the actual facts.

Objective test that califonia uses, 2 types of test

Subjective test: predisposition of the person who commited the crime Objective: the conduct of the PD

Law

That which is laid down, ordained, or established...a body of rules of action or conduct prescribed by controlling authority, and having binding legal force." However, not all rules are laws, fewer still are criminal laws, and not all have "binding legal force

Scope of Warrant

The Fourth Amendment prohibits general warrants. It requires a particular description of what is to be searched or seized. General searches through a person's belongings are impermissible. The search cannot extend beyond what is described in the warrant. Although search warrants require specificity, if a warrant is issued for a person's residence, items in that residence may be searched even if they do not belong to that individual. In the following case, police officers obtained a search warrant and conducted a search for weapons in the home of a suspect's foster mother. A judge later ruled that the warrant was not supported by probable cause, and the homeowners sued individual police officers for executing an illegal search warrant.

Fourth Amendment Protections

The Fourth Amendment protects the "right of the people to be secure in their persons, houses, papers, and effects." Before searching or seizing private property, normally law enforcement officers must obtain a search warrant—an order from a judge or other public official authorizing the search or seizure. Advances in technology allow the authorities to track phone calls and vehicle movements with greater ease and precision. Nevertheless, the use of such technology can still constitute a search within the meaning of the Fourth Amendment.

Model Penal Code Approach

The MPC uses a substantial-step test with a broad scope to distinguish an attempt. Searching for following a potential victim falls under this umbrella (attempt to locate victim made)

Federal Sentencing Guidelines

The Sentencing Reform Act created the U.S. Sentencing Commission, which performs the task of standardizing sentences for federal crimes. The commission's guidelines establish a range of possible penalties for each federal crime. Originally, the guidelines were mandatory, in that the judge was required to select a sentence from within the set range and was not allowed to deviate from it.

Cyber Crime

The U.S. Department of Justice broadly defines computer crime as any violation of criminal law that involves knowledge of computer technology for its perpetration, investigation, or prosecution. Many computer crimes fall under the broad label of cyber crime, which describes any criminal activity occurring via a computer in the virtual community of the Internet. Most cyber crimes are simply existing crimes, such as fraud and theft of intellectual property, in which the Internet is the instrument of wrongdoing.

Importance of Stare Decisis

The U.S. Supreme Court has ruled, "Stare decisis is of fundamental importance to the rule of law. Case law and statutory law make for predictability in the law In the words of the Court: "Acknowledgments of precedent serve the principal purposes of stare decisis, which are to protect reliance interests and to foster stability in the law"

What are the special circuit courts?

The United States Court of Appeals for the District of Columbia; The Court of Appeals for the Federal Circuit

What is the highest court in the Federal Court System?

The United States Supreme Court. It usually grants certiorari to 100 or fewer cases of the 7,000+ petitions that it receives each year.

Last Act Test

The accused has taken the final step and has done all that was intended, but the crime has not been completes. This is before the "victim dies" step" In the crime of attempt, a test that asks whether the accused had taken the last step or act toward commission of the offense, had performed all that he or she intended and was able to do in an attempt to commit the crime, but for some reason did not complete the crime

Police Power

The authority of a state to enact and enforce a criminal statute

Broad Application of Rico

The broad language of RICO has allowed it to be applied in cases that have little or nothing to do with organized crime. RICO incorporates by reference twenty-six separate types of federal crimes and nine types of state felonies. If a person commits two of these offenses, he or she is guilty of "racketeering activity." Under the criminal provisions of RICO, any individual found guilty is subject to a fine of up to $25,000 per violation, imprisonment for up to twenty years, or both. Additionally, any assets (property or cash) that were acquired as a result of the illegal activity or that were "involved in" or an "instrumentality of" the activity are subject to government forfeiture.

Model Penal Code

The courts have had difficulty deciding what the test for legal insanity should be. Federal courts and some states use the substantial-capacity test set forth in the Model Penal Code: A person is not responsible for criminal conduct if at the time of such conduct as a result of mental disease or defect he or she lacks substantial capacity either to appreciate the wrongfulness of his [or her] conduct or to conform his [or her] conduct to the requirements of the law.

What is the court hierarchy?

The decisions of higher courts are Mandatory Authority for lower courts within that jurisdiction. By contrast, the decisions of a lower court are merely Persuasive Authority to the courts above it in the same jurisdiction. Therefore, attorneys give greater weight to decisions from higher courts because they control the decision-making in the lower courts.

Criminal Solicitation

The encouraging, requesting, or commanding of another person to commit a crime Words commanding or urging the commission of a crime plus specific intent equals solicitation For example, if Alice commands Bob to assault Charlie, and Alice intends for Bob to assault Charlie, then Alice is guilty of solicitation. However, if Alice commands Bob to assault Charlie without intending that an actual crime be committed (perhaps believing that Charlie has given consent), then there is no solicitation. (An "offer" to Commit a Crime) The first person gets charged with i.e., murder and solicitation and the person doing the ACT gets charged with JUST murder While solicitation is, in effect, an attempted conspiracy, it is possible to have a conspiracy without a prior solicitation. NO overact required for solicitation

Misprision of Felony

The failure to report a known crime; the concealment of a crime

Bill of Rights

The first ten amendments to the U.S. Constitution, which were made part of the Constitution in 1791

Criminal Liability

The following two elements normally must exist simultaneously for a person to be convicted of a crime: The performance of a prohibited act (actus reus). A specified state of mind, or intent, on the part of the actor (mens rea).

Civil Law

The form of the law that governs relationships between parties

What is an Attachment?

The legal term for an officially sanctioned seizure of property. Real estate, cars, and other property may be "attached."

Degree

The level of seriousness of an offense

Rule of Law

The maximum that an orderly society must be governed by established principles and known codes that are applied uniformly and fairly to all of its members, also called supremacy of law. has to have established principles and Applied fairly!

Concealment of a dirk or dagger

The mere possession of a dirk or dagger alone is not a crime. It must also be shown that the weapon was carried concealed on the individual's person. A dirk or dagger is not considered to be concealed if it: • is carried in a sheath which is worn openly suspended from the waist of the wearer • is folded or closed and is not capable of "ready use" • consists of a cylindrical device which would have to be unscrewed, turned around, and then screwed back on before it could be used as a knife

psycholegal error

The mistaken belief that if we identify a cause for conduct, including mental or physical disorders, then the conduct is necessarily excused.

Substantive Criminal Law

The part of the law that defines crimes and specifies punishments

Elaborate Conspiracy

The parties to the conspiracy may not all be aware of one another's identity or even involvement, sometimes described as wheel or chain conspiracies

Causation in Fact vs. Proximate cause

The people who live at house A negligently start a fire that causes their house to burn down. The fire also causes house B nextdoor to burn down. The fire travels all the way to house Z (burning all the houses in between, as well). Is house A liable for ALL the houses that have burned? Many states will only find house A liable for the damage/injury of house B ... most will not allow Z to make a claim against A, because the damage is too remote. It was probably not foreseeable that the fire would travel so far (and there are other factors like the wind and temperature, etc.) the caused the fire to travel so far. Exactly WHERE the line is drawn varies from state to state and court to court. I stab you in the leg. You go to the hospital for treatment, where you slip on a puddle, hit your head on the floor, and die due to brain damage caused in the fall. Though this passes the "cause in fact" but-for test, but for my stabbing you never would have slipped in the hospital, it doesn't pass the proximate cause test. You slipping and falling in the hospital wasn't at all forseeable and I can't be held responsible for it. For example, if Driver A runs a red light and hits Driver B, injuring him, Driver A's act of running the red light is the likely actual cause of Driver B's injuries. In other words, Driver B would not have been injured but for Driver A's running the red light. However, actual causation can sometimes be quite broad, drawing otherwise unrelated events into the chain of causation. Because of this, proximate cause - the second type of causation - functions as a limitation. A woman is pushing a baby carriage down the street when she witnesses Driver A's car accident with Driver B (described above). She is so distracted by the accident that she lets go of the carriage momentarily and it runs away from her, striking a pedestrian and causing him to fall, hurting his back. Using only an actual cause analysis, it is likely that Driver A is indeed responsible for the pedestrian's back injury. This is because but for Driver A's running the red light, there would not have been an accident in the first place. With no accident to distract the woman, she would not have let go of the carriage and so the pedestrian would never have been struck and never would have fallen. Yet there may not be proximate cause because a pedestrian getting hit by a baby carriage is probably not a foreseeable result of running a red light. Since there is not likely proximate causation, a court is not likely to find Driver A liable for the pedestrian's back injuries.

At the Age of Responsibility

The person is charged as an adult and is considered a criminal, there is not a possibility for a defense of infancy (regardless of mentality) and the person is usually convicted as Found Guilty

not guilty by reason of insanity (NGRI)

The plea of a defendant, or the verdict of a jury or judge in a criminal proceeding, that the defendant is not guilty of the offense charged because at the time the crime was committed, the defendant did not have the mental capacity to be held criminally responsible for his or her actions.

The Difference Between the Characteristics Of A Crime and Other Deviance

The presence of a public prohibition and the authority of the government to enforce the prohibition

Defense Counsel vs. Prosecutor

The prosecutor represents the complainant and the state to present evidence to prove that the accused is guilty of the crime charged. The defense attorney presents evidence to disprove the allegations of the complainant and witnesses.

Legislative History

The record of debates, committee reports and meetings, legislators' statements. and other evidence of what the legislature intended when it enacted a particular statute

What is Legislative History?

The record that develops as an idea makes its way through the legislative process. Attorneys use legislative history to determine the intended purpose of a statute when the language of the statute is unclear.

Premeditated murder and manslaughter

The result of the crime might be the same but sentencing might be different you have to look at the intent of the person.

Procedural Law

The rules by which a court hears and determines what happens in civil lawsuit, criminal or administrative proceedings.

Natural Law

The rules of conduct inherent in human nature and on the natural order, which are thought to be knowable through intuition, inspiration, and the exercise of reason without the need to refer to man-made laws (Inherently Known)

Concurrence

The simultaneous coexistence if (1) an act in violation of the law and (2) a culpable mental state

Mens Rea

The specific mental state of the defendant at the time of the crime; a guilty mind. Two forms of mens rea are general and specific intent

Reasonable Doubt Standard

The standard of proof necessary for conviction in criminal trials

Abandonment Defense to Attempt

The voluntary and complete abandonment of the intent and purpose to commit a criminal offense. Abandonment is a defense to a charge of attempted criminal activity. Also calles renunciation

Arson

The willful and malicious burning of a building (and, in some states, vehicles and other items of personal property) is the crime of arson. At common law, arson applied only to burning down another person's house. The law was designed to protect human life. Today, arson statutes have been extended to cover the destruction of any building, regardless of ownership, by fire or explosion. Every state has a special statute that covers the act of burning a building for the purpose of collecting insurance. (Of course, the insurer need not pay the claim when insurance fraud is proved.)

PC 417(b): Drawing, Exhibiting, or Unlawful Use of a Firearm at a daycare center (Felony)

To arrest a person for drawing or exhibiting a firearm in a rude, angry or threatening manner, or for unlawfully using it in any fight or quarrel on the grounds of any day care center, the necessary crime elements are: any person who, while in the presence of another, except in self-defense draws or exhibits any loaded firearm in a rude, angry, or threatening manner unlawfully uses a loaded firearm in a fight or quarrel on the grounds of a day care center when that center or facility is open for use

4. Harm (to some other/thing)

To be a crime, an act must cause harm to some legally protected value. Questioned by some because they may be causing harm only to themselves

18. ATTEMPT - CONDUCT (COMMON LAW & MPC)

To be guilty of attempt, a defendant must engage in an "overt act" which goes beyond mere preparation. The common law is more hesitant to impose punishment where no harm has been committed, while the MPC is much more strict for attempt crimes. Courts are ultimately looking for enough conduct to determine if the defendant was seeking to convert resolution into action. Legislatures are seeking to criminalize excessive risk creation. Physical Proximity Test: D must have achieved physical proximity to the completion of the offense such that there was a reasonable likelihood of its accomplishment. (Rizzo) Modern Test: An overt act is enough conduct that shows that the Defendant was "committed to the offense". (Bowen, Rise). Aggregate Dangerousness Test (Holmes Test): Considers the (1) nearness of the danger; (2) greatness of the harm; and (3) the degree of apprehension felt. If the expected value of these factors is sufficiently high, then find conduct sufficient. (Rizzo) MPC Corroboration Test: The defendant must complete a substantial step in a course of conduct planned to culminate in the commission of a crime that strongly corroborates the actor's criminal purpose. Consider: 1. Lying in wait, searching for, or following the victim 2. Enticing or seeking to entice the contemplated victim to go to the place contemplated for its commission 3. Scouting the place contemplated for commission of the crime 4. Unlawful entry of a structure in which it is contemplated the crime will be committed 5. Possession of materials to be used in commission of the crime 6. Soliciting an innocent agent to engage in conduct constituting an element of the crime Statutory Rule: Solicitation and conspiracy, while distant from commission, are criminalized by statute. Other Considerations: a. Res Ipsa Loquitur: If there is more than one reasonably possible intent based on D's conduct, then conduct is not sufficient for an attempt. Not a stand-alone tests; used to support other tests. b. Intervention: should police intervene sooner or later, based on conduct rules? c. Abandonment: should D be given more time to reverse course? d. Deterrence: Isn't a lax rule bad, because it undermines the purpose of deterring dangerous people? New York v. Rizzo: Men drove around town with guns looking to rob a payroll man but he was nowhere to be found. - Physical Proximity Rule: If in all reasonable probability the crime itself would have been committed but for timely interference, then the conduct was sufficient to establish attempt. Essentially assessing level of dangerousness. Michigan v. Bowen: Police caught men in the act of robbing an old woman's house while she was home. - Corroboration Rule: Conviction for larceny requires (1) felonious intent and (2) an overt act going beyond mere preparation. To go beyond mere preparation, a defendant must demonstrate enough conduct to show they intend to convert resolution into action. United States v. Harper: Defendant sabotaged an ATM and waited for the repairman to show up so he could rob him. - Rule: Making an appointment with a potential victim is not of itself such a commitment to an intended crime as to constitute an attempt, even though it may make a later attempt possible.

Liability of Legal Entities

Today, corporations normally are liable for the crimes committed by their agents and employees within the course and scope of their employment. For liability to be imposed, the prosecutor generally must show that the corporation could have prevented the act or that a supervisor authorized or had knowledge of the act. In addition, corporations can be criminally liable for failing to perform specific duties imposed by law (such as duties under environmental laws or securities laws).

A building is considered inhabited if it is customarily used as a dwelling even if the residen is temporarily absent

True

One who advised and aided a friend to have sexual relations with a 16-year old girl could be guilty of unlawfully sexual intercourse even though he was not present when the act was committed.

True

One who engages in lewd or dissolute conduct in a public place is guilty of disorderly conduct

True

One who solicits or agrees to engage in an act of prostitution is guilty of disorderly conduct under PC 647(b)

True

Taking soil from another's vacant lot without permission constitutes the crime of trespass

True

Tearing down a "no hunting" sign on another's property constitutes the crime of trespass

True

The use of offensive words other than in a public place does not constitute disturbing the peace

True

The word "abet" generally signifies knowledge and wrongful purpose.

True

There is no such crime as being an accessory to a misdemeanor

True

To be guilty of window peeking, the building must be inhabited under PC 647(i)

True

To constitute a riot, the object of the assembly need not necessarily be unlawful

True

What are Trial Courts called in the Federal Court System?

United States District Courts

Myths of insanity defense

Used all the time to get out of things -Used in 1% of felony cases -Fails about 75% of the time -70% of cases, which it is successful, because both prosecution and defense in agreement -less than 1/3 of insanity pleas involve victim death - spend equal or longer time in mental facility if found NGRI Think experts can't agree on person being insane -8-year period, 92% agreed person was schizophrenic. -usually background of mental history

Regulatory offenses.

What are strict liability offenses?

Specific intent, general intent, transferred intent, motive, strict liability offenses.

What five terms are related to Mens Rea.

The state asks people to do what is reasonable, it does not ask people to do what is unreasonable.

What is the reasonableness test?

If you intend to harm one person or thing and harm or injure another person or thing the intent will be transferred.

What is transferred intent?

Statutory duties, contractual duties, voluntary assumption of care, duty based relationships, duty based on creation of peril.

When do you have a legal duty to act?

Exigent Circumstances

When law enforcement has probable cause and no sufficient time to secure a warrant

When Does Jeopardy Attach?

When the first witness is sworn in jury trials A guilty plea has been accepted by the trial court The first witness is sworn in bench trials

What is "Granting Certiorari"?

When the highest appellate court grants the petition of an unsatisfied party to review the intermediate court's decision.

PC 16530(a): Statutory definition of a concealed firearm (continued)

Whether or not a firearm is actually physically concealed has little to do withwhether it meets the statutory definition of a concealable firearm. Based on this definition, handguns are concealable. Shotguns and rifles, with barrel lengths greater than 16 inches, are not concealable. Any device which has a barrel 16 inches or more in length which is designed to be interchanged with a barrel less than 16 inches in length may also be considered a concealable firearm. Concealable firearms also include flare guns, rocket launchers, and some tasers for the purpose of this section.

Statue of Limitations

With some exceptions, such as the crime of murder, statutes of limitations apply to crimes just as they do to civil wrongs. In other words, the state must initiate criminal prosecution within a certain number of years. If a criminal action is brought after the statutory time period has expired, the accused person can raise the statute of limitations as a defense. The running of the time period in a statute of limitations may be tolled—that is, suspended or stopped temporarily—if the defendant is a minor or is not in the jurisdiction. When the defendant reaches the age of majority or returns to the jurisdiction, the statutory time period begins to run again.

malice

Wrongful act done intentionally without justification or excuse.

Entrapment

a defense that claims the defendant would not have broken the law if not tricked into doing it by law enforcement officials, a defense that claims the defendant would not have broken the law if not tricked into doing it by law enforcement officials

Charged Define

a formal complaint, indictment, or information has been filed against the person being concealed.

what is Escape?(836.6(a)),(836.6(b)) & (4530(a)),(4530ba))

a person who attempts to escape or escapes from from the custody of an officer or jail, they must know that or reasonably should know that they are under arrest. - escaping from an officer without violence is a misdemeanor - escaping from an officer WITH violence that causes serious injury is a felony - escaping from jail or prison with or without violence is a (felony) "escape early"

PC 16530(a): Statutory definition of a concealed firearm

a pistol, revolver, and firearm capable of being concealed upon the person shall apply to and include:any device designed to be used as a weapon from which is expelled a projectile by force of an explosion or other form of combustion that has a barrel less than 16 inches in length

Conceptual Intergration

a type of theoretical integration in which a theoretical perspective consumes or uses conceptsfrom many other theoretical models

Abrogate

abolish, nullify, repeal

misdemeanor

an offense less serious than a felony and which may be punished by a fine or sentence to a local prison for less than one year

PC 971The distinction between an accessory before the fact and a principal, and between principals in the first and second degree is abrogated( non existent);

and all persons concerned in the commission of a crime, who by the operation of other provisions of this code are principals therein, shall hereafter be prosecuted, tried and punished as principals and no other facts need be alleged in any accusatory pleading against any such person than are required in an accusatory pleading against a principal. No distinction between principles in the first and second degree

PC 417(c): Drawing, exhibiting, or unlawful use of a firearm in the presence of a peace officer (Felony)

any person who, in the immediate presence of a peace officer draws or exhibits any firearm, whether loaded or unloaded in a rude, angry or threatening manner who knows or reasonably should know, by the officer's appearance, action or identification that he or she is a peace officer engaged in his or her performance of their duty

PC 247: Unlawfully discharging a firearm (Felony)

any person willfully and maliciously discharges a firearm at an: unoccupied aircraft, unoccupied motor vehicle, uninhabited building or dwelling house

PC 23920: Knowledge of the alteration to the identification marks of a firearm

any person with knowledge of the alteration to the identification marks of a firearm to: buy, receive, dispose of, sell or offer for sale, possess that weapon

Coroner's Jury

appointed by coroner to hear evidence as to cause of death.

Conduct Crimes

are those in which the actus reus is concerned with prohibited behavior regardless of consequences (example: blackmail).

836.6(b) PC

attempting to escape or escaping custody of a police officer. felony.

4532 PC

attempting to escape or escaping from county jail. felony.

4530(a) PC

attempting to escape or escaping state prison with violent force. felony

148(d) PC

attempting to remove an officers fire arm. felony

Define: Disarming an officer by attempting to remove a firearm (148(d))

attempting to remove or take a firearm from the immediate presence of the officer or public officer while in the performance of his or her duties

4550 PC

attempting to rescue or rescuing someone from a police officer or lawful custody. felony.

Short barreled rifle

barrel length of less than 16 inches overall length of less than 26 inches The barrel length is measured from the closed breach face to the tip of the muzzle excluding any removable flash suppressors

A deliberate physical attack upon a person is called

battery

Custom

behaivor which has become traditional with time

Corpus delicti

body of crime

Canon law

church law

permission granted by a person voluntarily and in his right mind is

consent

An agreement between two or more parties for the doing or not doing of some definite thing is

contract

A patient's failure to act prudently and reasonably, or doing that which a reasonable person would not do under similar circumstances is called

contributory negligence

Estabar motion

court can reduce type of crime during a hearing

Subpoena

court order commanding witness to appear in court

Commitment

court order sending person to jail, hospital, etc.

Jurisdiction

court's legal authority to act

An act that violates criminal law is called

crime

Crimen falsi

crime involving falsification

Conspiracy

crime of two or more person planning to commit a crime

Injuring the name and reputation of another person by making false statements to a third person is

defamation

A person being sued is called the

defendant

Constructive possession:

defendant knowingly exercises control over, or the right to control an object either directly or through another person. These aren't my pants defense (when they say its not there pant when they are searched and drugs are found), Need not be in D's actual possession if close enough for him to exercise dominion and control over it

Video Watched: Dani Palm; John harper Jr. wreakless driving. Dani palme shot Jr and testisfied against Jr. hung jury Jr plead misdemeanor. Palm shot JR

defense of unconsciousness Convicted of 2nd degree murder, charge dropped to manslaughter

148(a)(1) PC

delaying an officer in the performance of their duties. misdemeanor.

testimony of a witness under oath and written down before trial for possible use when the case comes to trial is

deposition

Culpable

deserving the blame, guilt or censure.

burglary

entering a building unlawfully with intent to commit a felony or to steal valuable property

PC 17500: Possession of a deadly weapon with intent to commit assault (Misdemeanor)

every person having upon him or her person any deadly weapon with intent to assault another NOTE: A deadly weapon is any weapon that when used offensively or defensively is capable of producing and is likely to produce, death or great bodily injury.

PC 25800(a) Possess a loaded firearm with intent to commit a felony (Felony)

every person who carries a loaded firearm with the intent to commit a felony NOTE: A firearm shall be deemed to be "loaded" (Penal Code Section 25800) whenever both the firearm and the unexpended ammunition capable of being discharged from the firearm are in the immediate possession of the same person PC 16840(a).

Corroboration

evidence which confirms, supports, substantiates

PC 246.3: Discharging a firearm in a grossly negligent manner (Felony)

except as otherwise authorized by law willfully discharges a firearm in a grossly negligent manner which could result in injury or death to a person Gross Negligence-conduct which is more than ordinary negligence (i.e., failure to exercise ordinary or reasonable care).

PC 26100(d): Discharging a firear from a motor vehicle (Felony)

except as provided in Fish and Game Code Section 3002 any person who willfully and maliciously discharges a firearm from a motor vehicle

A statement given concerning some scientific, technical, or professional matter by an expert, such as a physician is called

expert testimony

One cannot be guilty of being a principle to a crime if he was not present when the crime was committed.

false

The criminal law does not provide for punishment of those who merely assist another in committing a crime

false

The term "principal to a crime" apples to felonies only.

false

A "feigned accomplice" is one who did not know the wrongfulness of the act committed

false, one who participates in a crime for evidence gathering and prosecution purposes only

A major punishment is imposed other than a misdemeanor is

felony

A fraudulent signature is called.

forgery

An intentional perversion of truth for the purpose of inducing another in reliance upon it to part with some valuable thing belonging to him or to surrender a legal right is calld.

fraud

Cyber Fraud

fraud is any misrepresentation knowingly made with the intention of deceiving another and on which a reasonable person would and does rely to her or his detriment. Cyber fraud is fraud committed over the Internet.

Venue

geographical location of court hearing a case.

Contraband

goods forbidden by law to possess, import, etc.

Switchblade knife

having the appearance of a pocket knife, including: - spring-blade knives - snap-blade knives - gravity knives - any other similar type knife • a blade two or more inches long which can be released automatically by: - a flick of a button - pressure on the handle - flip of the wrist (butterfly knives), or other mechanical device - the weight of the blade - any other mechanism This definition does not include knives designed to be opened with one hand utilizing thumb pressure applied solely to the blade or a thumb stud attached tothe blade.

Classic voluntary manslaughter

heat of passion- walking in on ur cheating spouse in ur bed and shoot them.

what is rescue? (PC4550)

helping to or attempting to help someone to escape prison or jail or officer having lawful custody (felony)

if someone buys with the intent to buy stolen property and its not actually stolen then its attempt

if someone buys with the intent to buy stolen property and its not actually stolen then its attempt

lack of physical or mental fitness is known as

incompetence

Deterrence perspective on punishment

individual offender should be punished so that they learn that committing a crime leads to punishment (specific deterrence) and so that other individuals like them will learn vicariously through them (general deterrence) -if mentally ill, they won't be deterred by punishment, and others who are mentally will not learn from them being punished.

136.1 PC

intimidating a witness. felony.

The final decision of a court in an action or suit is

judgment

Court trial

jury waived; judge determines guilt or innocence.

Justification versus Excuse

justification declares the allegedly criminal act legal; excuse admits the act's criminality, but declares the allegedly criminal actor not to be worthy of blame Excuses admit that the action committed by the defendant was wrong and that it violated the criminal law but claim that the defendant should be excused from criminal liability by virtue of special conditions or circumstances that suggest that the actor was not responsible for his or her deeds. The majority of excuses are personal in nature, that is, they claim that the defendant acted on the basis of some disability or some abnormal condition, such as intoxication, insanity, or immaturity. Even when a defendant suffers from a disability, however, that disability alone is not sufficient to excuse him or her of criminal responsibility. Only when the disability has the effect of in some way contributing to the criminal activity in question will the actor be excused. Like justifications, excuses are affirmative defenses and must be raised by the defendant.

What is the crime of preventing or disuading witnesses or victims by intimidation to testify (P.C. 136.1 (a)) "intimidation of a witness"

knowingly and maliciously prevents or dissuades or attempts to disuade any witness or victim from going to court to testify (felony)

a rule of conduct made by a government body is

law

Contemporary

modern or current

What license must a physician have to dispense, prescribe or administer controlled substances?

narcotic

Failure to do something that a reasonable person would do under ordinary circumstances that ends up causing harm to another person or a person's property is

negligence

The most common type of medical tort liability is

negligence

Lawful tear gas weapon

net weight of 2.5 ounces or less solely for self defense. Exception has been convicted of a felony or crime involving assault, addicted to narcotics, attempting to sell the tear gas to a minor, under 16, is 16 or 17 without written parental consent and not accompanied by parent

Omissions:

no act at all but still considered a crim -must have a legal duty to act - PC 142: any peace officer who has the authority to arrest a person charged with criminal offense and willfully refuses .... Peace officer refusing to receive or arrest parties charged with a crime. FELONY.

A latin term signifying that a person is not of sound mind is

no compos mentis

Bailiff

officer assigned to keep order in the court

End-to-end integration

one theory will come before or after another in terms of temporal ordering of casual factors-tends to propose a certain ordering of the component theory that are being emerged-better used for theories that focus more on remotes causes of crime - two or more theories that focus on more immediate causes of crime would be harder to combine ex: weak social bond—>peer associations—> crime

Decedent

one who is dead, the deceased

Accomplice

one who is liable to prosecution for the identical offense charged against the defendant on trial in the cause in which the testimony of the accomplice is given

Authorized persons for possessing striking weapons

peace officers-autorized for enforcement private security guard-course of employment private security officer-has shoulder patches animal control-with baton course

Medical research is often published in medical journals that are evaluated by members of the medical community through a process known as.

peer review

Define: providing false ID to a peace officer? (PC148.9)

present yourself as another person real or ficticious, to a peace officer, to avoid going to court or getting a ticket - Note: have to be lawfully arrested or detained (misdemeaner) - The Vehicle Code states in (VC31) that no person shall knowingly give false information, written or oral, to a peace officer who is engages in enforcing the vehicle (misdemeanor)

The parties to crimes are classified as

principles and accessories

Difference between probation and parole

probation (no time served)and parole (time served).

Define: Falsly reporting an emergency (PC148.3(a))

reports any emergency to the authorites knowing the report is false (misdemeanor) - Note: this crime is a (felony) if the person reporting knows or should know that the response is likely to cause great bodily injury or death and such injury is sustained as a result of the response

The presumption or inference of negligence when an accedent is otherwise unable to be explained in terms of ordinary and known experience is called.

res ipsa loquitur

What is lynching? (405a)

taking by means of riot(by means fo force by two or more people) of any person in the lawful custody of a peace officer - felony

405a PC

taking someone from custody by mean of riot from a police officer. felony.

Circumstantial Evidence:

that from which other facts may be concluded.

3 reasons why american industrialism ened when it did

the civil war, governement support, technological breakthrough

Which one of the following might involve a decision based upon bioethics?

the use of fetal tissue transplantation for research

Accessories

this, such as friends or relatives who hide a criminal, who assist the perpetrator after the crime, in order to help him avoid detection or apprehension

Nullify

to dilute or abolish

Theft of Trade Secrets and Other Intellectual Property

trade secrets constitute a form of intellectual property that for many businesses can be extremely valuable. The Economic Espionage ActFootnote makes the theft of trade secrets a federal crime. The act also makes it a federal crime to buy or possess another person's trade secrets, knowing that the trade secrets were stolen or otherwise acquired without the owner's authorization. Violations of the Economic Espionage Act can result in steep penalties: imprisonment for up to ten years and a fine of up to $500,000. A corporation or other organization can be fined up to $5 million. Additionally, the law provides that any property acquired as a result of the violation, such as airplanes and automobiles, is subject to criminal forfeiture, or seizure by the government. Similarly, any property used in the commission of the violation, such as servers and other electronic devices, is subject to forfeiture. A theft of trade secrets conducted via the Internet, for instance, could result in the forfeiture of every computer or other device used to commit or facilitate the violation as well as any assets gained.

Who are principals

All person concerned in the commission of a crime, whether it be felony or misdemeanor, or whether they directly commit the act constituting the offense, or aid and abet in its commission, or not being present, have advised and encouraged its commission, and all persons counseling, advising, or encouraging children under the age of fourteen years, or persons who are mentally incapacitated, to commit any crime, or who, by fraud, contrivance, or force, occasion the drunkenness of another for the purpose of causing him to commit any crime, or who, threats, menaces, command, or coercion, compel another to commit any crime, are principals in any crime so committed.

Parties to Crime

All those who take part in the commission of a crime, including those who aid and abet and are therefore criminally liable for the offense

PC 817

Allows officer to obtain an arrest warrant prior to any criminal charges being filed

Receiving Stolen Goods

It is a crime to receive goods that a person knows or should have known were stolen or illegally obtained. To be convicted, the recipient of such goods need not know the true identity of the owner or the thief, and need not have paid for the goods. All that is necessary is that the recipient knows or should know that the goods are stolen, which implies an intent to deprive the true owner of those goods.

15. FELONY MURDER (MPC)

MPC FELONY MURDER 1. MPC rejects felony murder rule by requiring that evidence be submitted to the jury to determine recklessness and indifference (similar to Michigan v. Aaron). Commission of a felony creates a presumption of extreme recklessness and that presumption goes to the jury unless the Court finds evidence that the presumption was inaccurate. A jury must still find extreme recklessness in relation to the death but it's not necessary that they do. a. Finally, the MPC is an example of eliminating SL here. Still has the felony murder rule here. Raises a presumption of extreme recklessness for murder. Its not a conclusive presumption, it's a rebuttable presumption.

Steagald Warrant

Arrest may not be made in a 3rd parties home without a Steagald Warrant, or search warrant to search the premises for the person. (Steagald v. US 1981)

Field Code

Drafted by the jurist David Dudley. A set of proposed standardized criminal and civil procedures and uniform criminal statutes that were adopted by the state of New York and served as a model for other states that sought to codify their laws

Identity Theft

Not surprisingly, there has been a marked increase in identity theft in recent years. Identity theft occurs when the wrongdoer steals a form of identification—such as a name, date of birth, or Social Security number—and uses the information to access the victim's financial resources. The Internet has provided even easier access to private data, as we will discuss further in Chapter 15. Frequent Web surfers surrender a wealth of information about themselves without knowing it. Most Web sites use "cookies" to collect data on those who visit their sites. Web browsers often store information such as the consumer's name and e-mail address. Finally, every time a purchase is made online, the item is linked to the purchaser's name.

Civil Law

Pertains to the duties that exist between persons and their government.

The courts do not recognize the existence of "fighting words" as a basis for disturbing the peace

False

The taking by means of riot of any person from the lawful custody of an officer constitutes a crime of The taking by means of riot of any person from the lawful custody of an officer constitutes a crime of rout

False

Under P.C. 26 a person under the age of 14 cannot be held criminally responsible for a crime. T/F

False

True or false: To be charged with aiding and abetting one must be present at the time the crime is committed

False - One does not have to be present to be guilty of aiding and abetting. However, merely being present at the time of the crime with out assisting does not constitute as aiding or abetting, nor does knowledge of the crime but failing to prevent.

The punishment for an accessory is one-half of that for a principal.

False, a felony "wobbler," punishable by imprisonment in the state or in the county jail not exceed..

Possession of a deadly weapon with intent to commit a felony

Felony every person who carries a loaded firearm with the intent to commit a felony

Different levels of 594s (vandalism) because of damages.

Felony is served in state prison if there is no other option by law (when it says or county jail not exceeding one year) (key word is State prison for felony) 594 B1= damgage 400$ or more state prisoner or jail time (wobbler) 594 2A= less than 400$ jail for less than a year OR fine of 1000$ Wobblers are determined by DA (17 B4)or Judge (17 B5) under PC 17 If DA issues under a misdemeanor jusge cannot kick up to felony 594 B2= less than 400$ can be in jail less than one year. (this is a misdemeanor)

15. FELONY MURDER (COMMON LAW)

Felony murder requires: (1) satisfaction of all formal requirements for the underlying felony, and (2) death during the felony. If D commits a felony and a death results, then D is guilty of murder. Felony murder is a strict liability offense but mistake of fact aside from death is still a defense. Rationale: 1. Severe penalties for deaths during felonies deter criminals from committing felonies; 2. Independent moral wrongs rule from Prince justifies strong retributivist penalties; 3. When D commits felony, he supplies "implied malice" for a more severe penalty (Faulkner prosecutor) Criticism: 1. Law cannot deter unintended acts or very rare results; 2. Deterrence of felonies requires increasing punishments for felonies, not deaths that occur during felonies. 3. Grading standards essentially assess degrees of risk and felony murder's risk is lower than negligence, but it is graded at the same level of culpability as murder. The law can't deter unintended events. Notes: Smith case is example for objective test of murder; agree that there is enough risk created to justify murder conviction or something lower 2. Modern limitations to felony murder a. Merger Rule (All Jurisdictions): Felony underlying the felony murder must be "independent" of the particular acts that bring about the death. Otherwise, the manslaughter-murder distinction becomes irrelevant. i. Manslaughter/assault are not independent enough, but rape, arson, or burglary are. b. Proximate Cause Rule (Majority Rule): D's felonious conduct must have been the genuine proximate cause of the death, meaning death must have been foreseeable to D. c. Inherently Dangerous Rule (Majority Rule): D is guilty of felony murder only if the felony is "inherently dangerous," determined either statutorily or through common law. d. Abolition (Three Jurisdictions Only; MPC Rule): Complete removal of the felony murder rule. 3. Cases a. Michigan v. Aaron (rejection of felony murder rule) Mich. 1980 RULE: "Malice" (intent to kill or inflict GBI, or extreme recklessness) is required element of any murder conviction and it is a question for the jury as to whether that intent is present.

Facts: Two men assaulted ∆'s sister in law, robbed her, and took several pieces of jewelry. They also cut the phone lines. ∆ was arrested while in possession of some jewelry, and eh provided information that led to the arrests of the two men. The two men testified that ∆ assisted in the commission of the crime by providing them with the floor plan of his sister in law's house and by telling them of the contents of the house and by agreeing to dispose of the loot. ∆ said that shortly before the robbery he said he wanted no part of it and was angry that the robbery had been committed. Issue: Was ∆ entitled to a jury instruction about his state of mind? Did Beeman aid or abated?

Holding: Yes. Reasoning: • conviction for aiding and abetting requires: • proof that the defendant acted with the knowledge of the criminal purpose • proof that the defendant acted with the intent of committing, encouraging, or facilitating the offense • The court said that knowledge was required • However, intent is required • There will be intent when the aider and abettor knows the full extent of the principal's criminal purpose and gives aid or encouragement with the intent or purpose of facilitating the principal's commission of the crime • Intent was central, so there should have been a jury instruction about it

People V Decina 2. Facts: Decina was an epileptic. While driving his car, he had an epileptic seizure and the car went out of control, killing four people. 3. Procedural Posture: Decina was convicted of negligent homicide. Both he and the prosecution appealed from the trial court's order for a new trial because of an error in the admission of evidence. 4. Issue: Whether an epileptic person commits a crime when the epileptic knowingly and voluntarily drives an automobile without assistance during a time when a seizure is possible

Holding: Yes. Negligent homicide

Why not punish thoughts?

Innocence Impossibility Fairness Volume

Circumstantial evidence

Instruments of the crime: Examples: * Condom *Slim jim, Pry tools *ski mask

Transferred intent

Intended act misses and falls on a 3rd unintended party

Convicted felons and addicts

Prohibited from possessing firearm - FELONY (12021) Inoperable weapons, police may inspect firearms if seen.

...

Prohibited to carry weapons at airport

Self-Fulfilling Prophecy

Results in individual confirming their status as criminals or delinquents by increasing the frequency or seriousness of their illegal activity.

Mental State at the Time of Offense Screening Evaluation (MSE)

Slobogin, Melton, & Showalter (1984) -attempts to screen out defendants whose crimes not influenced by mental disorder examiners gather and evaluate info about defendants history of mental disorders, offense itself, and current mental state -criticized for lacking clear scoring system and strict procedures for administering test (Nicholson, 1999)

Objective Approach or Test

Standard: Does government behavior create substantial risk that someone not predisposed will commit the offense? Evidence: Government's conduct Effect: If government's conduct is excessive, then defendant will be acquitted regardless of predisposition Where Used: MPC and minority of states The objective test to assessing entrapment is based on "the belief that the methods employed on behalf of the Government to bring about conviction cannot be countenanced."35 If government agents have acted in a way that is likely to instigate or create a criminal offense, regardless of the defendant's predisposition to crime, then—according to the objective approach—the defendant can successfully raise the defense of entrapment. The objective approach to assessing entrapment is sometimes referred to as the defense of outrageous government conduct.

Subjective Approach (or test)

Standard: Is defendant predisposed to commit the crime? Evidence: Defendant's record, statements, etc. Effect: If defendant is predisposed, then he or she will be convicted regardless of government conduct. Where Used: Federal government and majority of states The test excludes from criminal liability people who are "otherwise innocent, who have been lured to the commission of the prohibited act through the Government's instigation."32 The subjective test attempts to distinguish between those who are blameworthy and those who are not, by asking whether a person "caught" by the government was predisposed to commit the crime in question. In doing so, it distinguishes "unwary criminals," who are ready and willing to commit the offense when presented with a favorable opportunity, from those who are not. Some courts have held that predisposition can be established by demonstrating a defendant's (1) prior convictions for similar crimes, (2) reputation for committing similar crimes, or (3) readiness to engage in a crime suggested by the police.33 The subjective test for entrapment uses the criterion of "origin of intent." It asks, "Did the criminal intent start in the mind of the officers, or was the defendant 'predisposed' to commit the offense when the officer first appeared on the scene?"34 Hence, following the subjective approach, traps may be legitimately laid by the government only for those who are already bent on crime.

What laws does a state jurisdiction enact?

State Law

Sources of Criminal Law

State and federal constitution, statutes and codes, judicial decisions, administrative agencies, common law

What are the two types of legislative authority?

Statues Legislative History

What type of law does the Legislative Branch produce?

Statutes

An accomplice is liable to prosecution for the identical offense as that charged against the principal.

True

Criminal guilt can be based on assistance willfully given for the purpose of facilitating another's commission of a criminal offense

True

If "A" helps "B" to voluntarily take a shot of heroin, and "B" dies from an overdose, "A" can be charged with manslaughter as being a principal to the death.

True

It is a crime to remain at the scene of a riot after having been warned to disperse, even if one is only an observer

True

It is possible for the one who commits an act not to be guilty of a crime, but the one who aids and encourages such act to be guilty.

True

Loitering around a public toilet for the purpose of engaging in any unlawful act is in violation of PC 647(d), disorderly conduct

True

No agreement to engage in an act of prostitution is a crime unless some act, besides the agreement is done

True

One can be guilty of being an accessory by helping another to plan a robbery.

True

22. ACCOMPLICE LIABILITY - COMPLICITY: CONDUCT

Very little conduct is usually required for complicity as opposed to attempt because the harm has already been committed. Common Law Rule: Accomplice's conduct requires actual aid or encouragement that must have in fact assisted in the crime's commission. Attempted aid is not sufficient. Innocent Agency Rule: If D causes another person to do the actus reus of a crime in circumstances that render the doer innocent or insane, then D is directly guilty of the offense. Requires causation - distinct from normal complicity. MPC Rule: D is guilty of offense if someone to whom he is legally accountable commits an offense. D is legally accountable if (1) D causes innocent/insane person to engage in conduct, (2) statute defines defendant as accountable, or (3) he is an accomplice. - Defendant is an accomplice if he (1) solicits, (2) aids, agrees to aid, or attempts (substantial step analysis) to aid, (3) fails to perform a legal duty (omission), or (4) engages in conduct that falls under a statute. - Solicitation: Defendant encourages or requests another to engage in specific conduct that would constitute such crime or an attempt to commit such crime. Irrelevant whether the actor failed to communicate with the person solicited if his conduct was designed to effect such communication. o Attempted aid is sufficient to establish complicity. Most people will be covered through complicity under the MPC than under Common Law. Rex v. Russell: Husband stood by and watched as his wife drowned their children and then committed suicide. - Rule: Mere silence can establish an omission, regardless of legal duty, that creates accomplice liability, since silence can communicate to the principal an "assent" of the act. o Attendant circumstances may establish silence as assent (husband wasn't just a bystander). North Carolina v. Walden: Another person beat a mother's child in front of her and the mother failed to intervene. - Rule: Failure of a parent who is present to take all steps reasonably possible to protect the child from an attack by another person constitutes an omission showing the parent's consent and contribution to crime. o Still have to demonstrate that the mother had the appropriate mens rea. Alabama v. Tally: Defendant aided his brothers-in-law in commission of a murder but they were unaware of his aid. - Rule: D is guilty of aiding only if (1) D's actions were in preconcert or known to the principal, such that principal would be encouraged or (2) D's actions in fact aided the principal in the offense, despite the fact that the aid would likely not have changed the outcome.

Complaint

a criminal charge made to a court.

The branch of study of moral issues, questions, and problems arising in the practice of medicine and in biomedial research is called

bioethics

The breaking of a law, promise, or duty is called

breach

Which of these generally results from an act of carelessness, without an intent to harm?

breach of cofidentiality

Failure to achieve an agreed upon result, even when the highest degree of skill has been used, is called

breach of contract

violation or omission of a legal or moral duty is called

breach of duty

Burglary is punishable as follows: (b) Burglary in the second degree:

by imprisonment in the county jail not exceeding one year or in the state prison.

Certiorari

order from appellate court to hear an appeal.

Coition

sexual intercourse, also coitus.

Characteristics of Criminal Law

- List of commands, Do's and Donts - Punishment for violations - Applies equally to everyone - Victim or victimless crimes --Victim- rape, murder --Victimless- prostitution, gambleing, drug sales - Moral condemnation of the community

Purpose of punishment: to deter from doing crime

- Prevention o General deterrence o Specfci deterrenace o Incapacitiation - Retribution - Rehabilitation ( in juvy that's the main goal)

- Penalizing criminal behavior

- Punishment. Least amount of punishment to achieve compliance.

- Defining criminal behavior

- Robbery: taking property from someone else of some value with fear and force. -Definition to help determine elements of the crime

corpus delicti

"Total body of the crime". 1) Act or omission to act 2) Existence of criminal agency. The corpus delicti deals with the rules of evidence w/c govern the commission of crimes. Proof of act & criminal agency as opposed to harm caused by an act of loss, damage or injury by accident or misfortune.

8. CAUSATION (MPC 2.03)

(1) (Overview Section) Conduct is the cause of a result when: (a) (actual cause) it is an antecedent but for which the result in question would not have occurred; and (b) (proximate cause - defined further below) the relationship between the conduct and result satisfies any additional causal requirements imposed by the Code or by the law defining the offense. (2) When (higher intent) purposely or knowingly causing a particular result is an element of an offense, the element is not established if the actual result is not within the purpose or the contemplation of the actor unless: (a) (transferred intent) the actual result differs from that designed or contemplated, as the case may be, only in the respect that a different person or different property is injured or affected or that the injury or harm designed or contemplated would have been more serious or more extensive than that caused; or (b) (proximate cause) the actual result involves the same kind of injury or harm as that designed or contemplated and is not too remote or accidental in its occurrence to have a [just] bearing on the actor's liability or on the gravity of his offense. (3) When (lower intent) recklessly or negligently causing a particular result is an element of an offense, the element is not established if the actual result is not within the risk of which the actor is aware or, in the case of negligence, of which he should be aware unless: (a) (transferred intent) the actual result differs from the probable result only in the respect that a different person or different property is injured or affected or that the probable injury or harm would have been more serious or more extensive than that caused; or (b) (proximate cause) the actual result involves the same kind of injury or harm as the probable result and is not too remote or accidental in its occurrence to have a [just] bearing on the actor's liability or on the gravity of his offense.

Common Elements Can Be Identified In The Context of Criminal Cases (Similarities between judicial systems)

(1) All systems have trial courts where criminal cases are filed, warrants are issued, juries are found, witness and evidence are received, verdicts are rendered, and sentences are passed. In the federal system, there are 94 trial courts, known as U.S. district courts. There is at least one district court in each state (2) All systems have at least one appellate court. These courts do not impanel juries or hear evidence. Instead, they review the decisions of trial judges for legal error. They do this through written and sometimes oral arguments. In the federal system, the 11 U.S. courts of appeal are the first, and often last, rung of appeal. They are often final because not all cases continue to the highest level of appeal, the U.S. Supreme Court

Negligent (criminal negligence) Mens Rea

(1) Behavior in which a person fails to reasonably perceive substantial and unjustifiable risks of dangerous consequences. The risk must be of such nature and degree that the failure to perceive it constitutes a gross deviation from the standard of care that a reasonable person would observe in the situation (2) Negligence of such a nature and to such a degree that it is punishable as a crime. (3) Flagrant and reckless disregard for the safety of others, or willful indifference to the safety and welfare of others Did not think about possibility but if he had used ordinary common sense he would have realized there was substantial risk. Example: I should have been watching the road instead of talking on my cell phone and using GPS

Types Of Offenses

(1) Criminal acts (2) Taboo acts, "Deviant" behavior violation, violation of mores (3) Violation of norms, "Uncivilized" behavior, immoral behavior

When does a Series of Conspiracies Exist?

(1) Each "spoke" knows that the other "spokes" exist, although they need not know their precise identity (2) The various "spokes" have a community of shared interests

A person aids and abets the commission of a crime when he:

- With knowledge of the unlawful purpose of the perpetrator and - With the intent or purpose of committing or encouraging or facilitating the commission of the crime, and - By act or advice aids, promotes, encourages or instigates the commission of the crime

Ways The Criminal Liability is Influenced

(1) It may result in a finding that the mens rea required for a specific crime was lacking, leading the court to conclude that no crime occurred, or (2) it may lead to a showing that although the requisite mens rea was present at the time of the crime, the defendant should be excused from legal responsibility because necessary mens rea but is able to successfully raise the defense of insanity, a not guilty by reason of insanity (NGRI) verdict should be returned. In short, as a defense, insanity recognizes that some people, by virtue of mental disease or mental defect, cannot morally and justly be held accountable for their actions. of mental disease or defect. The first perspective allows that a given mental condition may negate an element of an offense, while the latter recognizes that a crime took place but excuses the perpetrator due to a lack of moral blameworthiness. If mens rea cannot be proven by the prosecution, a finding of not guilty should result. If the defendant possessed the

Substantial Step Toward the Commission of that Offense

(1) Significant activity undertaken in furtherance of some goal (2) An act or omission that is a significant part of a series of acts or omission, constituting a course of conduct planned to culminate in the commission of a crime (3) An important or essential step toward the commission of a crime that is considered sufficient to constitute the crime of criminal attempt. A substantial step is conduct that is strongly corroborative of the actor's criminal purpose. According to one court, a substantial step is "behavior of such a nature that a reasonable observer, viewing it in context, could conclude beyond a reasonable doubt that it was undertaken in accordance with a design to violate the statute"

Elements of Crime of Attempt

(1) Specific intent to commit a criminal offense (2) A substantial step undertaken toward the commission of the intended offense Elements of a crime: (a) Intent to commit crime (attempt) (b) failure to complete (attempt) (c) apparent possibility of commission (attempt) (d) Overact toward commission (actus reus) Example: if a husband laces his wife's cocktail with cyanide, it is no defense that by chance the intended victim decided not to drink the deadly potion. One defendant claimed he could not attempt rape in an old Model A coupe because it was too cramped to make the act possible. The court threw out this defense. Sometimes a criminal defendant is accused of both the crime (e.g. robbery) and the attempt in case the jury felt he tried but did not succeed.

Corpus Delicti Components

(1) That a certain result has been produced (2) Someone's criminal act as the cause of the injury

Elements of Crime

(1) The basic components of crime (2) In a specific crime, the essential features of that crime, as specifies by law or statute

Corpus Delicit Rule

(1) The body or essence of a criminal offense that proves that the alleged crime has been committed, but not who committed the crime (2) In practice, a principle of law that says that an out-of court confession, unsupported by other facts, is insufficient to support a criminal conviction

Six Steps for a Person to Intentionally Commit a Crime (Dessler)

(1) The individual conceives the ides of the crime (2) He or she then evaluates the idea (3) The person forms the intention to go forward (4) He or she prepares to commit the crime (5) The individual commences the acts necessary to complete the crime (6) Finishes the actions defined by law as necessary to complete the crime The person is not punished during the first three stages of the process. In the first two stages, the individual lacks a mens rea (intent). In addition, since people are not punished for thoughts alone, there is no crime until there is an actus reus. It is the activity in the middle range (the fourth and fifth stages), which comes after the formation of the mens rea but before the attainment of the criminal goal, that is legally considered inchoate, or incomplete, conduct

The M'Naughten rule's Possibilities

(1) a lack of mens rea (the person didn't know what he or she was doing) and (2) an acceptable legal excuse (the person didn't know that it was wrong). Either alternative provides an acceptable defense under M'Naughten

CATEGORIES OF EXCUSES

(1) duress, (2) involuntary intoxication, (3) mistake, (4) age, (5) entrapment, (6) insanity, and (7) diminished responsibility. All of these excuses are discussed in this chapter. Also discussed here is the special area of syndrome-based defenses, a form of excuse that has recently entered the legal limelight.

Requirements of a lawful arrest

(1) making the purpose of the arrest known to the person arrested and (2) using a valid arrest warrant (if the arrest is to be made on the authority of a warrant—something that is not necessary if the crime was committed in the officer's presence)

JUSTIFICATIONS AS A DEFENSE (an affirmative defense)

(1) necessity, (2) self-defense, (3) defense of others, (4) defense of home and property, (5) resisting unlawful arrest, and (6) consent

The Roles of Punishment of in the Principle of Legality

(1) nullen crimen, nulla poena, sine lege, which means "there is no crime, there is no punishment, without law" (2) nullum crimen sine poena, which means "no crime without punishment

Classification of Offenses into Four Types

(1) property crimes, (2) personal crimes, (3) public order offenses, and (4) morals offenses.

12. MURDER (MPC)

(a)Purposefully or knowingly causing the death, or (b) extreme recklessness regarding death 2.10.1 - Criminal Homicide (1) A person is guilty of criminal homicide if he purposely, knowingly, recklessly, or negligently causes the death of another human being. (2) Criminal homicide is murder, manslaughter or negligent homicide. Notes: Actus resus is simple; conduct that causes the death of another Mens rea is satisfied by any option 2.10.2 - Murder (1) Except as provided in 210.3(1)(b), criminal homicide constitutes murder when: (a) it is committed purposely or knowingly; or (b) it is committed recklessly under circumstances manifesting extreme indifference to the value of human life. Such recklessness and indifference are presumed if the actor is engaged or is an accomplice in the commission of, or an attempt to commit, or flight after committing or attempting to commit robbery, rape or deviate sexual intercourse by force or threat of force, arson, burglary, kidnapping or felonious escape. (2) Murder is a felony of the first degree... Notes: 1. Result based crime when the culpability is purposeful or knowingly (?) 2. Remains a result-based crime when recklessness is the culpability, but includes certain circumstances that must be satisfied with the culpability of recklessness. (?) 3. Reckless murder essentially retains the common law's depraved heart murder 4. MPC does not maintain grievous bodily injury (GBI)

Kenneth Bianchi

-5 months in 1977-1978, Bianchi "Hillside strangler" raped/strangled several young women an left bodies on hillsides in LA -Denied involvement in murders -under hypnosis, evil alter ego "Steve" surfaced and confessed -2 psychiatrists who examined him convinced that suffered from multiple personality disorder, and "Ken" unaware and not responsible for "steve" crime. -lawyers filed insanity. -Took expert on hypnosis to discover that Bianchi was pretending to be hypnotized and consciously inventing multiple personalities

Postpartum blues

-50-75% of mothers experience it -crying, irritability, anxiety and mood changes that can last two weeks.... linked to hormonal changes following childbirth

Durham Standard or the Product Test

-Accused is not criminally responsible if his unlawful act was product of mental disease or mental defect (Durham v. United States, 1954) -inserted notion that mental illness as possible cause of criminal behavior -courts responded w/ suspicion and hostility -thought jurors might put too much weight on expert testimonials Removed from DC courts in United States v. Brawner, 1972

Model Penal Code

-American Law institute - Has greatly advanced the pursuit of a rational crim code -A learned and influential doc -Bowman: I think it is useless for people who work in the field it is worthless

The M'Naghten Case, 1843

-Daniel M'Naghten (paranoid delusions of govt ppl trying to kill him) -set out to kill prime minister of England (Robert Peel) because thought was part of conspiracy. -Mistakenly shot and killed secretary -9 medical experts testified he was insane -Found him not guilty by reason of insanity. -spent rest of life in Broadmoor insane asylum -Consequence: M'Naghten rule

NGRI defendant characteristics

-Males who haven't committed violent crime, have no prior history of criminal offenses, and have history of hospitalizations for severe mental illness -psychosis, moo disorders, and mental retardation = most commonly associated mental illness (Melton 2007)

Smith v. United States (1954)

-Some states added IRRESISTIBLE IMPULSE into definitions of insanity to take into account defendants VOLITIONAL CAPACITY (inability to control their behavior) -defendant could be acquitted if reason dethroned by diseased mental condition depriving him of willpower to resist insane impulse to perpetrate the deed, though knowing it to be wrong -this amendment was short-lived -Problem: too hard to tell when impulse was irresistible. -attempt to clarify with policemen at the elbow test

Rogers Criminal Responsibility Assessment Scales (R-Cras)

-attempts to translate legal standards of insanity into components such as ability to control ones thoughts and ability to control behavior -25 items, each rated on numerical scale. Judgment on them based on in-depth interview as well as review of relevant documents (mental health records, police reports) -items quantified into 5 scales: Malingering, organicity (significant brain disorder), major psychiatric disorder, loss of cognitive control, loss of behavioral control

Ultimate issue/opinion testimony

-experts still permitted to testify about defendant's mental state, wouldn't be permitted to state their opinion explicitly about defendant being sane at time of crime. -sanity at time of crime left to juries

Fulero & Finkel (1991)

-found that even when experts avoided offering a conclusion about defendants insanity, mock jurors mistakenly remembered that conclusion had been offered.

Wheatman & Shaffer (2001)

-found that giving jurors info about what happens to defendant found NGRI leads to more NGRI verdicts than when info not provided)

Advantages of R-Cras

-guides and organizes clinical judgements about whether defendant is criminally responsible for crime -forces eval to make judgments explicit and look at several aspects of behavior before making decision -high reliability in scoring, highly consistent with eventual court decisions (95% with sanity, 70% insanity decisions)

Defendants found Not Guilty by Reason of insanity

-immediately civilly committed and end up spending, on average, an equal or greater amount of time in a secure mental health hospital.

Policeman at the Elbow test

-impulse had to be so overwhelming that criminal would have committed act even if police officer stood beside criminal at time of crime

Finkel (1995)

-juries answer series of questions about behavior, state of mind, and culpability -NGRI verdict only possible if defendant judged to have total disability of the mind and isn't culpable for creating that disability

Test development in insanity limited

-lack of gold standard for determining insanity -judges also frequently agree with expert opinion on insanity, belittling the meaning of them being in agreement.

Rita Simon

-one of the first researchers to investigate how jurors interpret definitions of insanity -used same case, had 10 juries deliberate using M'Naghten instructions, 10 using Durham instructions -Found: Two instructions produced no significant difference in verdicts. Believed jurors took formal language presented in insanity instructions and translated it to concepts and meanings of their own ideas of insanity and its effects

Retribution perspective on punishment

-punishment for a crime should be proportionate to the harm committed. -if they didn't exercise free will and understand what they are doing, they shouldn't be punished.

Formulations to Distinguish between Preparation ans Substantial Step

1) The last act test, which asks the acts whether the conduct completed is dangerously close to completing the crime itself 2) The notion that the more serious the threatened harm, the more justified the court would be in examining acts further back in the series of acts that would lead to crime completion 3) The idea that the clearer the intent to commit the offense is, the less proximate the acts need to be to the completion of the crime to constitute the crime of attempt

Sexual battery

1.Any person who touches an intimate part of another while that person is restrained 2. touching is against the victims will 3. touching is for purposes of sexual gratification *Wobbler

10. State X defines felony murder as a "homicide in the commission of arson, rape, robbery, burglary, or kidnapping," and defines burglary as "the breaking and entering of a building with the intent to commit a crime therein." Vic ran an automobile junk business and lived in a small apartment over the shop where he conducted his business and where the most valuable parts were kept. An assortment of fenders and bumpers was in the yard near the shop. The yard was surrounded by a high fence. Dan needed a bumper and decided to climb over the fence into Vic's junkyard to steal it. He carried a gun. After he climbed the fence, Vic's dog started barking and awakened Vic. When Vic called out, Dan fired a warning shot and started to run for the fence. The shot startled Vic, who fell out of the second story window and died. If Dan is tried for the felony murder of Vic, his best defense is: (A) Vic became the aggressor and he had the right to use reasonable force to defend himself. (B) He withdrew from the felony. (C) His acts were not the proximate cause of Vic's death. (D) He was not in the commission of a felony required by the statute.

10. (D) (D) is the best answer. In felony-murder Multistate questions, the issue is most often whether any underlying felony has occurred. Of the underlying felonies recognized in State X, the only possibility given in the facts is burglary, burglary being defined as breaking and entering a "building." Dan never entered a building. It is possible that a court would consider Vic's fenced yard to be an integral part of his combined shop residence (under the common law, this is known as the curtilage), but then again the court might not, so (D) is Dan's best argument because larceny is not one of the predicate felonies for a felony murder. This question asks for the "best defense" to the charge of felony murder under the statute given in the question. The best approach is to eliminate the wrong answers. (A) is incorrect because Vic never became an aggressor or even ventured to use force against Dan, who therefore had no imaginable right of self-defense. (B) is incorrect because, for purposes of the felony murder rule, a death occurring during the escape from the scene of the crime is considered to take place in the course of the felony. Dan was trying to escape when he fired his gun, which shot startled Vic and caused him to fall. These acts are clearly within the course of the felony. (C) is incorrect because Dan's firing of the shot is the stated cause-in-fact of Vic's death, and there is no intervening cause. Under the felony murder rule, there is strict liability for a death caused during the commission or escape from a felony; the killing need not have been intentional.

2. Able and Baker planned to rob a bank. Able entered the bank while Baker waited in the car as a lookout. Baker noticed a police car arrive and honked his horn once, this being the prearranged signal. He then drove away. Able heard the signal but proceeded to present a note to the teller demanding money. Able was killed, as he tried to leave the bank, by a bank guard. Baker is tried for felony murder. The result is most likely: (A) Guilty because he was a conspirator. (B) Not guilty because he withdrew from the conspiracy. (C) Not guilty because he was not present at the time. (D) Not guilty because the killing was justifiable.

12. (D) For purposes of the Multistate exam, apply the redline rule (Commonwealth v. Redline, 391 Pa. 486, 137 A.2d 472) with respect to the murder of a co-felon by a police officer. Thus, where a police officer killed the defendant's co-felon as they were fleeing the scene of a robbery, the felony murder rule is inapplicable and the defendant is not guilty. The killing is considered justifiable homicide. Thus, (D) is correct. (A) is incorrect; having agreed with Able to rob the bank, Baker is a conspirator. (B) is incorrect because there is no indication that Baker withdrew from the conspiracy by notifying the other conspirator, Able, that he was withdrawing. The prearranged signal was a warning of trouble, not a communication of withdrawal. Baker therefore remains criminally responsible for any crimes committed within the scope of the conspiracy. (C) is incorrect because the conspiracy was complete when the criminal agreement was formed and a substantial step was taken furtherance of the crime; it is not necessary for a co- conspirator to be present on the scene of the target crime to be guilty of conspiracy.

13. A state statute divides murder into degrees. First degree murder is defined as murder with premeditation and deliberation or a homicide in the commission of arson, rape, robbery, burglary or kidnapping. Second degree murder is all other murder at common law. In which of the following situations is Defendant most likely to be guilty of first degree murder? (A) Immediately after being insulted by Robert, Defendant takes a knife and stabs and kills Robert. (B) Angered over having been struck by Sam, Defendant buys rat poison and puts it in Sam's coffee. Sam drinks the coffee and dies as a result. (C) Intending to injure Fred, Defendant lies in wait and, as Fred comes by, Defendant strikes him with a broom handle. As a result of the blow, Fred dies. (D) Defendant, highly intoxicated, discovers a revolver on a table. He picks it up, points it at Alice, and pulls the trigger. The gun discharges, and Alice is killed.

13. (B) Choice (B) is correct. If defendant had killed Sam immediately after Sam struck him, the crime would probably be voluntary manslaughter because there would be adequate provocation, and the killing would be in the sudden heat of passion. Here, however, defendant went out and bought rat poison, and then put it in Sam's coffee. The fatal act of putting the poison in the coffee took place long after the heat of passion had cooled, making the killing deliberate and premeditated. Choice (A) is murder because the verbal insult is not sufficient provocation to reduce the crime to manslaughter. However, there is no interval between the insult (before which defendant had no intent to kill) and the actual killing. Therefore, even though there is malice, there is no premeditation and deliberation, and choice (A) is incorrect. Choice (C) is incorrect because an intention to kill is required for deliberate premeditated first degree murder. The intention to inflict serious bodily harm is adequate to provide malice for second, not first, degree murder. Choice (D) is incorrect because voluntary intoxication, while not a defense to murder, can prevent a defendant from killing with premeditation and deliberation. Arguably, this could be a highly reckless killing which would be depraved heart murder, a second degree murder under the facts given.

Questions 15-17 are based upon the four case summaries below. For each question, select the case that would be most applicable as a precedent. (A) People v. Smith Officer Smith had grounds to arrest Green during an armed robbery. Green was fleeing in his vehicle. Smith, on foot, fired at the tires of the speeding vehicle in an attempt to capture Green. The tires blew out when hit by Smith's gunfire and the car crashed into an embankment, resulting in Green's death. Smith is charged with manslaughter. Held not guilty. (B) State v. Watson Officer Watson observed a car go through a red light. He gave chase, putting on his flashing lights. When the car failed to stop, he forced the car off of the road as it was going over a bridge. The car went into the water and the driver drowned. Held guilty of manslaughter. (C) Commonwealth v. O'Neil Officer O'Neil observed Pic steal $10 from a cigarette machine in the subway and immediately restrained him physically. Pic succeeded in breaking loose, but in the course of the struggle fell onto the live third rail and was electrocuted. Held not guilty of manslaughter. (D) People v. Mack Officer Mack attempted to arrest Foxy Fanny at a burlesque house for indecent exposure. As he approached her, she grabbed a knife from a stagehand and lunged at Mack. He pulled his gun and shot her in the leg. She later died of complications. Held not guilty of criminal homicide. 15. At a rock festival, X and Y engage in a fist fight with each other and no one else. D, a police officer, breaks up the fight by hitting Y over the head as hard as he can with a billy club. Y gets a fractured skull and dies. D is charged with manslaughter.

15. (B) The correct answer is (B). The fist fight involving X and Y and no one else would be construed as a misdemeanor such as disturbing the peace or assault and battery. The police officer who broke up the fight used deadly force when he hit Y over the head as hard as possible with a billy club. The use of deadly force against a misdemeanor falls within the purview of answer (B) as a precedent. (D) holds that deadly force is justifiable as a matter of self-defense when a legal arrest is resisted by deadly force.

461 2- default when PC doesn't say how much time (look at 18 PC for the amount of time when it talks bout state prison)

16 mos low term 2 year mid term 3 year high term When it says crime punishable in county jail not exceedinf 6 mos Misdemeanor up to a year in jail (this is offense definition) but the punishable is up to 6 mos When it doesn't state a time in jail.

(A) People v. Smith Officer Smith had grounds to arrest Green during an armed robbery. Green was fleeing in his vehicle. Smith, on foot, fired at the tires of the speeding vehicle in an attempt to capture Green. The tires blew out when hit by Smith's gunfire and the car crashed into an embankment, resulting in Green's death. Smith is charged with manslaughter. Held not guilty. (B) State v. Watson Officer Watson observed a car go through a red light. He gave chase, putting on his flashing lights. When the car failed to stop, he forced the car off of the road as it was going over a bridge. The car went into the water and the driver drowned. Held guilty of manslaughter. (C) Commonwealth v. O'Neil Officer O'Neil observed Pic steal $10 from a cigarette machine in the subway and immediately restrained him physically. Pic succeeded in breaking loose, but in the course of the struggle fell onto the live third rail and was electrocuted. Held not guilty of manslaughter. (D) People v. Mack Officer Mack attempted to arrest Foxy Fanny at a burlesque house for indecent exposure. As he approached her, she grabbed a knife from a stagehand and lunged at Mack. He pulled his gun and shot her in the leg. She later died of complications. Held not guilty of criminal homicide. 16. D, a police officer on highway patrol, observes V's car speeding with the lights out. D puts on his siren and gives chase at high speed. V's car fails to negotiate a turn, goes off the road, and V is killed. D is charged with manslaughter.

16. (C) The correct answer is (C). D's giving chase to V's speeding car is not the application of deadly force. This fact pattern most closely resembles (C): the justified application of non- deadly force (if even that) which unforeseeably results in death.

Lawful possession of a concealable firearm

16530(a) PC Any device designed to be used as a weapon from which a projectile by force of an explosion or other form of combustion that has a barrel less than 16 inches in length Has nothing to do with if the weapon is actually concealed or not. If the barrel can be changed to shorter than 16 inches maybe considered concealable

(A) People v. Smith Officer Smith had grounds to arrest Green during an armed robbery. Green was fleeing in his vehicle. Smith, on foot, fired at the tires of the speeding vehicle in an attempt to capture Green. The tires blew out when hit by Smith's gunfire and the car crashed into an embankment, resulting in Green's death. Smith is charged with manslaughter. Held not guilty. (B) State v. Watson Officer Watson observed a car go through a red light. He gave chase, putting on his flashing lights. When the car failed to stop, he forced the car off of the road as it was going over a bridge. The car went into the water and the driver drowned. Held guilty of manslaughter. (C) Commonwealth v. O'Neil Officer O'Neil observed Pic steal $10 from a cigarette machine in the subway and immediately restrained him physically. Pic succeeded in breaking loose, but in the course of the struggle fell onto the live third rail and was electrocuted. Held not guilty of manslaughter. (D) People v. Mack Officer Mack attempted to arrest Foxy Fanny at a burlesque house for indecent exposure. As he approached her, she grabbed a knife from a stagehand and lunged at Mack. He pulled his gun and shot her in the leg. She later died of complications. Held not guilty of criminal homicide. 17. X is stealing apples from a tree in an apple orchard. D observes him and calls for him to come to him. X runs away. D shoots at him, wounding him in the leg. D is charged with assault and battery with a deadly weapon.

17. (B) The correct precedent to apply is (B). D would have been justified in using reasonable force to protect property from trespass or theft. D did not have the right to use deadly force to apprehend a non-violent suspect to a misdemeanor. The misdemeanor of picking apples hardly justified the use of deadly force.

An unauthorized weapon in a prohibited location

171 (b) PC Felony any person who brings or possesses a specified weapon within any state or local public building to any meeting required to be open to the public

18. In which of the following cases is the defendant guilty of murder? (A) D and V have an argument in a bar. V throws whiskey in D's face. D wipes his face, pulls a knife, and fatally stabs V. (B) V slaps D's son in D's presence. D becomes enraged and kills V. (C) D observes V stealing a hubcap from his car. D immediately shoots V. (D) V, a police officer, insults a juvenile, age 16, by calling him a "dirty little bastard" without justification. The juvenile stabs and kills the officer.

18. (D) (D) is correct. The verbal insult by the police officer is not sufficient provocation to reduce murder to manslaughter. Words alone are generally never a sufficient provocation. If, however, the words are informational (conveying information of a fact which constitutes reasonable provocation, e.g., a wife telling her husband she's having an affair); this could constitute provocation to the husband of the same sort as if he had made an ocular observation of his wife's adultery. A juvenile, age 16, is capable of possessing the mens rea necessary for murder. This question involves an intentional killing in each fact pattern. Therefore, the distinction between murder and other homicide crimes depends on the provocation to reduce the crime to manslaughter. (A) is incorrect because the physical violence of throwing the drink in D's face in the course of an argument is sufficient provocation to reduce the killing to manslaughter because it was within the heat of passion. (B) is incorrect because the physical violence inflicted on D's son is likewise sufficient provocation to reduce the crime to manslaughter and D's response was immediate. (C) is incorrect. D did not have the right to use deadly force to protect his property. However, he did have the right to use nondeadly force. His use of excessive force constitutes manslaughter, not murder.

19. John was fired from his job. Too proud to apply for unemployment benefits, he used his savings to feed his family. When one of his children became ill, he did not seek medical attention for the child at a state clinic because he did not want to accept what he regarded as charity. Eventually, weakened by malnutrition, the child died as a result of the illness. John has committed: (A) Murder. (B) Involuntary manslaughter. (C) Voluntary manslaughter. (D) No form of criminal homicide.

19. (B) (B) is correct. This is a classic case of involuntary manslaughter. John's conduct was highly reckless. There is an affirmative duty imposed upon a parent to feed his children if he is reasonably capable of so doing. John possessed this capability though the seeking of public assistance. His failure to do so was either the willful, wanton conduct or the criminally negligent conduct necessary to support the mens rea for involuntary manslaughter. This conduct was the proximate cause of the child's death. (A) is incorrect because, even though there is a duty to act on John's part and his failure to perform that duty caused his child's death, he did not possess the necessary mens rea for murder. He did not intend to kill or inflict serious bodily harm on his child. His actions did not so depart from proper conduct for malice to be implied under an abandoned or malignant heart theory. There is no felony-murder theory. (C) is incorrect. The malice necessary for murder is not present, so there is no need to look for the heat of passion necessary to reduce the killing to voluntary manslaughter. Voluntary manslaughter is always an intentional killing and this killing was not. (D) is incorrect because the facts are sufficient to constitute the crime of involuntary manslaughter.

2. Bonnie is a registered nurse at the Parkland Hospital. Victor is her ex-lover who left her for a younger woman. Clyde is a local bookie. Victor has defrauded him of $100,000. One day, as Victor was walking in front of Clyde's car. Clyde deliberately ran over him. Victor survived the collision but was mortally injured and was taken to the Parkland Hospital barely conscious, where he was first seen by Bonnie, the emergency room nurse. Bonnie gave him an injection of curare, a deadly poison, and Victor died immediately. As a result of these actions: (A) Bonnie is guilty of murder and Clyde is guilty of attempted murder. (B) Clyde is guilty of murder and Bonnie is guilty of attempted murder. (C) Both Bonnie and Clyde are guilty of murder. (D) Both Bonnie and Clyde are guilty of attempted murder.

2. (A) Clyde assaulted Victor with the intention of killing him, but his actions did not result in Victor's death because Victor was killed by a totally independent criminal cause while at the hospital. Clyde is not guilty of murder, but he is guilty of attempted murder. Therefore, (C) is incorrect and (A) is correct. Bonnie's action is the proximate cause of death; that is, but for the injection Bonnie gave Victor, he would not have died. Bonnie's act was intentional. Therefore, Bonnie is guilty of murder, making (B) and (D) incorrect.

20. D was the official state executioner at Song prison. Furnham was sentenced to death by a competent tribunal, all appeals were exhausted, and D was to perform the execution. D hated the prison warden and arranged for him to be substituted for Furnham in the gas chamber. Thinking that the warden was in the gas chamber, D released cyanide pellets which killed the occupant of the gas chamber. In fact, prison officials had discovered D's attempt at substitution, and had removed the warden and replaced him with Furnham in the gas chamber. If D is tried for murder, the most likely result will be: (A) Guilty because he committed homicide with malice aforethought. (B) Not guilty because the killing was justified. (C) Not guilty because the killing was excused. (D) Not guilty because D has the defense of mistake of fact.

20. (B) The correct answer is (B). D is on trial for murder of Furnham, not the attempted murder of the warden. As the official state executioner, he did have the legal right to kill Furnham under the defense of public authority and so that killing was justified. (A) is incorrect because, if there is justification, by definition there is no malice. (C) is incorrect because public authority is a justification, not an excuse. The state executioner is called upon by law to kill the person he kills because it is the community's belief that capital punishment is justified for the crime committed. (D) is incorrect because mistake of fact is irrelevant to murder; if there is intent to kill and a different victim is killed due to a mistake as to identity, the crime is still murder under the doctrine of transferred intent.

21. Victim was the leader of a religious cult and Defendant was a member of that cult. For 30 consecutive days, Victim preached a sermon in which he proclaimed that he had the power to call lightning down from the heavens to strike down followers whom he considered to be sinners. Defendant believed he had this power. One day, Victim saw Defendant's 6-year-old daughter take a tomato from the vine in the garden. Victim summoned Defendant and Defendant's daughter before him and proclaimed that he was about to strike Daughter with lightning for her misdeed. Defendant took a knife and stabbed Victim, killing him. If charged with a homicide crime, Defendant's defense of defense of others will: (A) Not be sustained, because his daughter was not in fact threatened. (B) Not be sustained, because Defendant's belief that his daughter was in danger was not reasonable. (C) Be sustained, because he actually believed his daughter was in danger. (D) Be sustained, because reflection is not required when there is an imminent threat of death to others.

21. (B) The correct answer is (B). The use of deadly force in defense of another is justified only if one has an honest and reasonable belief that the other person is in imminent danger of death or great bodily harm. Defendant's cult membership tends to show the sincerity of his belief that his daughter was in danger but does nothing to show that it was reasonable. Defendant's belief that Victim could call lightning down on his daughter is not objectively reasonable. (Note that First Amendment religious freedoms, even if relevant, are not involved in any of the answers). (A) is incorrect. It is reasonable belief in a threat, even if unfounded, which establishes the defense. An unreasonable belief in a threat is no justification even if a threat is actually made. (C) is incorrect because an actual belief in the danger is not enough; the belief must be reasonable. (D) is incorrect because, though literally true, it is irrelevant. "Reflection" is not an element of defense-of-others as it is of first-degree murder. It is not whether the defendant reflected or not but whether he held a reasonable belief in imminent danger which determines whether the homicide is justified.

13. NEGLIGENCE AND THE GRADING OF HOMICIDE: MPC MANSLAUGHTER AND NEGLIGENT HOMICIDE

210.3 Manslaughter (1) Criminal homicide constitutes manslaughter when (a) it is committed recklessly. Notes: - Subjective standard - Conscious disregard of a high probability of a risk of death - Contrast against the common law, which held that recklessness was 210.4 Negligent Homicide (1) Criminal homicide constitutes negligent homicide when it is committed negligently: (2) Negligent homicide is a felony of the third degree. Notes: A. Objective standard B. Closer to the common law "involuntary manslaughter rule"

22. X holds a gun on D and orders him to light fire to a boat owned by V, which is sitting next to the dock. Because he fears that X will shoot him if he refuses to obey, D complies. V runs down the dock and onto the boat in an attempt to recover some valuable personal property on the boat. The boat capsizes because of the fire, and V drowns. The definition of arson has been expanded by statute to include boats. D is charged with the murder of V. He should be found: (A) Not guilty, because duress is a defense to arson. (B) Not guilty, because V voluntarily put himself in danger. (C) Guilty, because duress is not a defense to murder. (D) Guilty, because he could have taken the gun away from X and avoided burning the boat.

22. (A) (A) correctly states that duress (coercion) is a defense to arson. D will be charged with felony murder, and if the arson conviction cannot be sustained the felony murder charge must fall. Duress may also be a defense to the other predicate felonies such as burglary and robbery. (B) is incorrect because contributory negligence and assumption of the risk are tort doctrines irrelevant to homicide law. (C) is incorrect. In general, duress is not a defense to murder. But it is a defense to the underlying felonies which are essential elements of felony murder. Here, duress is a defense to the underlying felony of arson, and therefore, there is no predicate for the felony murder. (D) is incorrect because the facts do not show that D could have easily avoided complying with X'S demand by disarming him. An unarmed person does not usually have the opportunity to disarm a gunman and is under no duty to make the attempt.

Questions 23 and 24 are based on the following facts: Able attended a flea market. Able noticed a vase and asked Dave how much it cost. Dave replied, "One hundred dollars." Able paid him the money and took the vase. Dave was not the owner of the vase and was merely a bystander. Dave absconds with the $100. 23. What crime has Dave committed with respect to the $100? (A) Embezzlement. (B) Larceny by trick. (C) Larceny. (D) Obtaining property by false pretenses.

23. (D) The correct answer is (D). Under the circumstances, by replying that the price of the vase was $100, Dave implied that he was the owner or was authorized to sell it, which he was not. When Able handed Dave the $100, Dave obtained title to the currency because Able unconditionally handed the money to Dave, who could then spend the money as he liked. Able relied on the false statement or false implication that Dave would sell him the vase and at that moment passed both title and possession to the money to Dave. This was obtaining money by false pretenses. (A), embezzlement, is incorrect because Dave did not have lawful possession of the money he used fraud to obtain. (B), larceny by trick, is incorrect because Dave obtained title - - not merely possession -- by fraud ("trick"). (C), larceny, is incorrect because the money was voluntarily given to Dave and, accordingly, there was no trespassory taking.

Able attended a flea market. Able noticed a vase and asked Dave how much it cost. Dave replied, "One hundred dollars." Able paid him the money and took the vase. Dave was not the owner of the vase and was merely a bystander. Dave absconds with the $100. 24. What crime has Dave committed with respect to the vase? (A) Embezzlement. (B) Larceny. (C) Obtaining property by false pretenses. (D) No crime.

24. (B) The correct answer is (B). All the elements of larceny were present. Dave took and carried away the personal property of another intending to deprive the owner of it permanently. His use of an innocent third party as a means does not absolve him of criminal responsibility. Thus, the crime is larceny. (A) is incorrect because Dave never had rightful possession of property which he subsequently converted. (C) is incorrect because it was the money, not the vase, which Dave obtained by false pretenses. (D) is incorrect because Dave is guilty of the crime of larceny.

25. In which of the following fact situations would the defendant be guilty of the crime charged? (A) Popeye takes V's outboard motorboat from the dock on a fishing trip and returns it to the dock three hours later. Popeye takes the fish he caught with him. Popeye is charged with common law larceny. (B) Popeye obtains permission to use V's boat, promising to return it as soon as the fishing trip is concluded, a promise he did not then intend to keep. The boat springs a small leak while Popeye is fishing and he decides to return the boat as promised, and does in fact return it. Popeye is charged with common law larceny by trick. (C) Popeye rents a boat for cash from an agency, giving false information about his name, address, and occupation on the rental application form. When he returns the boat, he is charged with obtaining property by false pretenses. (D) Popeye borrows his friend's boat to go on a fishing trip, promising his friend he will return it by sunset. The fishing is excellent and Popeye stays out all night. When he returns the boat the next morning, he is charged with embezzlement.

25. (B) The correct answer, (B), is a classic case of larceny by trick. Larceny by trick is where possession is obtained through fraud; that is, one gets possession with the intent at the time to permanently deprive the owner of that property. Popeye obtained possession (not title) by a fraudulent representation. The crime was complete when he gained possession; the later return of the boat does not affect Popeye's culpability. (A) is incorrect because the most reasonable inference to be drawn is that Popeye did not act with the intent to deprive V of his property permanently. (As for the fish, they were presumably unowned prior to Popeye's appropriation of them, so his title and possession are good.) (C) is incorrect for two reasons. The false representations as to name and address are not material misrepresentations. Also, as in (B), there is no intent to deprive the owner of property permanently. (D) is incorrect because, even though Popeye did not return the boat as soon as he had promised to, he never formed an intent to convert it, which is an essential element of embezzlement as it is of larceny.

Unlawful possession of a concealable firearm without a license

25400 PC Misdemeanor unless certain conditions are met Any person who; carries concealed firearm that is capable of being concealed (loaded or unloaded) upon one's person within any vehicle under that persons control or direction without a license or permit

26. As V is walking down the street, D comes up from behind and says, "I have a gun pointed at your heart. Hand over your money." V hands over his money to D. In fact, D does not have a gun. The most serious crime with which D could properly be charged is: (A) Armed robbery. (B) Attempted armed robbery. (C) Robbery. (D) Larceny.

26. (C) Since V obtained the money by intimidation and a threat of force, he is guilty of robbery, as (C) states. (A) is incorrect because since V was not in fact armed, he cannot be guilty of armed robbery. (B) is incorrect because V did not intend (and therefore did not attempt) to commit armed robbery. And (D) is incorrect because larceny, of which V must be guilty since he is guilty of robbery, is a lesser included offense.

Bringing a firearm into a motor vehicle

26100(a) PC Misdemeanor any driver or owner of any motor vehicle who permits any person to carry into, or bring into the vehicle a firearm in violation of 25850(a) PC

27. Motorist took an automobile, which was worth about - $200, to Repairman to have the brakes realigned and the muffler replaced. The prices to be charged for each of these items was posted on the wall of Repairman's garage. When Motorist returned for his car after it was repaired, he was presented with a bill for $253, in accordance with the posted prices. Considering the bill excessive, Motorist left the garage. He returned after the garage had closed, and drove his car out of the lot owned by Repairman, adjoining the garage. If charged with the crime of larceny, motorist will be: (A) Guilty, if the stealing of services constitutes larceny. (B) Guilty, if he took the car without the consent of Repairman. (C) Not guilty, if the charge for the repairs was excessive. (D) Not guilty because the car was the property of Motorist.

27. (B) Although Motorist has title to his car, he lacks rightful possession because the repairman has a possessory lien on the car. Larceny is a crime against possession, even where, as here, it is committed by the property's owner. (B) is therefore correct. (A) is incorrect because the theft, if any, is of personal property (the car), not services. (C) is incorrect because the Repairman was rightfully in possession and the Motorist's trespassory taking was larceny. (D) is incorrect because it is legally possible, in a situation like this one, for the record owner to be guilty of larceny of property to which he has title; larceny is a crime against possession.

Questions 29 and 30 are based on the following facts: M was the manager of a local bank. One morning while he was in his office at the bank with A, the assistant, manager, D broke into M's office and pointed a gun at M and A. D told M that he was going to stay in the office with A and would kill him if M did not return immediately with $100,000. M went to the bank vault as soon as it opened took $100,000 in cash and gave it to D. D then escaped with the money but was later apprehended. 30. If D were charged with the crime of kidnapping A, he would be: (A) Guilty because A was imprisoned against his will. (B) Guilty because D used unlawful force to Imprison A. (C) Not guilty because A was neither secretly imprisoned nor carried away. (D) Not guilty because D did not demand ransom.

30. (C) The correct answer is (C). Under the Model Penal Code, the victim must be either (1) removed from his residence or place of business, (2) removed a substantial distance from where he was found, or (3) confined for a substantial period in a place of isolation. This third provision prevents D from being guilty of kidnapping A, who was confined in the office while M went to get the money. (A) is incorrect. This would be the crime of false imprisonment, which is committed when a defendant so restrains another person as to interfere substantially with his liberty. (B) is incorrect. Again, this would be false imprisonment, which may be accomplished by a show of force to which the victim submits. (D) is not correct. The demand for ransom is not an element of kidnapping.

28. Johnson took a diamond ring to a pawnshop and borrowed $20 on it. It was agreed that the loan was to be repaid within 60 days, and if it was not, the - . pawnshop owner, Defendant, could sell the ring. A week before expiration of the 60 days, Defendant had an opportunity to sell the ring to a customer for $125. He did so, thinking it unlikely that Johnson would repay the loan, and if he did, Defendant would be able to handle him somehow, even by paying him for the ring if necessary. Two days later, Johnson came in with the money to reclaim his ring. Defendant told Johnson the truth that he sold the ring because he thought Johnson would not reclaim it and offered to give Johnson $125. Johnson demanded his ring. Defendant said, "Look, buddy, that's what I got for it and it's more than it's worth." Johnson reluctantly took the money. Larceny, embezzlement, and false pretenses are separate crimes in the jurisdiction. Defendant most appropriately could be found guilty of: (A) Larceny. (B) Embezzlement. (C) Obtaining property by false pretenses. (D) None of the above.

28. (B) The correct answer is (B). For embezzlement, the property must already be in the embezzler's lawful possession when he misappropriates it. Embezzlement usually requires that the property be entrusted to the embezzler, who then fraudulently converts that which is in his possession. The intent to restore the rightful owner with money is no defense. The defendant's subsequent good faith, his payment of money to the rightful owner, does not change the crime. (A) is not correct. For larceny, there must be a trespass in the taking; the thief must take the property out of the victim's possession, which means he cannot already have it in his possession. (C) is not correct. Obtaining property by false pretenses requires the defendant, by his lies, to obtain title to the victim's property. Defendant had possession but not title. (D) is not correct.

Questions 29 and 30 are based on the following facts: M was the manager of a local bank. One morning while he was in his office at the bank with A, the assistant, manager, D broke into M's office and pointed a gun at M and A. D told M that he was going to stay in the office with A and would kill him if M did not return immediately with $100,000. M went to the bank vault as soon as it opened took $100,000 in cash and gave it to D. D then escaped with the money but was later apprehended. 29. If M were charged with embezzlement of $100,000 of bank funds, he would be: (A) Guilty because there was no substantial threat to use force against M or a member of his household. (B) Not guilty because he did not have mens rea. (C) Guilty because the defense of duress is unavailable to such a serious crime as embezzlement. (D) Not guilty because he was coerced to take the money by the substantial threat to use unlawful force against another person.

29. (D) (D) is the correct answer. The defense of duress or coercion arises when the defendant engages in conduct which would otherwise be criminal, but his guilt is excused because he was coerced to do so by the use of or threatened use of physical force upon him or upon a third person. Such force used or threatened is sufficiently intense if a person of reasonable firmness in the same situation would be unable to resist. (A) is incorrect. The substantial threat of force to another person is all that is required for duress; there is no requirement that the person threatened be a member of his household. (B) is incorrect. M had the mens rea to steal because he took the money intending to deprive the Bank of it, but he was under a threat. (C) is incorrect because homicide crimes are the only crimes to which, as a matter of law, duress is never a defense.

Firearm possession by a convicted felon, narcotic addict, or other restricted persons

29800(a)(1) PC Felony Any person who has been convicted of a felony or is addicted to any narcotic drug or two convictions or 29800(a)(2) or offenses enumerated in PC 23515

Possession or a firearm by a person convicted of certain misdemeanors

29805 PC Felony Prohibited from possessing a firearm within 10 years of the conviction any person who has been convicted of specified misdemeanor and with in 10 years of the conviction owns or has possession, custody, or control of any firearm

3. Fearing burglars, Daughter, who lives alone, put a mechanical device in her home which would fire a shotgun at close range at the front door of her house, if the door were forced open at a time when the device was operative. Daughter set the device and left on a two-week vacation. Seeing ten days worth of newspapers piled in front of Daughter's front door, V, a neighbor, was afraid that Daughter had injured herself, and therefore forced the front door open to investigate. The shotgun fired and killed V.

3. (C) The correct answer is (C). The most serious crime in both cases is murder. Case I falls under the rule that a killing by a mechanical device is considered as if committed in person by whomever placed the device. Under the facts, the neighbor V was clearly not a burglar, he was killed without justification, excuse or mitigation and, accordingly, the homicide is murder. Case II, the mercy killing, is an intentional killing -- a shortening of Mother's life, and the victim's consent tom homicide is never a defense. (A), (B), and (D) are not correct because the facts given do not reduce the seriousness of the crimes to manslaughter.

Questions 4 and 5 are based on the following facts: Dante knew that his father was about to change his will and disinherit Dante. He therefore decided to kill him by poison. He asked his friend, Rexall, a pharmacist, to obtain some Zylene, a deadly poison which comes in a plain white pill, and to give it to him without recording the transaction. Because Rexall suspected Dante's motive, she supplied Dante with a small quantity of Dorvo, a muscle relaxant, instead of Zylene. Dorvo is harmless if administered in small quantities, except for the less than one percent of the population who are allergic to the drug. Dante substituted the Dorvo pill given him by Rexall for the aspirin which his father took each night for his arthritis. Dante's father was allergic to the drug and died from a reaction to it. Rexall was distraught and confessed the entire affair to the police, explaining that she had failed to report Dante's conduct to the authorities because she feared that it would end their friendship if she did. 4. Rexall is an accomplice to: (A) murder. (B) manslaughter. (C) criminally negligent homicide. (D) no degree of criminal homicide.

4. (D) Rexall did not knowingly assist Dante in his murder plans. In fact, she took steps which she thought would thwart Dante's plan. She is therefore not guilty as an accomplice to any homicide crime, and (D) is correct. (A), (B), and (C) are, accordingly, incorrect.

Drawing, exhibiting, or unlawful use of a deadly weapon (other than firearm)

417(a)(1) Misdemeanor any person who except in self defense in the presence of another person draws or exhibits any deadly weapon in a rude, angry, or threatening manner unlawfully used a deadly weapon in any fight or qurrel

Drawing, exhibiting, or unlawful use of a firearm

417(a)(2) Misdemeanor Any persons who while in the presence of another person, except in self-defense draws or exhibits any firearm (loaded or unloaded) in a rude, angry, or threatening manner unlawfully uses a firearm in any fight or quarrel

Drawing, exhibiting, or unlawful use of a firearm at a day care center

417(b) PC Felony any person who, while in the presence of another, except in self defense draws or exhibits any loaded firearm in a rude, angry, or threatening manner unlawfully uses a LOADED firearm in a rude, angry or threatening manner Unlawfully uses a loaded firearm in a fight or quarrel on the grounds of a day care when that center or facility is open for use

Draw, exhibiting, or unlawful use of a firearm in the presence of a peace officer

417(c) PC Felony any person Loaded or unloaded rude, threatening manner who knows or reasonably should have know, by the officers appearance, action, or ID that he or she is a peace officer engaged in his or her performance of their duties

Using a laser scope or laser pointer with intent to cause apprehension or fear is covered under...

417.25 PC Misdemeanor aims or points a laser scope or laser pointer except in self defense in a threatening manner with intent to cause a reasonable person apprehension or fear

Draw or exhibiting a firearm in a motor vehicle

417.3 PC Felony any person except in self defense to draw or exhibit any firearm (loaded or Unloaded) in a threatening manner such as to cause a reasonable person apprehension or fear of bodily harm in the presence of another person who is the occupant of a motor vehicle proceeding on a public street of highway

An imitation firearm

417.4 PC Misdemeanor draws or exhibits an imitation firearm in a threatening manner against another person in such a way as to cause a reasonable person apprehension or fear of bodily harm

Dante knew that his father was about to change his will and disinherit Dante. He therefore decided to kill him by poison. He asked his friend, Rexall, a pharmacist, to obtain some Zylene, a deadly poison which comes in a plain white pill, and to give it to him without recording the transaction. Because Rexall suspected Dante's motive, she supplied Dante with a small quantity of Dorvo, a muscle relaxant, instead of Zylene. Dorvo is harmless if administered in small quantities, except for the less than one percent of the population who are allergic to the drug. Dante substituted the Dorvo pill given him by Rexall for the aspirin which his father took each night for his arthritis. Dante's father was allergic to the drug and died from a reaction to it. Rexall was distraught and confessed the entire affair to the police, explaining that she had failed to report Dante's conduct to the authorities because she feared that it would end their friendship if she did. 5. In a common law jurisdiction, Dante is guilty of (A) murder only. (B) murder and conspiracy. (C) attempted murder only. (D) attempted murder and conspiracy.

5. (A) The correct answer is (A). Dante intended to kill his father by giving a poison pill to him. He administered a different pill, but it did precisely what he intended, that is, it killed his father. Therefore, this is an intentional killing and common law murder. (B) is incorrect. He is not guilty of conspiracy because there was no agreement by Dante and Rexall with the intent to commit an unlawful act nor an intent to agree to act together to carry out a common criminal purpose. Rexall did not agree with Dante to commit the crime, but rather tried to thwart it. It is not attempted murder because, by his action of substituting the poison pill for the aspirin, he killed his father. (C) and (D) are therefore incorrect.

6. Vic suffers from a rare disease. If his skin is punctured, the wound is very likely to become infected, and the infection is likely to spread to his lungs. Since the disease destroys his ability to fight infection, the lung infection can be fatal. Defendant dislikes Vic, but does not know of his disease. One day, Defendant and Vic had a verbal argument. Defendant pulled a knife which caused a cut on Vic's leg, the result that Defendant intended. The cut became infected, the infection spread to Vic's lungs, and Vic died three days later. Defendant shoots three bullets toward Victim, intending that the bullets miss and only scare him. All three bullets initially miss Victim, but one bullet hits the wall behind him and ricochets into Victim, killing him. Defendant is guilty of: (A) Murder in Case I only. (B) Murder in Case I1 only. (C) Murder in both cases. (D) Murder in neither case.

6. (C) The correct answer is (C) since both cases involve murder. In Case I, Defendant does not intend Vic's death, but does intend to inflict great bodily harm by cutting his leg with a knife, which is intent enough to establish malice. Although death from such a cause was unforeseeable, one who intentionally inflicts great bodily harm takes her victim as she finds him. Thus Case I is murder. In Case II, Defendant fires three bullets in an enclosed space to scare Vic, unintentionally killing him with a ricocheting bullet. The intent to create a very high risk of death or great bodily harm, for no good reason, is "depraved heart" murder. (A), (B), and (D) are therefore incorrect.

Possession of a firearm in a school safety zone or on school grounds in or within 1000 feet of the grounds of a public or private school providing instruction in K-12

626.9 Felony any person who possesses a firearm (loaded or unloaded) in a place that the person knows or reasonably should have known is a school safety zone without written permission of the school superintendent or designee or equivalent school authority

7. V is having a love affair with Farrah, and lies to her about his marital status. When she finds that V is lying Farrah shoots V. Wife finds V in his wounded condition and asks him what happened. He says a woman shot him, but refuses to identify Farrah. Wife walks away. V dies of the wound, but would not have died if medical assistance had been summoned. Who is guilty of a homicide crime? (A) Farrah only. (B) Wife only. (C) Both Wife and Farrah. (D) Either Wife or Farrah, but not both.

7. (C) (C), holding Wife and Farrah guilty, is correct. (A) is of course incorrect because it does not acknowledge Wife's guilt. It is clear Farrah is guilty of a homicide crime, either murder or voluntary manslaughter. Wife is also guilty. It was her mere inaction (refusing to summon medical assistance) which was the cause (the "but for" test) of V's death. Generally the law imposes no duty to aid someone in need. However, the marital relationship imposes a duty of affirmative action to render reasonable help to a spouse in danger. (B) and (D), which fail to assert unconditionally that Farrah is guilty, are incorrect.

Questions 8 and 9 are based on the following facts: Dunbar and Balcom went into a drugstore, where Dunbar reached into the cash register and took out $200. Stone, the owner of the store, came out of the back room, saw what had happened, and told Dunbar to put the money back. Balcom then took a revolver from under his coat and shot and killed Stone. Dunbar claims that Stone owed her $200 and that she went to the drugstore to try to collect the debt. She said that she asked Balcom to come along just in case Stone made trouble, but that she did not plan on using any force and did not know that Balcom was armed. 8. If Dunbar is prosecuted for murder on the basis of felony murder and the jury believes her claim, she should be found: (A) Guilty because her companion, Balcom, committed a homicide in the course of a felony. (B) Guilty because her taking Balcom with her to the store created the risk of death that occurred during the commission of a felony. (C) Not guilty because she did not know that Balcom was armed and thus did not have the required mental state for felony murder. (D) Not guilty because she believed she was entitled to the money and thus did not intend to steal.

8. (D) If the jury believes Dunbar, she is not guilty of the underlying crime of larceny because she had no intent to steal. Rather, she was trying to collect money owed to her, in a nonviolent manner. Since she is not guilty of the underlying felony of larceny, which is an essential element of robbery, she is not guilty of robbery. Because she was not engaged in the commission or attempted commission of a felony, she cannot be guilty of felony murder. (D) is therefore correct. (C) is incorrect because felony murder has nothing to do with the armed state of a companion, but rather with whether the criminal participants are engaged in the commission or attempted commission of a felony. If Dunbar had attempted to rob the drugstore, she would be guilty of felony murder, even if she did not know that her companion had a gun. (A) and (B) are incorrect because they wrongly assume that Dunbar was engaged in the commission of a felony. Note: If you came down on the guilty side of this problem, you would have an almost impossible time distinguishing between choices (A) and (B), because each is a basis for felony murder if in fact there was an underlying felony. This question illustrates the test strategy that if it is impossible to distinguish between two choices on one side of an issue, the correct answer will probably be on the other side.

Dunbar and Balcom went into a drugstore, where Dunbar reached into the cash register and took out $200. Stone, the owner of the store, came out of the back room, saw what had happened, and told Dunbar to put the money back. Balcom then took a revolver from under his coat and shot and killed Stone. Dunbar claims that Stone owed her $200 and that she went to the drugstore to try to collect the debt. She said that she asked Balcom to come along just in case Stone made trouble, but that she did not plan on using any force and did not know that Balcom was armed. 9. If Dunbar is prosecuted for murder on the basis of being an accessory to Balcom in committing a murder and the jury believes her claim, she should be found: (A) Guilty because in firing the shot Balcom was trying to help her. (B) Guilty because she and Balcom were acting in concert in a dangerous undertaking. (C) Not guilty because she had no idea that Balcom was armed and she did not plan to use force. (D) Not guilty because she was exercising self-help and did not intend to steal.

9. (C) This question focuses on the amount of activity needed for one to be considered an accessory to a crime. It is necessary to aid in the perpetrator's criminal enterprise. If Dunbar's story is believed, she did not intend to use force and did not knowingly go to the store with a person who was armed. Therefore, she did not aid in Balcom's criminal enterprise, nor did she encourage it. (C) is therefore correct. (D) is not correct because this is a defense to larceny or robbery that negates an intent to steal. It is not a defense to murder. (B) is incorrect because it is not factually true. Dunbar intended to take the money owed her in a quiet, nonviolent manner which was not dangerous. (A) is incorrect because it focuses on the wrong person. It is Dunbar's mental state which will determine her accomplice liability, not the mental state of the perpetrator of the crime.

What is Precedent?

A binding prior court decision. A prior decision is binding on a court only if it raises the same legal issue as the case currently before the court.

juvenile offender

A child who violates the criminal law or who commits a status offense.Also, a person subject to juvenile court proceedings because a statutorily defined event caused by the person was alleged to have occurred while the person was below the statutorily specified age limit of original jurisdiction of a juvenile court. Transfer to adult court requires a showing by the prosecution that the child was able to "appreciate the wrongfulness of [his or her] conduct," that the child had "a guilty knowledge of wrongdoing," or that the child was "competent to know the nature and consequences of his [or her] conduct and to know that it was wrong." The defense of infancy is based on the chronological age of the defendant. It has not been successful when based on claims of mental immaturity. Courts have held that those who have passed the chronological age necessary for criminal responsibility cannot raise the defense of infancy based on psychological determinations of mental immaturity because"if by reason of mental disease or defect a particular individual has not acquired [the ability to observe legal requirements] his remedy is a defense based on idiocy or insanity."

Intoxication

A claim of intoxication is generally not regarded as an effective defense even when the intoxication results from alcoholism, and laws that codify the defense usually set strict limits on the use of the intoxication defense. Intoxication is not a useful defense because most intoxicated individuals are responsible for their impaired state, and the law generally holds that those who voluntarily put themselves in a condition in which they have little or no control over their actions must be held to have intended whatever consequences ensue. In some jurisdictions and for certain crimes, however, voluntary intoxication may lessen criminal liability. Such is the case where a high level of intoxication makes it impossible for a person to form the mens rea necessary for a given offense.

Perfect Self-Defense

A claim of self-defense that meets all of the generally accepted legal conditions for such a claim to be valid.Where deadly force is used, perfect self-defense requires that in light of the circumstances, the defendant reasonably believed it to be necessary to kill the decedent to avert imminent death or great bodily harm and that the defendant was neither the initial aggressor nor responsible for provoking the fatal confrontation

Twelve Tables (derived the Roman law)

A collection of basic rules related to family, religious, and economic life, appear to have been based on common and fair practices generally accepted among early tribes that existed before the establishment of the Roman republic. The Roman law influenced the U.S. legal traditions in many ways

battered woman's syndrome (BWS)

A condition characterized by a history of repetitive spousal abuse and "learned helplessness" (the subjective inability to leave an abusive situation).Also,"a series of common characteristics that appear in women who are abused physically and psychologically over an extended period of time by the dominant male figure in their lives; a pattern of psychological symptoms that develop after somebody has lived in a battering relationship; or a pattern of responses and perceptions presumed to be characteristic of women who have been subjected to continuous physical abuse by their mate[s]."iii Also called battered person's syndrome. As is the case with syndromes generally, BWS is not in itself a defense. It is a condition said to characterize women who live in abusive relationships. BWS may, however, provide additional justification for a woman who kills a battering spouse during an episode of battering— when the threat of serious bodily harm or death is imminent. As an excuse for killings that do not occur within the context of an immediate threat, however, BWS has proven less effective in eliminating criminal liability, although it may lessen it, as provocation generally does (resulting, for example, in a manslaughter conviction rather than a conviction for firstdegree or second-degree murder).

Duress

A condition under which one is forced to act against one's will. Also called compulsion. duress is a defense only when the crime committed is less serious than the harm avoided. Some jurisdictions limit the applicability of the duress defense to less serious crimes; others state that it cannot be used as a defense to a charge of homicide; and still others broaden the ban to all crimes of personal violence.

Void-For-Vagueness Principle

A constitutional principle that refers to a statute defining a crime that is so unclear that a reasonable person of at least average intelligence could not determine what the law purports to command or prohibit Vague Laws: (1) Unclear as to restrictions (2) Discriminatory Enforcement (3) Arbitrarily applied

Identification Model

A corporation can be held responsible for more serious criminal offenses, including those that require a mental element, or mens rea, for their commission. The identification doctrine builds on the principle that a corporation is an abstract entity that has no mind of its own but is actively directed by its officers and senior officials , whose initiatives can be identified with those of the company. Identification liability stops at wrongdoing in the boardroom, while vicarious liability extends corporate responsibility to the acts of all employees

Corporate Criminal Liability

A corporation is a legal entity created under the laws of a state. At one time, it was thought that a corporation could not incur criminal liability because, although a corporation is a legal person, it can act only through its agents (corporate directors, officers, and employees). Therefore, the corporate entity itself could not "intend" to commit a crime. Over time, this view has changed. Obviously, corporations cannot be imprisoned, but they can be fined or denied certain legal privileges (such as necessary licenses).

Natural Crime

A crime against a law of nature rather than a against a legal law, was present in the criminal law at its inceptopm

property crime

A crime committed against property, including (according to the FBI's UCR Program) burglary, larceny, motor vehicle theft, and arson. Example: burglary, arson, criminal mischief (vandalism), property damage, motor vehicle theft, passing bad checks, commission of fraud or forgery, and so on

specific intent

A crime may require not only the doing of an act, but also the doing of the act with a specific intent or objective.

29. INSANITY

A defendant has committed a crime but is not guilty because he is legally insane. Legal insanity requires that a defendant is unable to tell right from wrong and is unable to control oneself. To establish insanity, a defendant must prove (1) mental disease and (2) satisfaction of a substantive doctrinal approach (cognitive, control, product, or MPC). - Mental disease is more likely to be found for psychotic conditions rather than personality disorders. - MPC allows insanity defense for substantial incapacity, while the control and cognitive tests are more willing to extend the defense only for full incapacity or almost full incapacity. ISSUES RAISED BY EACH TEST (all require mental disease): 1. COGNITIVE TEST: Did the defendant's mental disease fully impair his ability to know (1) the nature or quality of his act (mistake of fact), or (2) the wrongfulness of his act (mistake of law)? 2. CONTROL TEST (MINORITY RULE): Did the defendant's mental disease, despite knowing his actions were wrong, fully impair his ability to control his actions? 3. PRODUCT (N.H. ONLY): Was the defendant's mental disease a but-for cause of his otherwise criminal conduct? 4. Due to the defendant's mental disease, did he have substantial incapacity (1) to appreciate the wrongfulness of his conduct (significant emotional understanding) or (2) to conform to conduct required by law?

Diminished Capacity

A defense based on claims of a mental condition that may be insufficient to exonerate a defendant of guilt but that may be relevant to specific mental elements of certain crimes or degrees of crime.Also called diminished responsibility.

syndrome-based defense

A defense predicated on, or substantially enhanced by, the acceptability of syndrome-related claims.

infancy defense

A defense that claims that certain individuals should not be held criminally responsible for their activities by virtue of their youth. Also called immaturity defense.

Impossibility Defense to Attempt

A defense to a charge of attempted criminal activity that claims the defendant could not have factually or legally committed the envisioned offense even if he or she had been able to carry through the attempt to do so. It is, for example, factually impossible to kill someone who is already dead

Necessity

A defense to a criminal charge that claims that it was necessary to commit some unlawful act in order to prevent or avoid a greater harm Identify evil, rank evils, and then choose the lesser evil

Self-Defense

A defense to a criminal charge that is based on the recognition that a person has an inherent right to self-protection and that to reasonably defend oneself from unlawful attack is a natural response to threatening situations Unprovoked, imminent (apparent) danger, necessity, and reasonable force

execution of public duty defense

A defense to a criminal charge, such as assault, that is often codified and that precludes the possibility of police officers and other public employees from being prosecuted when lawfully exercising their authority.

reasonable force

A degree of force that is appropriate in a given situation and is not excessive; the minimum degree of force necessary to protect oneself, one's property, a third party, or the property of another in the face of a substantial threat

PC 20910: Unrecognizable blade weapons: Writing pen knife

A device that appears to be a writing pen, but has concealed a pointed, metallic shaft used as a stabbing instrument. The knife is exposed by: - removal of the device's cap or cover - mechanical action - gravity - locks into place when extended

PC 20310: Unrecognizable blade weapons: Air gauge knife

A device that appears to be an air gauge with a concealed pointed, metallic shaft used as a stabbing instrument. The knife is: - exposed by mechanical action, or gravity - locks into place when extended

PC 21110: Blade weapons: Ballistic knife

A device that propels a knifelike blade as a projectile by means of a coil spring, elastic material, or compressed gas. This does not include any device that propels an arrow or a bolt by means of any common bow, compound bow, crossbow, or underwater spear gun

PC 24310: Camouflaging firearm container

A device: • not readily recognizable as containing a firearm • designed and intended to enclose a firearm • allows the firing of the firearm by external controls while the firearm is in the container

Phishing

A distinct form of identity theft known as phishing has added a different wrinkle to the practice. In a phishing attack, the perpetrator "fishes" for financial data and passwords from consumers by posing as a legitimate business, such as a bank or credit-card company. The "phisher" sends an e-mail asking the recipient to update or confirm vital information, often with the threat that an account or some other service will be discontinued if the information is not provided. Once the unsuspecting individual enters the information, the phisher can use it to masquerade as that person or to drain his or her bank or credit account.

What is a Bill?

A draft of an idea for a new law.

competent to stand trial

A finding by a court that the defendant has sufficient present ability to consult with his lawyer with a reasonable degree of rational understanding and that the defendant has a rational as well as factual understanding of the proceeding against him or her.

incompetent to stand trial

A finding by a court that—as a result of a mental illness, defect, or disability—a defendant is unable to understand the nature and object of the proceeding against him or her or is unable to assist in the preparation of his or her own defense.

PC 33215: Short-barreled rifle

A firearm designed or redesigned to fire a fixed cartridge and has one of the following characteristics: - a barrel length of less than 16 inches - an overall length of less than 26 inches - a modified or altered rifle with one of the above characteristics - any device which can be readily restored to fire a fixed cartridge which, when so restored • any part, or any combination of parts, designed and intended to convert a device into a device defined in subdivision (a) to (c) inclusive, or any combination of parts may be readily assembled if those parts are in the possession or under the control of the same person

PC 33215: Short-barreled shotgun

A firearm designed or redesigned to fire a fixed shotgun shell and has one of the following characteristics: - a barrel length of less than 18 inches - an overall length of less than 26 inches - a modified or altered shotgun with one of the above characteristics - any device which can be readily restored to meet one of the above descriptions - any part(s) designed to convert a device into meeting one of the above description

PC 20910: Unrecognizable firearms: Pen gun

A firearm enclosed within what appears to be a writing pen

Firearm concealment in a vehicle

A firearm is considered concealed in a vehicle if it is stored: in a glove compartment (locked or unlocked) under the seat in a door or center console compartment under another object (e.g., clothing), etc. A concealable firearm may be transported within a vehicle as long as it is locked in the vehicle's trunk or in a locked container

PC 31500: Unrecognizable firearms: Belt buckle gun

A firearm mounted on, or that appears to be an integral part of, a belt buckle that can be detached from the buckle and used as a weapon

PC 24710: Unrecognizable firearms: Wallet gun

A firearm mounted or enclosed in a case resembling a wallet designed to be, or capable of being, carried in a pocket or purse, and that can be fired while mounted or enclosed

PC 24410: Unrecognizable firearms: Cane gun

A firearm mounted or enclosed in a stick, staff, rod, crutch, or similar device designed to be, or capable of being, used as a walking aid and that can be fired while mounted or enclosed

PC 31500: Unconventional pistol

A firearm with both of the following characteristics: • does not have a rifled bore • a barrel length of less than 18 inches or an overall length of less than 26 inches

Deadly Force

A force likely to cause death or great bodily harm.

What is a writ?

A form of written command in the name of a court or other legal authority to act, or abstain from acting, in some way.

Hacking

A hacker is someone who uses one computer to break into another. The danger posed by hackers has increased significantly because of botnets, or networks of computers that have been appropriated by hackers without the knowledge of their owners. A hacker may secretly install a program on thousands, if not millions, of personal computer "robots," or "bots," that allows him or her to forward transmissions to an even larger number of systems.

The Insanity Defense Reform Act (IDRA)

A part of the 1984 Crime Control and Prevention Act that mandated a comprehensive overhaul of the insanity defense as it operated in the federal courts.The IDRA made insanity an affirmative defense to be proved by the defendant by clear and convincing evidence and created a special verdict of not guilty by reason of insanity

Act

A performance, a deed, or movement, as distinguished from remaining at rest

Transferred Intent

A legal construction by which an unintended act that results from intentional action undertaken in the commission of a crime may also be illegal Examples: With malice aforethought Nate Nogood intends to shoot his girlfriend and misses her, and the bullet hits a passerby, killing him. Nogood may be charged with first degree murder since the intent to commit murder is transferred to the actual crime. Steve Swinger takes a punch at Harvey Hasgood, his hated enemy, misses Hasgood and hits Hasgood's date, Teri Truehart in the nose, breaking it. Truehart can not only sue Swinger for damages due to the assault, but can claim punitive damages because the malice against Hasgood attaches to the hit upon Truehart.

Legal Cause (proximate cause)

A legally recognizable cause; the type of cause that is required to be demonstrated in court in order to hold an individual criminally liable for causing harm

Bill of Attainder

A legislative pronouncement that an individual is guilty of a crime. Bills of attainder are prohibited to the federal and state governments by the U.S. constitution. In addition to violating that, the separation of powers doctrine is violated, which reserves the adjudication of criminal cases to the judiciary

crime elements

Are the essential elements required for prosecution purposes. These elements come from the codes.

actus reus

A necessary first feature of most crimes. An act in violation of the law; a guilty act

PC 33600: Zip gun

A non-commercially produced weapon (i.e., not imported, designed, or taxed as a firearm) capable of firing a projectile by the force of an explosion or other form of combustion NOTE: Almost any homemade firearm can qualify as a zip gun

Common Law States

A jurisdiction in which the principles and precedents of common law continue to hold sway

Consent

A justification, offered as a defense to a criminal charge, that claims that the person suffering an injury either agreed to sustain the injury or accepted the possibility of injury before the activity was undertaken.

PC 20610: Unrecognizable blade weapons: Lipstick case knife

A knife enclosed within, and made part of, a lipstick case

PC 20410: Unrecognizable blade weapons: Belt buckle knife

A knife which is an integral part of a belt buckle and consists of a blade with a length of at least 2½ inches

ignorance of the law

A lack of knowledge of the law or of the existence of a law relevant to the situation at hand. neither ignorance of the law nor a misunderstanding of the law provides for an acceptable defense, and criminal proceedings assume that "every one capable of acting for himself knows the law."18 This assumption does not mean, of course, that everyone is actually familiar with each and every law, but it effectively compels people to learn the standards set by the law in their sphere of activity ignorance or mistake of law may be a defense in cases in which a given offense requires specific intent or in which mistake of law negates the mens rea required by a statute. Ignorance of the law may also be an excuse where the law is not adequately published or is incapable of being known.

What is a Statute?

A law enacted by a legislature

Fleeing Felon Rule

A now-defunct law enforcement practice that permitted officers to shoot a suspected felon who attempted to flee from a lawful arrest.

Reasonable Person

A person who acts with common sense and who has the mental capacity of an average, normal, sensible human being.The reasonable person criterion requires that the assumptions and ideas on which a defendant acted must have been reasonable in that the circumstances as they appeared to the defendant would have created the same beliefs in the mind of an ordinary person.

Causation:

A person who commits a negligent act is not liable unless his or her act was the cause of the plaintiff's injuries. The two types of causation that must be proven are (1) causation in fact (actual cause) and (2) proximate cause (legal cause). - Factual causation: "but for" test. Actor's conduct sat a chain of events in motion which led to the harmful result. - Proximate cause: indicates a nearness on time and place between tha actor and the result

Accessory After the Fact

A person who did not participate in a crime but who furnished postcrime assistance to keep the offender from being detected or form being arrested

Insanity

A person who suffers from a mental illness may be incapable of the state of mind required to commit a crime. Thus, insanity may be a defense to a criminal charge. Note that an insanity defense does not enable a person to avoid imprisonment. It simply means that if the defendant successfully proves insanity, she or he will be placed in a mental institution.

Principal in the Second Degree

A person who was present at the crime scene and who aided, abetted, counseled, or encouraged the principal

Accomplice

A person who, with intent to promote or facilitate the commission of crime, gives assistance or encouragement to the principal. An accomplice is liable as a principal before and during a crime (1) Intent to Aid (2) Intent to commit underlying offense (3) Assistance by: Physical Conduct, psychological influence, and omission

Principal in the First Degree

A person whose acts directly resulted in the criminal misconduct in question

Motive

A person's reason for committing a crime. NOT the same as mens rea!

Natural Rights Theory

A philosophical outgrowth of natural law . The theory holds that individuals naturally possess certain freedoms that may bit be encroached upon by other individuals or governments

What is "Stare Decisis?"

A principle that requires a court to follow its own prior decisions when faced with the same legal issue. From "stare decisis et quieta non movere," meaning "to stand by things decided and not disturb settled points."

Tort

A private or civil wrong or injury; "the unlawful violation of a private legal right other than a mere breach of contract, express or implied. A tort may also be the violation of a public duty if, as result of the violation, some special damage accrues to the individual." A tort may give rise to civil liability, under which the injured party may use the person or entity that caused the injury and ask that the offending party be ordered to pay damages. Civil law is more concerned with assessing the liability than it is with intent

M'Naughten rule

A rule for determining insanity that asks whether the defendant knew what he or she was doing or whether the defendant knew that what he or she was doing was wrong

Jurisdiction and Identification Challenges

A threshold issue is, of course, jurisdiction. Jurisdiction is normally based on physical geography, as discussed in Chapter 2. Each state and nation has jurisdiction, or authority, over crimes committed within its boundaries. But geographic boundaries simply do not apply in cyberspace. A person who commits an act against a business in California, where the act is a cyber crime, might never have set foot in California but might instead reside in New York, or even in Canada, where the act may not be a crime. Identifying the wrongdoer can also be difficult. Cyber criminals do not leave physical traces, such as fingerprints or DNA samples, as evidence of their crimes. Even electronic "footprints" can be hard to find and follow. For instance, e-mail may be sent through a remailer, an online service that guarantees that a message cannot be traced to its source. For these reasons, laws written to protect physical property are often difficult to apply in cyberspace. Nonetheless, governments at both the state and the federal level have taken significant steps toward controlling cyber crime. California, for instance, which has the highest identity theft rate in the nation, has established a new eCrime unit to investigate and prosecute cyber crimes. Other states, including Florida, Louisiana, and Texas, also have special law enforcement units that focus solely on Internet crimes.

Legal Defense: Justification

A type of legal defense in which the defendant admits to committing the act in question but claims it was necessary in order to avoid some greater evil

Excuses (an affirmative defense)

A type of legal defense in which the defendant claims that some personal condition or circumstance at the time of the act was such that he or she should not be held accountable under the criminal law.

Criminal Law

A wrong against society set forth in a statue and punishable by a fine and or imprisonment -- or in some cases, death. -- because crimes are offenses against society as a whole, they are prosecuted by a public official, such as a district attorney (D.A.) or an attorney general (A.G.), not by the victims. Once a crime has been reported, the D.A.'s office decides whether to file criminal charges and to what extent to pursue the prosecution or carry out additional investigation.

State of Mind

A wrongful mental state, or mens rea, also is typically required to establish criminal liability. The required mental state, or intent, is indicated in the applicable statute or law. Murder, for example, involves the guilty act of killing another human being, and the guilty mental state is the desire, or intent, to take another's life. For theft, the guilty act is the taking of another person's property. The mental state involves both the awareness that the property belongs to another and the desire to deprive the owner of it.

The intent of the Criminal Justice Realignment Act of 2011 was to: a. Change the place where sentences for certain crimes are to be served b. Limit the prosecution of juvenile offenders c. Build more prisons in the state of California d. Eliminate misdemeanor crime

A) Change the place where sentences for certain crimes are to be served

In California, how many people must be involved in the crime of conspiracy?

A) One or more B) Two or more C) Three or more D) Four or more Answer: (B) Two or more

Which statements are true?

A) Resists, Disobeys, Interferes, Delays, or even "exhorts others" to do same, with firefighter or emergency rescue personnel at a fire is a misdemeanor, PC 148.2. B) PC 148.3 filing a false police report of a felony, misdemeanor or emergency. C) Falsely reporting an emergency involving great bodily injury or death is a felony and $10,000 fine. D) PC 148.4 willfully and maliciously tampers with, molest, injures, or breaks any fire protection equipment, fire protection installation, fire alarm apparatus, wire, or signal or sends, gives, transmits, or sounds any false alarm of fire, by means of any fire alarm system or signal or by any other means or methods. This is a felony with a $10,000 fine if (1) great bodily injury or (2) death occurs. E) All of the above. Answer: (E) All of the above

The principle that binds courts to stand by prior decisions and to not disturb settled points of law a. Stare Decisis b. Certiorari c. Ex post facto principle d. Writ of prohibition

A) Stare Decisis

Which one of the following constitutes an attempt to commit a crime?

A) Two or more persons plan a robbery and buy a gun. B) "A" tries to kill "B" using baking powder thinking it to be poison. C) "X" buys explosives to be used in a safe burglary. D) Intending to commit a burglary, "A" removes a window screen but is frightened away. Answer: (A) Two or more persons plan a robbery and buy a gun.

21. ABANDONMENT (MPC & COMMON LAW)

ABANDONMENT Occurs when a defendant abandons an attempt before crime's completion, but perhaps with sufficient actus reus and mens rea to satisfy an attempt crime. Abandonment demonstrates that the defendant is not actually dangerous. Common Law (Majority) Rule: Abandonment is not a defense (but it may reduce sentence). MPC (Minority) Rule: Abandonment is a defense only if D voluntarily abandoned the criminal effort in a complete and voluntary way, as to denounce his criminal purpose. - Abandonment is not voluntary if it is motivated by an external factor that threatens to impede the defendant's commission of the offense or increases likelihood of apprehension. - Sensible since the MPC broadens attempt liability. Ross v. State of Mississippi: Man abandoned his attempted rape when the victim pled with him. - Rule: Abandonment involves conscious renunciation of conduct. Where a defendant, with no other impetus other than a victim's pleading, voluntarily ceases his conduct, he has abandoned his attempt. However, where D ceases the conduct because of V's or third party's intervention, D has not voluntarily abandoned the attempt.

Reasonable Failures

Acts that might be noncriminal, even when serious harm results. Example: The captain of a passenger airplane may decide to avoid an emergency landing to save the life of a heart attack victim if doing so might endanger the other passengers

8. CAUSATION (COMMON LAW)

Actual Cause: "but for" the conduct of D, result would have occurred when and how it did. - The result would not have occurred without the actions of D - If the result would not have occurred OR would have occurred in a meaningfully different manner, then no result Proximate Cause: If D could have anticipated that her/his conduct would bring about the statutory harm in the way that it happened, then that conduct will count as a proximate cause of the harm. a. Whether the harm that actually occurred, in the way that it occurred (including any causal interventions from other forces), closely resembled the harm that the defendant did or should have anticipated. b. Generally, was the result a foreseeable consequence of the harm? c. Inquiry: How closely must the ultimate harm come to resembling what was or should have been anticipated? What breaks Chain of Liability? a. Remoteness: Too remote or accidental to have a just bearing b. Not Foreseeable: The result came about in a manner that was not foreseeable based on the conduct. c. Wrongful intervention: Was an intervening actor's conduct wrongful enough to break the causal chain? d. Intervention MAY NOT provide relief from liability e. Responsibility of Victim: Refusal to accept medical treatment will not break the chain

Exceptions to Miranda Rule

Although the Supreme Court's decision in the Miranda case was controversial, it has survived several attempts by Congress to overrule it. Over time, however, the Supreme Court has made a number of exceptions to the Miranda ruling. For instance, the Court has recognized a "public safety" exception that allows certain statements to be admitted even if the defendant was not given Miranda warnings. A defendant's statements that reveal the location of a weapon would be admissible under this exception. Additionally, a suspect must unequivocally and assertively ask to exercise her or his right to counsel in order to stop police questioning. Saying, "Maybe I should talk to a lawyer" during an interrogation after being taken into custody is not enough.

At Trial

An accused may not be tried if not rational or if incapable of assisting in his or her own defense Trial may proceed when defendant regains ability to assist in defense. Temporary detention is permitted while awaiting competency restoration. Longer detentions must occur through civil commitment procedures

Mere Preparation

An act or omission that may be part of a series of acts or omissions constituting a course of conduct planned to culminate in the commission of a crime but that fails to meet the requirements for a substantial step (attempt). Also, preparatory actions or steps taken toward the completion of a crime that are remote from the actual commission of the crime

Criminal Conspiracy

An argument between two or more people to commit or to effect the commission of an unlawful act or to use unlawful means to accomplish an act that is not unlawful (An "Agreement" to Commit a Crime) Conspiracy ends as soon as people were caught!!!! The person doing the ACT gets charged with conspiracy and (i.e., murder), the person who started it would be charge with solicitation, and, i.e, accomplice

Principle of Legality

An axiom that holds that behavior cannot be criminal if now law exists that defines it as such.Today, the principle of legality is commonly analyze as a due process principle

Felony Murder

An exception to the general notion that criminal liability does not accrue when the harm that results is different in kind from the harm intended. Felony murder statutes hold a person involved in the commission of a felony responsible of homicide if another person dies during the offense, even though the death may have been unintentional

The castle exception

An exception to the retreat rule that recognizes a person's fundamental right to be in his or her home and also recognizes the home as a final and inviolable place of retreat. Under the castle exception to the retreat rule, it is not necessary to retreat from one's home in the face of an immediate threat, even where retreat is possible, before resorting to deadly force in protection of the home

The Miranda Rule

An important question many courts faced in the 1950s and 1960s was not whether suspects had constitutional rights—that was not in doubt—but how and when those rights could be exercised. Could the right to be silent (under the Fifth Amendment's protection against self-incrimination) be exercised during pretrial interrogation proceedings or only during the trial? Were confessions obtained from suspects admissible in court if the suspects had not been advised of their right to remain silent and other constitutional rights? To clarity these issues, the United States Supreme Court issued a landmark decision in 1966 in Miranda v. Arizona, which we present here. Today, the procedural rights required by the Court in this case are familiar to almost every American.

Entrapment

An improper or illegal inducement to crime by enforcement agents. Also, a defense that may be raised when such inducements occur. If the individual is known to possess illegal drugs and then an uncover police officer affords a chance to sell and the individual consents to sale, a legal arrest can be made. If the individual refuses the sale and the officer offers ten times his original price and the individual consents to sale, the arrest would be illegal Entrapment cannot be effectively raised as a defense, however, where government employees "merely afford opportunities or facilities for the commission of the offense" or where law enforcement officers or their representatives engage in the "mere fact of deceit."

Strict Liability and Overcriminalization

An increasing number of laws and regulations impose criminal sanctions for strict liability crimes. Strict liability crimes are offenses that do not require a wrongful mental state to establish criminal liability. Strict liability crimes are particularly common in environmental laws, laws aimed at combatting illegal drugs, and other laws affecting public health, safety, and welfare. Under federal law, for example, tenants can be evicted from public housing if a member of the household or a guest used illegal drugs. The eviction can occur regardless of whether the tenant knew or should have known about the drug activity.

Inside Trading

An individual who obtains "inside information" about the plans of a publicly listed corporation can often make stock-trading profits by purchasing or selling corporate securities based on this information. Insider trading is a violation of securities law. Basically, securities law prohibits a person who possesses inside information and has a duty not to disclose it to outsiders from trading on that information. A person may not profit from the purchase or sale of securities based on inside information until the information is made available to the public.

AT TIME OF ACT

An individual's act may be excused if, at the time of the crime, he or she is legally insane under the applicable standard (M'Naughten, MPC, etc.) If not guilty by reason of insanity, accused is assessed and committed if dangerous. If not, he or she is released If guilty but mentally ill, convict is provided treatment during incarceration/ punishment

Tortfeasor

An individual, business, or other legally recognized entity that commits a tort

PC 22410: Blade weapons: Shuriken

An instrument, without handles, that is used as a throwing weapon. A shuriken consists of a metal plate having three or more radiating points with one or more sharp edges and designed in the shape of a star, polygon, trefoil, cross, diamond, or other geometric form

PC 22010: Striking Weapons: Nanchaku

An instrument: • consisting of two or more sticks, clubs, bars, or rods to be used as handles • connected by a rope, cord, wire, or chain • in the design of a weapon • used in connection with the practice of a system of self-defense (e.g., karate or other forms of martial arts) It is lawful to manufacture a nunchaku for sale to, or to sell to, a school which holds a regulatory or business license and teaches the art of self-defense. It is also lawful to possess a nunchaku on the premises of such a school

Omission to Act

An intentional or unintentional failure to act, which may impose criminal liability if a duty to act under the circumstances is specified by law i.e., relationship, statute, contract, assumption of care, creation of peril, duty to control conduct, landowner duty ome examples: failing to file a required tax return, failing to register locally as a convicted felon, failure of a parent to obtain medical help for a child, failure to stop after a vehicle collision, failure to rescue someone fallen overboard from your ship. Other examples are a bit harder to spot: failure to stop someone who is beating your child, failing to rescue a comrade injured in a remote wilderness (murder?). Failure to maintain fire safety in a nightclub has resulted in indictments for criminal manslaughter when deaths occur from a fire. There may be a difference in punishment between negligent omissions and reckless or intentional omissions. The legal duty to act may arise from the relationship with the victim (parents), or relationship with the criminal (parents, employees), or by statute/ordinance (specific definitions of required acts), contract (e.g., crossing guard), land ownership (pollution, security for visitors), intentional creation of danger (punched someone in the street and left him to be run over), or by voluntary assumption of the duty (guardians).

morals offense

An offense that was originally defined to protect the family and related social institutions. Included in this category are crimes like lewdness, indecency, sodomy, and other sex-related offenses, such as seduction, fornication, adultery, bigamy, pornography, obscenity, cohabitation, and prostitution. Denote a category of unlawful conduct that was criminalized originally to protect the family and related social institutions. This category includes lewdness, indecency, sodomy, and other sex-related offenses, such as seduction, fornication, adultery, bigamy, pornography, obscenity, cohabitation, and prostitution

Norm

An unwritten rule that underlies and inherent in the fabric of society

More

An unwritten, but generally known, rule that governs serious violations of the social code

PC 32310: Large capacity magazine

Any ammunition-feeding device which has the capacity to accept more than ten rounds: • may not be manufactured, imported, kept or exposed for sale, given or loaned except by law enforcement agencies, California peace officers, or licensed dealers; • importation not continuously possessed by the same owner prior to January 1, 2000.

PC 25605: Exempt from the license requirement for possession of concealable firearm

Any citizen or legal resident of the United States who: is over the age of 18 years resides or is temporarily within the State of California is not prohibited by law (e.g., convicted felon, mental patient, etc.) may, without a permit or license: purchase, own, possess, keep, carry, either openly or concealed any firearm that is capable of being concealed upon that person within that person's: place of residence, place of business, on private property that is owned or lawfully possessed by that person

Liability of Conspiracy

Any conspirator is liable for any crime that was a natural or probable consequence of any crime a co-conspirator commits even if the crime was not intended.

PC 21710: Striking weapons: composite knuckles

Any device or instrument: - made wholly or partially of composite materials, other than a medically prescribed prosthetic - that is not metal knuckles are worn for purposes of offense or defense in or on the hand - that either protects the wearer's hand while striking a blow or increases the force of impact from the blow or injury to the individual receiving the blow

PC 417.3: Drawing or exhibiting a firearm to a person in a motor vehicle (Felony)

Any person except in self-defense to draw or exhibit any firearm (loaded or unloaded) in a threatening manner such as to cause a reasonable person apprehension or fear of bodily harm in the presence of another person who is the occupant of a motor vehicle proceeding on a public street or highway.This law applies only to vehicles that are on public streets, roadways or highways (including stopping for lights, signs, or other impediments in the flow of traffic). It does not apply to vehicles that are parked in a driveway, atthe curb, or in a parking lot.

PC 30510: Possession of assault weapon

Any person possesses an assault weapon defined under Penal Code Section 30510 unless: - Weapon obtained prior to June 1, 1989 - Lawfully registered - 50 cal. rifle of any type that can fire a .50 BMG cartridge (Penal Code Section 30530(a)) • Exception lawfully possessed before Jan, 1 2005 • Penal Code Section 30905 mandates registration by April 30, 2006 • Penal Code Section 30900 defines registration

PC 626.9: 8) School safety zones, playgrounds, youth facilities (Felony)

Any person who Possesses a firearm (loaded or unloaded) In a place that the person knows or reasonable should have known In a school safety zone, Without written permission of the school district superintendent or designee

PC 171b: Possession of an unauthorized weapon in state o public building (Felony)

Any person who brings or possesses a specified weapon within any state or local public building to any meeting required to be open to the public

PC 25400: Unlawful possession of a concealable firearm without a license (Misdemeanor)

Any person who carries concealed a firearm that is capable of being concealed (loaded or unloaded) upon one's person within any vehicle under that person's control or direction without a license or permit A key element of this crime is that the firearm is carried concealed. A firearm in plain view where there is no intent to conceal it would not be in violation

PC 23900: Alteration or removal of identifying marks from a firearm (Felony)

Any person who changes, alters, removes or obliterates the name of the maker, model, manufacturer's number, other mark of identification including any distinguishing number or mark assigned by the Department of Justice on any firearm without having secured written permission from the Department of Justice to make such a change, alteration or removal

PC 246: Discharge a firearm at an inhabited or occupied dwelling, vehicle, or aircraft (Felony)

Any person who maliciously and willfully discharges a firearm at an: inhabited dwelling house, inhabited housecar, inhabited camper,occupied building, occupied motor vehicle, occupied aircraft

PC 16590: Possession of a Prohibited Weapon (Felony)

Any person who manufactures or causes to be manufactured imports into the state sells, offers for sale, exposes for sale, keeps for sale gives, lends, or possesses any weapon that has been specified within this penal code section as prohibited

PC 417(a)(2):Drawing, exhibiting, or unlawful use of a firearm (Misdemeanor)

Any person who while in the presence of another person, except in selfdefense draws or exhibits any firearm (loaded or unloaded) in a rude, angry, or threatening manner unlawfully uses a firearm in any fight or quarrel Actual injury or assault is not required for the crime to be complete.

PC 22810: Possession of Tear gas or tear gas weapon (Misdemeaor)

Any person, firm, or corporation who knowingly: possesses, sells, offers for sale, exposes for sale, transports, any prohibited tear gas or tear gas weapons except when otherwise permitted

Persons on probation

As an expressed condition of probation, probationers may be prohibited or restricted from: owning, possessing, having custody or control of any firearm Individuals who have such restrictions to their probation and are found to own, possess, or have custody or control of any firearm are guilty of a FELONY.(PC 29815(a) PC 26045(a)-(c) may exempt individuals who reasonably believe that the property or person of another or of themselves is in immediategrave danger.

Unequivocal Test

At the point when conduct can no longer be either criminal or noncriminal, an attempt is made. Since holding a victim against his or her well is a crime, the act is unequivocal (victim rendered physically incapacitated) Defendant is culpable if he or she has unequivocally manifested an intent to commit the crime

27. BWA

Battered Woman Syndrome is not itself a separate justification. It merely guides the fact finders' understanding of the circumstances. Psychological evidence is relevant to establish the genuineness and reasonableness of a defendant's belief. BWS has both a psychological and external societal component. Majority Rule: BWS can demonstrate genuineness or reasonableness of D's fear of a partner's attack, but the traditional elements of self-defense (trigger, imminence, reasonableness) must still be satisfied. Most jurisdictions maintain the imminence requirement out of fear of legalizing vigilanteism. Minority Rule: Some jurisdictions have relaxed the imminence requirement in the context of battered women and children. The rationale is that battered women cannot actually defend themselves properly when a threat to their life is truly imminent. New Jersey v. Kelly: Husband assaulted his wife in public and she stabbed him in self-defense. - Rule: Expert testimony tending to establish BWS is admissible to help a jury analyze both (1) honest belief in necessity of force, and (2) reasonableness in that belief. Admission of evidence also helps explain to jury why the defendant stayed in the marriage despite a history of beatings. North Carolina v. Norman: Husband beat his wife constantly. After an escalation in the beatings, the wife shot her husband while he slept. - Majority Rule: D is entitled to self-defense only when: (1) she believed deadly force necessary to save herself from imminent death or GBI, and (2) such a belief was reasonable. Imminence cannot be relaxed because it would legalize vigilanteism. o Argue that force was necessary at the time she chose because it was the only opportunity she had to defend herself. Washington v. Janes: Stepfather beat his stepson constantly, so the son formulated a plan and killed him. - Minority Rule: Self-defense requires the defendant to show: (1) a reasonable apprehension of great personal injury, and (2) imminent danger of that injury being done in the near future. Imminence is a subjectivized objective standard that assesses the defendant's perception in light of all facts perceived by the defendant.

Steps in Criminal Case: Arrest

Before a warrant for arrest can be issued, there must be probable cause to believe that the individual in question has committed a crime. As discussed earlier in this chapter, probable cause can be defined as a substantial likelihood that the person has committed or is about to commit a crime. Note that probable cause involves a likelihood, not just a possibility. Arrests can be made without a warrant if there is no time to get one, but the action of the arresting officer is still judged by the standard of probable cause.

What are three ways a judge/judicial opinion can add to the body of law?

By announcing a new principle of law (common law); By interpreting or clarifying a constitution, statute, or regulation; By applying the law to a new set of facts, creating precedent to be followed in the future.

How Can a Court Deduce A Statute's Meaning?

By looking at the common law, if it influenced the statute, by examining relevant case law, by looking at similar statutes, by looking to the law of other jurisdictions, and by checking the legislative history

How Does Constitutional Provisions Determine The Nature Of Criminal Law?

By setting limits on just what can be criminalized, or made legal. Constitutional requirements hold that criminal laws can only be enacted where there is a compelling public need to regulate conduct. The U.S. Supreme Court has held that "to justify the exercise of police power the public interest must require the interference, and the measures adopted must be reasonably necessary for the accomplishment if the purpose The Constitution also demands that anyone accused of criminal activity be accorded due process. Similarly, the Constitution helps ensure that the accused are provided with the opportunity to offer a well-crafted defense

Controlling criminal law may be found in which of the following sources? a. State and federal constitutions and statutes only b. Statutes and published court decisions only c. Constitutions, statutes and published court decisions d. Constitutions, statutes, published court decisions and contracts

C) Constitutions, statutes and published court decisions

Our criminal law in this country comes from three primary sources: Select the one that DOES NOT apply: a. United States Constitution b. Case Law c. Procedural Law d. Statutes passed by Congress and the State Legislation

C) Procedural Law

To what does the 87 refer in the case citation: Reeler v. Superior Court, 87 Ca.App. 4th 481? a. The district in which the case was tried b. The page on which the case appears c. The volume of the reports in which the case appears d. The number assigned to the case on appeal

C) The volume of the reports in which the case appears

-unconscious defense-

Causes of unconsciousness: If you are unconscious while you commit a crime, California law excuses your actions.26 This legal defense is applicable to people who, for example, commit criminal acts while sleeping, delirious, suffering from an epileptic seizure, or who are involuntarily intoxicated. On that note, it is important to understand that if you commit a crime while unconscious or sleeping... but while voluntarily intoxicated... you will still be held criminally liable. The most common example of this is "sleep-driving" while under the influence of Ambien or Lunesta.

Which of the following is correct regarding the construction of penal statutes: a. Are to be strictly construed b. The rule of the common law prevails c. Are to be construed with a fair import of terms d. Letter of the law prevails in question of conflict

C) Are to be construed with a fair import of terms

Accessory

Can only be an accessory after the fact. The crime must be a felony. Can only be an accessory for harboring, aiding or helping the principle after the crime took place with the specific intent of helping the principle escape or avoid arrest, trial and conviction. Must have knowledge of the crime.

...

Cane Gun

12020(a)

Carrying CONCEALED upon the person; wobbler.

What type of law does the Judicial Branch produce?

Case Law

What is the difference between "common law" and "case law"?

Case law includes ANY judicial decision. Common law is a subset of case law and refers to only those areas of case law that developed in the absence of statute.

The Essence of Criminal Conduct

Consists of concurrence of a criminal act with a culpable mental state. The critical issue of concurrence is what distinguishes murder from homicide committed in self-defense, for example, or rape from consensual sex

Violent Crime

Certain crimes are called violent crimes, or crimes against persons, because they cause others to suffer harm or death. Murder is a violent crime. So is sexual assault, or rape. Robbery—defined as the taking of money, personal property, or any other article of value from a person by means of force or fear—is also a violent crime. Typically, states have more severe penalties for aggravated robbery—robbery with the use of a deadly weapon. Assault and battery, which will be discussed in Chapter 12 in the context of tort law, are also classified as violent crimes. --Each violent crime is further classified by degree, depending on the circumstances surrounding the criminal act. These circumstances include the intent of the person committing the crime and whether a weapon was used. For crimes other than murder, the level of pain and suffering experienced by the victim is also a factor.

Dissociative Disorder

Characterized by a splitting or dissociation of normal consciousness. Included here are multiple personalities, psychogenic amnesia, and psychogenic fugue.

Impulse Control Disorders

Characterized by an inability to control impulses and by impulsive behavior. Included are kleptomania, pyromania, and pathological gambling.Addictions, including drug addiction, alcoholism, and food addition, may also fall into this category

Schizophrenia

Characterized by symptoms like disordered thought processes and delusions or hallucinations. Symptoms must exist for more than six months for a diagnosis of schizophrenia to be made. Catatonic and paranoid schizophrenia are subtypes.

Under PC 26 all persons are capable of committing a crime expect:

Children under the age of 14 Person acting under threat or duress Person acting under mistake of fact Persons who act by accident Persons who are mentally incapacitated

Personality Disorders

Chronic, inflexible, and maladaptive personality patterns that are generally resistant to treatment.Antisocial personality disorder is one subtype.

17. INTOXICATION (COMMON LAW & MPC)

Common Law (Majority) Rule: Evidence of voluntary intoxication is relevant only to negate specific intent, while evidence of involuntary intoxication is a complete defense against both specific and general intent if it completely incapacitates the defendant such that the defendant is legally "insane." (Majewski) - There is no voluntary intoxication defense for general intent crimes because voluntary intoxication is itself negligent. Common Law (Minority) Rule: Evidence of voluntary intoxication can never be admitted as a defense. (Egelhoff) - Deters drinking to excess (both generally and in the specific instance) - Comports with society's morals that one who is voluntarily impaired should be responsible for his consequences. - Avoids jury confusion. Involuntary Intoxication: never a basis for diminished capacity defense, only serves to negate mens rea. However, both Common Law and MPC recognize involuntary intoxication as a defense when it yields complete incapacity that meets the legal test for insanity (Egelhoff) ("A drugged intent is still an intent") MPC Rule: Intoxication is no defense unless it negates an element of the offense. Self-imposed unawareness is immaterial when recklessness is an element of the offense. Evidence of intoxication can only be used to negate purpose or knowledge. - Moral equivalence justification (i.e. if you voluntarily drink to excess, you are the reason you're not aware of the circumstances). - Voluntary intoxication is never a defense to negligence because it is negligent to drink to excess. Director of Public Prosecutions v. Majewski: Defendant voluntarily took speed and drank and then assaulted the bar owner. - Rule: Intoxication is only a defense to specific intent offenses. Drinking itself establishes at least negligence. Montana v. Egelhoff: Defendant drank to an intoxication level of 0.36 and shot his two companions in the head. Montana prohibited intoxication as a defense. - Rule: State laws that reject all voluntary intoxication defenses do not violate due process. Since half of homicides involve intoxicated defendants, state has good reason to believe the ban will have important effects for deterrence and incapacitation. Regina v. Kingston: Defendant was slipped drugs to get him to release his pedophilic tendencies. Defendant subsequently molested a young boy. - Rule: If the defendant still had the specific intent to commit a crime despite being involuntarily intoxicated, he is guilty. Absence of moral fault is insufficient to negative mens rea - a drugged intent is still an intent. o Court was worried about disentangling the multiple causes.

Credit Card Numbers

Companies take risks by storing their online customers' credit-card numbers. Although the consumer can make a purchase more quickly without entering a lengthy card number, the electronic warehouses that store the numbers are targets for cyber thieves. Stolen credit-card numbers are much more likely to hurt merchants and credit-card issuers (such as banks) than consumers. In most situations, the legitimate holders of credit cards are not held responsible for the costs of purchases made with a stolen number.

23. ACCOMPLICE LIABILITY - COMPLICITY: MENS REA (COMMON LAW & MPC)

Complicity mens rea always concerns the mens rea of the defendant with respect to the other actor's doing of the actus reus. Common Law Rule: Sliding scale between purpose and knowledge which takes into account the seriousness of the offense and the facts of the specific case. - Natural and Probable Consequences Rule (some CL jurisdictions): If a defendant has shown complicity in a prior offense and the ultimate offense was the natural and probable consequence of the prior offenses in the way they were committed, then he is guilty of the ultimate offense regardless of whether he satisfies the complicity requirements for the ultimate offense. o Parallels the felony murder doctrine because the defendant is punished as if he knew or intended the ultimate offense. MPC Rule: D is accomplice if (1) he is legally accountable for the other actor's conduct, and (2) for - Conduct: Possesses Purpose of promoting/facilitating an offense - Result: Possesses culpability for underlying offense - Circumstance: Possesses Hope or Belief of the circumstance. Facilitation: When a defendant in fact renders aid to a principal, believing it probable that he is rendering aid, he is guilty of facilitation, a lower-grade offense than actual complicity offense. - Easier to prove facilitation, since it requires only "probable belief" and not purpose or knowledge - Adopted by some states as a middle ground between the Common Law and MPC approach. United States v. Peoni (Conduct): Defendant was charged as an accessory to possessing counterfeit bills when third party was caught with the counterfeit bills defendant created. - Rule: D is an accomplice only if he purposely promoted/facilitated the other actor's criminal conduct. Purpose requires that the actor have a stake in the venture (narrow interpretation). Backun v. United States (Conduct): Defendant knowingly sold stolen silverware to another, who transported it across state lines. Defendant knew the other actor would transport it across state lines. - Rule: Defendant is an accomplice if he knowingly promotes or facilitates the other actor's criminal conduct. o Defendant ultimately caused the commission of the offense (mastermind) and his purpose was to have the goods transported in interstate commerce. Michigan v. Marshall (Result): Defendant gave his car keys to his friend, knowing he was drunk, and then went to sleep. Friend subsequently hit another driver and killed himself and the other driver. - Rule (Sliding Scale Minority Rule): A defendant must possess purpose or knowledge with respect to the principal actor's result. Where the offense was not counseled by the defendant, accomplished by another acting jointly with him, or occurred in the attempted achievement of a common enterprise, the defendant does not possess purpose or knowledge of the result. o Courts are generally conservative where the defendant didn't actually do anything. Story v. United States (Result): Drunken defendant permitted his friend, who he knew was drunk, to drive his car while he sat in the passenger seat. Friend ultimately hit and killed third companion. - Underlying Offense Rule: If D knowingly puts a dangerous instrumentality in the hands of another, sits by his side, and permits him to behave recklessly, without protest, the defendant is just as responsible for the offense as the principal actor. California v. Durham: Defendant and partner went on a crime spree and his partner ultimately killed a cop. Prior to killing, partner had used his gun in a robbery. - Natural & Probable Consequence Rule: A defendant must satisfy the complicity requirements for the first offense and the death must be a probable consequence of the prior offense in the way it was committed.

12025(a)(2) P.C.

Concealed weapon upon PERSON. Anyone who carries upon person a pistol, revolver, or any other firearm capable of being concealed is a MISDEMEANOR. if caught with second conviction, it will be a FELONY.

Private Law

Concerned itself with contracts, personal possessions, the legal status of various types of persons (citizens, free persons, slaves, freedom, guardians, husbands, and wives, ans so on), and injuries to citizens

conduct crime and a result crime

Conduct and result crimes are those in which the actus reus is defined in terms of a prohibited outcome that has to be caused in a particular way by specific conduct. Arson involves a combination of a prohibited result (damage or destruction of property) and conduct (the property must be destroyed by fire).

Conspiracy and overt act

Conspiracy is an agreement between two or more persons to commit the offenses charged or to aid, abet, counsel or encourage, or otherwise procure their commission and with the specific intent to commit such offenses, followed by an overt act committed in this State by one or more of the parties for the purpose of accomplishing the object of the agreement

Liabilities of the Corporate Officers and Directors

Corporate directors and officers are personally liable for the crimes they commit, regardless of whether the crimes were committed for their private benefit or on the corporation's behalf. Additionally, corporate directors and officers may be held liable for the actions of employees under their supervision. Under the responsible corporate officer doctrine, a court may impose criminal liability on a corporate officer who participated in, directed, or merely knew about a given criminal violation.

Court can reduce to a misdemeanor 17B5

Court can reduce to a misdemeanor 17B5 this can only happen at preliminary hearing People v estabar. - court can reduce during hearing

6. VOLUNTARINESS

Crime requires at least one voluntary act. "Every crime must have a conduct (actus reus) element composed of (1) a voluntary act (2) that causes social harm. Mere thoughts alone are not sufficient." Voluntariness: Bodily action completed or not completed as a result of the actor's will (e.g. choice). Separate from "conduct": demands specific conduct with a mental state: - Involuntary Conduct: 1. Physically coerced movement 2. Reflexes 3. Medical issues 4. Unconscious acts 5. Concussed sleep walking 6. Structural arguments (e.g. "I had to steal to eat.") Justification for Requirement A. Retributivist: No conduct for which to blame the individual. B. Consequentialist: No deterrence for involuntary conduct.

Mala In Se

Crimes those that are regarded, by tradition and convention, as wrong in themselves. Such acts are said to be inherently evil and immoral and are sometimes called against conscience. Mala in se crimes, such as murder, rape, and other serious offenses, are almost universally condemned and probably would be so even if strictures against behaviors were not specified in the criminal law

criminal law vs. civil law

Criminal Law is the body of rules and regulations that defines and specifies punishments for offenses of a public nature or for wrongs committed against the state or society. Also called penal law. Civil law is the form of the law that governs relationships between parties

TRANSFERRED INTENT

Criminal intent can be transferred from one object to another 1) the specific intent requirement is written into the statue defining the crime. It can be recognized by inclusion of words that call for a particular state of mind (knowingly...willful...maliciously)

Criminal Procedures

Criminal law brings the force of the state, with all of its resources, to bear against the individual. Criminal procedures are designed to protect the constitutional rights of individuals and to prevent the arbitrary use of power on the part of the government. The U.S. Constitution provides specific safeguards for those accused of crimes. The United States Supreme Court has ruled that most of these safeguards apply not only in federal court but also in state courts by virtue of the due process clause of the Fourteenth Amendment. These protections include the following:

Willful Blindness Doctrine

Criminal liability can accrue where a corporation deliberately disregards criminal misconduct

Criminal Negligence

Criminal negligence involves the mental state in which the defendant takes an unjustified, substantial, and foreseeable risk that results in harm. A defendant can negligent even if she or he was not actually aware of the risk but should have been aware of it. A homicide is classified as involuntary manslaughter when it results from an act of criminal negligence and there is no intent to kill.

Insanity

Criminal's state of mind AT THE TIME THE CRIME WAS COMMITTED -requires due to mental illness, defendant lacks moral responsibility and culpability for their crime, and therefore shouldn't be punished.

Young v State, Ct of App of MD,1985 Author: Smiles Facts: D was found guilty of attempted robbery of a bank. In the early after, a police team observed D driving an auto in such manner as to give rise to a reasonable belief that D was casing several banks. D manifestly endeavored to conceal his presence by parking behind the bank, which he apparently selected to rob. He disguised himself w/ an eye patch and made an identification of him by turning up his collar and by donning sunglasses and a knit cap, which he pulled over his forehead. He put on rubber surgical gloves, clipped his belt w/ a scanner w/ police bank frequency, Except for the scanner, which he had placed on his belt while casing the bank, all this was done immediately before he left his car. As the D approached the bank he partially hid his face w/ his left hand, and his right hand was in his jacket pocket, in which he was carrying a loaded handgun. D walked to the bank door to enter and discovered that the bank was closed and the doors locked. He ran to his car and immediately drove away. State argues that it is more than legally sufficient to establish beyond a reasonable doubt that the D had a specific intent to commit the crime. D argues that evidence is not compelling. Issue: Whether the conduct leading to the D's apprehension established that he performed the necessary overt act towards the commission of armed robbery?

Ct's analysis: Yes, using a "substantial step" as the criterion in determining whether an overt act is more than mere preparation to commit a crime. Rule: A person is guilty of an attempt to commit a crime if, acting w/ the kind of culpability otherwise requires for commission of the crime, he purposely does or omits to do anything which, under the circumstances as he believes them to be, is an act or omission constituting a substantial step in a course of conduct planned to culminate in his commission of the crime. He asked how much time can you get for attempted robbery???.

20. IMPOSSIBILITY (MPC & COMMON LAW)

D could have accomplished the offense, but for some lucky turn of events. Even though D has attempted to achieve the crime, the ultimate commission of the crime is impossible. It is almost never a defense because the defendant has already demonstrated his dangerousness. Common Law Rule (Minority Rule): Legal impossibility is a defense while factual impossibility is not. - There is no principled distinction between the two in practice and ultimately just allows courts to apply the doctrine at their discretion to sympathetic defendants. o Allows courts to assess level of continued dangerousness. MPC Rule (Majority Rule): Liability rests on the facts as D perceived them when D decided to act, not as they actually were. Abandons factual/legal distinction. - Exceptions: (1) de minimus infractions; (2) mitigation; and (3) legality New York v. Dlugash: Companion shot man and five minutes later, the defendant shot the body. He claimed he thought the man was dead. - Rule (application of MPC): Liability rests on the facts as the defendant believed them to be when he acted. Whether the body was actually dead is irrelevant. - Prosecution had to frame it so the defendant had the knowledge or purpose to kill to satisfy attempt under the MPC. Extreme indifference wouldn't have been enough. - If D believed that V was alive, then D is guilty of attempted murder, regardless of whether V was actively alive. MPC eliminates defense of impossibility in almost all situations.

ISSUE: How far must a suspect's actions go in order to constitute the crime of attempted robbery?

Defendant, wearing a poncho that only partially concealed his rifle, approached the entrance to a liquor store, but turned away without making entry when a customer approached him. Held: "Approaching the liquor store with a rifle and attempting to hide on the pathway immediately adjacent to the liquor store when observed by a customer is, in the opinion of this court, a sufficient direct act toward the accomplishment of the robbery." Conviction affirmed. People v. Vizcarra (1980) 110 Cal.App.3d 858, 862.

what are the Five purposes of Criminal Law

Defining criminal behavior excusing criminal behavior classifying and grading criminal behavior penalizing criminal behavior preventing future criminal behavior

Concurrence

D must have mental state at same time he engages in act Penal code section 20: in every crime or public offens there must exsist a union ot joint operation of act and intent or criminal negligence. If you don't have intent to steal while you enter and then you steal that's theft.

How many justices are in the California Supreme Court? a. 5 b. 9 c. 10 d. 7

D) Seven

Which one is MOST TRUE regarding an infraction? a. you do not have the right to a public defense attorney b. It cannot be punished by imprisonment c. You are entitled to a trial by jury d. All of the above

D) all of the above

Disorders Evident in Infancy, Childhood, or Adolescence

Disorders characterized by early onset, including mental retardation, attention-deficit disorder, hyperactivity, anorexia, bulimia, stuttering, sleepwalking, and bed-wetting

Entrapment

Entrapment is a defense designed to prevent police officers or other government agents from enticing persons to commit crimes in order to later prosecute them for those crimes. In the typical entrapment case, an undercover agent suggests that a crime be committed and somehow pressures or induces an individual to commit it. The agent then arrests the individual for the crime. For entrapment to be considered a defense, both the suggestion and the inducement must take place. The defense is not intended to prevent law enforcement agents from setting a trap for an unwary criminal. Rather, its purpose is to prevent them from pushing the individual into a criminal act. The crucial issue is whether the person who committed a crime was predisposed to commit the illegal act or did so only because the agent induced it.

The Criminal Act

Every criminal statute prohibits certain behavior. Most crimes require an act of commission—that is, a person must do something in order to be accused of a crime. In criminal law, a prohibited act is referred to as the actus reus, or guilty act. In some instances, an act of omission can be a crime, but only when a person has a legal duty to perform the omitted act, such as filing a tax return. The guilty act requirement is based on one of the premises of criminal law—that a person should be punished for harm done to society. For a crime to exist, the guilty act must cause some harm to a person or to property. Thinking about killing someone or about stealing a car may be morally wrong, but the thoughts do no harm until they are translated into action. Of course, a person can be punished for attempting murder or robbery, but normally only if he or she has taken substantial steps toward the criminal objective. Additionally, the person must have specifically intended to commit the crime to be convicted of an attempt.

Mistake

Everyone has heard the saying "Ignorance of the law is no excuse." Ordinarily, ignorance of the law or a mistaken idea about what the law requires is not a valid defense. A mistake of fact, however, as opposed to a mistake of law, can excuse criminal responsibility if it negates the mental state necessary to commit a crime.

A "public place" is any place such as an office building, movie house, department store, etc., generally open to the public

False

If two or more persons assemble together and advance toward the commission of an act which would be a riot if committed, such is legally an unlawful assembly

False

One can be guilty of prowling only if the act was done at night under PC 647(h)

False

PC 647(g) requires an officer to place one who is found under the influence of drugs in a public place in civil protective custody

False

Ex Pasto Facto

Formulated enacted, or operating retrospectively. Literally, "after the fact." As prohibited by the Constitution, no punitive law may be applied to acts committed before the law was enacted and effective or means "after the deed" An ex post facto law is one that (1) makes an action done before the passing of the law (and which was legal when done) criminal and punishes such action; (2) aggravates a crime, or make it more serious than it was when committed; (3) inflicts a greater punishment than existent law sis at the time the crime was committed; or (4) alters the legal rules of evidence and requires less, or different, testimony than the law required at the time of the offense in order to convict the offender (NO Retro Active LAW MAKING)

7. OMISSION

GENERAL PRINCIPLE: D not criminally liable for omission or failure to act unless law also imposes on D a duty to act in that precise factual context. FAILURE/DUTY RULE: Failure to act in face of legal duty—an "omission"—can count as necessary "act" required to support a criminal conviction, as long as that omission produces criminal harm such as a death, a serious injury, or serious financial loss. COMMON LAW LEGAL DUTIES: a. Personal Relationship (spouse to spouse, parent to minor child) (Kuntz) b. Creation of Peril (when A endangers B, A has a duty to rescue B) (Kuntz) c. Contract (e.g. implied contract between babysitter and child) d. Voluntary Assistance or Assumption of Care e. Control Conduct of Others f. Landowner Status 1. RULE: When Victim justifiably uses force to fend off Aggressor, Victim has NO DUTY to assist Aggressor in any manner that may conceivably create risk of bodily injury or death to Victim, or to others. 7-6 A limitation on spousal duty. a. SUB-RULE: Duty to summon aid may in fact be revived only after Victim of Aggressor has fully exercised Victim's right to seek and secure safety from personal harm. 2. RULE of Mutual Reliance: Two people who live together, despite lack of close relationship, have a duty to summon medical care. 3. RULE: Person must have knowledge of the facts indicating a duty to act, and that Person was physically capable of performing the act. (CHECK) 4. RULE OF OMISSION: cited as dicta in Kuntz case: "When a Person places another in a position of danger, and then fails to safeguard or recuse that person, and the Person subsequently dies as a result of this omission, such an omission may be sufficient to support criminal liability. This duty may include peril resulting from a defendant's criminal negligence."

Facts: At an intersection in Las Vegas, Defendant approached a disguised police officer, the Decoy. Decoy pretended to be intoxicated and asleep. He had a ten dollar bill noticeably sticking out of his pocket. Defendant attempted to wake the decoy and move him from the street, trying to warn him of possibly getting arrested. Until this point, defendant did not display and predisposition to commit larceny. When Defendant saw the ten dollar bill, he reached down and grabbed it, saying "Thanks, home boy". He was then apprehended Issue: Was the defendant a victim of entrapment, meaning that the officer employed extraordinary temptations and inducements to persuade the defendant to commit the crime?

Holding: Yes. The defendant approached the officer with the intent to aid him, not to commit larceny. Due to an extraordinary temptation, created by the officer, the defendant was entrapped

sine quanon "but for" rule "had not"

Holds in affect that "without this, that would not be," or "but for the conduct of the accused, the harm in question would not have occurred"

14. VOLUNTARY MANSLAUGHTER

Homicide that would otherwise constitute murder may be reduced to voluntary manslaughter when the following criteria are satisfied: (1) the homicide was performed in a 'heat of passion' caused by sudden provocation, (2) the provocation was adequate in the eyes of the law to create a resentment likely to cause violence in a reasonable person - adequacy measured by reference to a set standard of categories that included (a) the defendant's suffering an aggravated assault, (b) the defendant's being engaged in "mutual combat" (such as a barroom brawl or other such manly confrontation), (c) the criminal victimization of a close relative, (d) the defendant's suffering (or perhaps observing) an illegal arrest, or (e) the defendant's personal observation of his wife's adulterous act, and (3) no period of cooling time has transpired such that a reasonable person's passion would have subsided.

Insanity

In California, the test for determining whether a defendant is legally insane is known as the M'Naghten test. Essentially a test of "right and wrong," the M'Naghten standard holds that if the defense can prove that the defendant committed the crime only because he/she didn't understand the nature of his/her act, OR couldn't distinguish right from wrong,

Steps in Criminal Case: Indictment or Information

Individuals must be formally charged with having committed specific crimes before they can be brought to trial. If issued by a grand jury such a charge is called an indictment. A grand jury does not determine the guilt or innocence of an accused party. Rather, its function is to hear the state's evidence and to determine whether a reasonable basis (probable cause) exists for believing that a crime has been committed and that a trial ought to be held. Usually, grand juries are called in cases involving serious crimes, such as murder. For lesser crimes, an individual may be formally charged with a crime by an information, or criminal complaint. An information will be issued by a government prosecutor if the prosecutor determines that there is sufficient evidence to justify bringing the individual to trial.

Persons under retraining orders

Individuals who are subject to temporary restraining orders or injunctions issued for a specified circumstance are prohibited from purchasing or receiving a firearm. Specified circumstances include restraining orders: to protect a person from harassment (Code of Civil Procedure 527.6) involving domestic violence (Family Code Section 6211) on behalf of an employee who has received threats or has been a target of violence (Code of Civil Procedure 527.8) Individuals under such restrictions who are found to have purchased or received, or attempted to purchase or receive, a firearm may be guilty of a FELONY. (PC 29825)

4 reasons why progressive era occurred when it did

Industrial age, growth of middle class, fear of radicalism, scientific authority

-ENTRAPMENT—Objective test

Law enforcement officers may provide opportunity for the commission of crime and extend their apparent cooperation for the purpose of detecting the offender. If the suspect, originally and independently of the officers had the intent, whenever the opportunity arose, to commit the acts constituting the crime charged, and if he or she does acts necessary to constitute the crime, he or her is guilty of the crime committed. He or she has no defense in the fact that officers engaged in detecting crime were present and provided the opportunity, or aided or encouraged the commission of the offense. applies to situations where you would not have committed the alleged offense but for for the harassment, threats, or coercion of the police or their agents. If you were entrapped, it basically means that the police... who were most likely undercover... persuaded you to commit a crime

32. NON CONSENT AND MENS REA

MENS REA Majority Rule: Rape a general intent offense. If the defendant reasonably believed the victim consented, he is not guilty of rape. General intent favors victims and encourages individuals to proceed with caution in determining whether a partner has voluntarily consented. Minority Rule: Rape is a strict liability offense. If the victim did not consent, it does not matter how reasonable the belief in consent was. In practice, it operates as a negligence standard since the conduct requires reasonable belief. MPC and Alaska: A defendant is guilty of rape only if he consciously disregards a substantial and unjustifiable risk of non-consent. Rationale: Allowing for specific intent would excuse intoxicated individuals who honestly believed the victim consented. General intent also better matches the conduct requirement - there is no point in requiring more culpability for intent when the defendant must act reasonably with respect to conduct. Marital Exemption: For the most part, it is no longer in effect because the rationale is outdated. However, it is still in play in some jurisdictions. Statutory Rape: Shift away from protecting chastity to preventing the exploitation of the young by those of greater age and sophistication. Looks at the power dynamic that exists between the parties. In most jurisdictions, it remains a strict liability offense, but some allow for a reasonable mistake of fact. Connecticut v. Smith (V resisted, but submitted after D threatened her) Conn. 1989 a. HOLDING: Rape should be (and is) a general intent crime. If rape were specific intent, then intoxication would function as a valid defense for rape crimes. Effectiveness of rape statute would then be undermined since alcohol contributes to so many sexual assaults.

Damage Awards in Civil Cases

May be both compensatory - in which the amount to be paid directly compensates the injured party for the amount of damage incurred - and punitive - in which the award serves to punish the defendant for some especially wrongful or treacherous act

Mayhem

Mayhem- every person who unlawfully maliciously derives a human being of a member of his body, or disables, disfigures, or renders it useless, or cuts or disables the tongue, or puts out an eye or slits the nose, ear, or lip, is utility of mayhem - penal code 203

4 recognized mental states

Mens rea- action before, during and after crime - General intent crime o Assalt w. firearm - Specific intent crime o It will say specific intent in the staute. (with the intent to commit rape) - Transferred intent crime o Intnent to harm, kill one person and accidently kill another person. - Constructive intent o No intention to injur but the action is dangerous and reckless (DUI manslaughter)

4. STATUTORY INTERPRETATION AND THE RULE OF LENITY METHODS (6) Common Law Majority/Minority Rule/MPC

Methods (6): 1. Original Intent/Meaning 2. Relying on common law definitions and doctrines. 3. Examining Legislative History 4. Harmonizing statute with other law (broader or narrower) 5. Updating statutory meaning (apply as society changes) 6.Rule of Lenity/Strict Construction: apply statute in light most favorable to D. *Must also follow stare decisis, not free to contradict statutory interpretation of higher courts. RULE OF LENITY: When Court has done its best to determine the meaning of statute using the above tools, but still finds meaning uncertain, Court should adopt interpretation that is more favorable to D. Keeler, McBoyle case. Sobiek Rule: Minority Rule (Rejection of Lenity): Though uncertainty exists, statute should be read in favor of State because common sense would have forewarned D about D's actions' illegality. Model Penal Code: rejects the rule of lenity, leaving many courts to use Rule as a tiebreaker since lenity is a restriction of judicial power to create common law crimes.

Offenses

Minor Violations-- Jaywalking, violating a building code, etc. Usually punished with a fine-- however, a guilty party can be put in jail for a few days, fined, or both, depending on state or local law. Whether a crime is a felony or a misdemeanor can determine in which court the case is tried and, in some states, whether the defendant has a right to a jury trial.

Guy Kills wife by choking becasue he lost temper becasue she didnt want to have sex mom was sleeping but she awaoke and became aware that her son was dismembering the wife in other room. and she washed his clothes after

Mom is a accessory Her actions were AFTER the murder, mom was asleep during the killing When you are aiding and abetting you are responsible for the natural improbable consequences from your actions of aiding and abetting

Physical Proximity Test

Once the accused has power to complete the crime almost instantly, the attempt is made. Here the victim has no chance of escape and is completely under the control of the accused A test traditionally used under common law to determine whether a person was guilty of attempted criminal activity. The physical proximity test requires that the accused has it within his or her power to complete the crime almost immediately Defendant is culpable if completion of the crime is within grasp, although the defendant's act need not be the penultimate act

Problems with Syndrome-Based Defense Enhancements

One of the central problems with all defenses based on syndromes is that there is no syndrome that includes homicide, or any other law-breaking behavior, as a symptom of or as an inevitable result of the syndrome. In other words, according to most specialists, most people suffering from syndromes are as capable of controlling their behavior as anyone else.54 In fact, most do. Speaking loosely, for every 100 people afflicted with some "syndrome," 99 do not violate the criminal law, and far fewer kill Another problem with syndrome-like excuses is that, in a fundamental sense, they seem to rely on a tactic of blame shifting, in which the victim of crime is made to seem less like a victim and more like a criminal in hopes that a jury will side with the defendant. Blame shifting attempts to convince the jury that the victim "had it coming" and builds on such notions as a history of abuse that can actually justify homicide.

Federal System of Government

One which two governments have jurisdiction over the inhabitants

Intoxication

Our law provides that "no act committed by a person while in a state of voluntary intoxication is less criminal by reason of his having been in such condition." This provision of the law means that if the evidence shows that the defendant was voluntarily intoxicated when allegedly he committed the offense charged, his intoxication is not a defense to such charge.

1. PURPOSES OF PUNISHMENT

Purposes of Punishment: 1. Consequentialism:. Purposes are: a. Deterrence. 1. Special Deterrence: laws ability to deter a particular individual or group of individuals subject to criminal penalties. 2. General Deterrence: laws ability to produce adherence among the general public. a. Intimidation of Society in General b. Moral Education b. Incapacitation: put them in jail. c. Rehabilitation: In 1970s, notion that people cannot be rehabilitated caught hold. Today, generally, if one gets rehabilitated, then one gets sense to another place outside jail. 2. Retributivism: calls for punishment delivered by the state according to the individual's moraL wrongs, nothing more.

30. RAPE: THE TRADITIONAL RULES

Rape is a general intent crime requiring sexual intercourse (1) achieved without consent and (2) accomplished by force or threat of force. - Non-consent and resistance are often interrelated because the presence of one can often imply the presence of the other. - When it comes to implementing rape statutes, actus reus and mens rea morph into the same thing, and force and non-consent morph into the same thing. The critical question is the victim's reasonable fear. - Rape is a circumstance crime. - Submission does not amount to consent. Categories of Rape: 1. Physical force used or threatened, resulting in submission and intercourse - rape. 2. Extortion or coercion (threat of unpleasant choice) resulting in submission and intercourse - depends on the facts and jurisdiction. 3. Fraudulently obtained consent - fraud in the fact is rape, but fraud in the inducement is not. THREAT OF PHYSICAL FORCE The force element of rape implies a resistance requirement. This in turn puts the burden on the woman to resist as opposed to the man to obtain consent before intercourse occurs. Resistance is not required if the victim believes she could not do so safely. Threat of force does not require physical violence; implicit threats are sufficient. The victim's fear of force must be objectively reasonable. Maryland v. Rusk: The defendant took the victim's car keys after she gave him a ride home. The victim was in an unfamiliar neighborhood late at night. The victim was afraid to resist. - Rule: If a defendant's acts or threats create a reasonable fear of imminent bodily harm in the victim serious enough to overcome the victim's ability to resist, then those acts or threats satisfy the force requirement. o Submission to intercourse in the presence of a use or threat of force is not consent. o The same type of evidence may be used in establishing both force and non-consent. In re MTS: Two teenagers were engaged in heavy petting, but penetration occurred without consent. - Affirmative Consent Rule: Any act of sexual penetration engaged in by a defendant without the victim's consent constitutes sexual assault because physical force inherent in penetration is sufficient to satisfy force requirement. o Consent is demonstrated when the evidence shows that a reasonable person would have believed that the victim affirmatively and freely consented to intercourse. Conduct is sufficient to establish consent. Pennsylvania v. Berkowitz: The defendant put the moves on the victim, who said no but failed to leave the room or resist in any other way. - Degrees of Non-Consent: Though saying "no" can establish a victim's refusal to consent, it is not a sufficient degree of non-consent that can imply the defendant's use or threat of force.

26. SELF DEFENSE (JUSTIFICATION)

SELF-DEFENSE (JUSTIFICATION) D may be justified in using deadly force in self-defense when D: (1) objectively or subjectively believes; (2) such force is necessary and proportionate; (3) to prevent a sufficiently imminent harm; (4) with or without imperfect justification. - Self-defense is a subset of necessity. Necessity requires that the defendant cannot avoid the harm by any lesser means, which requires imminence, lack of legal alternative, or in some jurisdictions, a duty to retreat. - All jurisdictions require that the defendant subjectively believe that the use of deadly force was necessary and proportional, but most also require an objective standard that the conditions were met. Many jurisdictions allow for imperfect justification. o Objective standards are often highly subjectivized in self-defense cases. o Imperfect justification: If the defendant was unreasonable in his belief, his defense is knocked down a grade. Mistake of fact in self-defense cases always relates to the attendant circumstances. - If the defendant starts the confrontation, or is sufficiently at fault for starting the confrontation, then the defendant loses self-defense. - If the defendant is mistaken as to the law (i.e. sufficient triggers), there is no self-defense justification. SEE CHART New York v. Goetz: Man believed he was being robbed on a New York subway by a group of youths. He shot all of them in self-defense, causing severe harm to all of them. - Rule: A genuine but unreasonable mistake as to the need for self-defense results in an (attempted) murder conviction. The objective standard can be subjectivized to allow for the consideration of the defendant's experiences. Ohio v. Thomas: Battered woman shot her common law husband after he approached her during an altercation. - Rule: Where the standard is wholly subjective, any genuine belief in the need for force will result in an acquittal.

3. VAGUENESS

Statutes must be sufficiently clear in meaning to avoid 5th and 14th Amendment Due Process issues. Overly vague statues are no different from having no statute at all. "Void for vagueness doctrine requires that a penal statute define the criminal offense with sufficient definiteness that ordinary people can understand what conduct is prohibited and in a manner that does not encourage arbitrary and discriminatory enforcement." Kolender v. Lawson (1983) Kolender v. Lawson: statute cannot apply to anyone under any circumstance, or, it can be applied to so many people, it might be better to strike down altogether. RULE, MAJORITY (O'CONNOR): Any law that is so vague that it permits standardless sweeps violates due process facially. Criminal statutes must articulate "with sufficient particularity what a suspect must do to satisfy the statute. RULE, DISSENT (WHITE): Statutes are not void for vagueness unless it is vague in all of its possible applications. If there is a range of conduct that is clearly within the reach of the statute, law enforcement personnel, as well as putative arrestees, are clearly on notice that arrests or such conduct are authorized by law. Papachristou v. City of Jacksonville: RULE: a statute is void for vagueness when it 1. fails to give a person of ordinary intelligence fair notice that his contemplated conduct is forbidden by the statute, and 2. because it encourages arbitrary and erratic arrests and convictions. US v. Petrillo, 1947: coercion to employ more radio employees, violates statute. RULE: a statute is void for vagueness when its terms are so vague, indefinite, and uncertain that a reasonable person cannot understand them. US v. Nash: Leonard loves this case. RULE: Since crime is a matter of degree, many laws are intentionally vague. Therefore, specificity of the language is not the primary reason for finding a statute sufficiently vague, especially if there is no better way to write the language in a way that better serves the purpose. Idea of vagueness law is "notice", that is, were parties sufficiently on notice?

Difference Between Civil Law and Common Law

The common law tradition is evolutionary in nature. Its change came incrementally, as courts adapted the law to different facts or social changes. The civil law tradition, on the other hand, is planned and statutory. Civil law is conceived of by some authority, is established, and then changes abruptly when the legislative authority revises the code

The Difference Between The Crime-Control Pole of The Continuum and The Due Process Model

The crime-control pole of the continuum is characterized by its emphasis on preventing, detecting, and prosecuting criminals. Efficiency is a value of the system. The due process pole emphasizes the preservation of individual liberties over the rights of the community to be free of crime or to have an efficient system. The crime-control model is satisfied when crimes are prevented or a crime is detected and the factual guilt of an offender is found in an efficient manner. The due process model, on the other hand, emphasizes legal guilt. Legal guilt incorporates factual guilt, but it also includes a fair process. A defendant is legally guilty only if her or she is factually guilty and the government respected the defendant's rights while investigating and prosecuting the case

Criminal Liability

The degree of blameworthiness assigned to a defendant by criminal court and the concomitant extent to which the defendant is subject to penalties prescribed by the criminal law

Accomplice Liability

The degree of criminal blameworthiness of one who aids, abets, encourages, or assists another person in the commission of a crime

Feature Distinguishing Between One Type of A Crime From Another

The degree of punishment

express malice

The deliberate intention to kill another person

General Intent

The form of intent that can be assumed from the defendant's behavior. General intent refers to an actor's physical conduct Such as involuntary manslaughter or negligent homicide. Also, Battery is a general intent offense. If a defendant commits a battery that results in harm to the victim, it does not matter if the defendant did not intend the harm. Suppose a defendant struck a victim in the leg with an axe. The defendant intended to frighten the victim into complying with his request to not trespass on his property. Later, the victim died from a resulting infection. At common law, the defendant committed a general-intent crime

DSM-IV

The fourth edition of the Diagnostic and Statistical Manual of Mental Disorders, published by the American Psychiatric Association.vi The DSM-IV lists 12 major categories of mental disorders

Forgery

The fraudulent making or altering of any writing (including electronic records) in a way that changes the legal rights and liabilities of another is forgery. Without authorization, Severson signs Bennett's name to the back of a check made out to Bennett and attempts to cash it. Severson is committing forgery. Forgery also includes changing trademarks, falsifying public records, counterfeiting, and altering a legal document.

PC 32415: Loan of larger-capacity magazine

The loan of the larger-capacity magazine occurs at a place or location where the possession of the large capacity magazine is not otherwise prohibited and the person who lends the larger capacity magazine remains in the accessible vicinity of the person to whom the large capacity magazine is loaned

Plurality Requirement

The logical and legal requirement that a conspiracy involve two or more parties

Property Crime

The most common type of criminal activity is property crime, in which the goal of the offender is some form of economic gain or the damaging of property. Robbery is a form of property crime, as well as a violent crime, because the offender seeks to gain the property of another. We look here at a number of other crimes that fall within the general category of property crime. (Note also that many types of cyber crime, discussed later in this chapter, are forms of property crime as well.)

D met Alvarez at 2:30 AM, and they agree to a drag race. Upon completing the course, Alvarez turned around sharply and started to do the course backwards. D followed. When they reached the end, they could not stop. D (123 mph) flew over a canal and got crushed to death on the other side. D landed in the canal and swam to safety. What are the two tests for causation? Should a D be legally liable for all actions that he was the cause-in-fact for?

The most commonly used test is the "but-for" test. A D's conduct is a cause-in-fact of the prohibited result if the said result would not have occurred but for the D's conduct. In rare cases, the "but for" test fails, and the court must use the "substantial factor" test. (two separate acts, each alone is sufficient to bring about the prohibited result. A D's conduct is a cause-in-fact of a prohibited result if the subject conduct was a "substantial factor" in bringing about the said result. Even where a D's conduct is a cause-in-fact of a prohibited result, courts may decline to impose criminal liability where either... The prohibited result of the D's conduct is beyond the scope of any fair assessment of the danger created by the D's conduct It would otherwise be unjust, based on fairness and policy considerations, to hold the D's criminally responsible for the prohibited result.

Dangerous Proximity Test

The nearness of completion, the degree of intended harm, and the victim's apprehension are all factors of this test. With an objective of murder and only two steps remaining, an attempt is made A test for assessing attempts, under which a person is guilty of an attempt when his or her conduct comes dangerously close to success Defendant must act in dangerous proximity to the crime to be culpable. Acts remotely connected to the crime are not adequate

Proximate Cause

The primary or moving cause that plays a substantial part in bringing about injury or damage. It may be a first cause that sets in motion a string of events whose ultimate outcome is reasonably foreseeable If, for example, a women poisons her husband's dinner, intending to kill him, bu the stays lat eat the office and she put the meal in the refrigerator, she may still be held liable for the crime of homicide if a boarder staying in the house gets up in the middle of the night, eats the meal, and dies NOT proximate cause: One person assaults another and chases him out of a building and through a driving rainstorm, the assault can not be said to be the proximate cause of the victim's death if he is struck by lightning (the primary cause of death) and killed during the pursuit, even though one might argue that but for the initial assault, he would not have been exposed to the elements and would therefore have lived. This so because the lightning strike was not related to the assault, that is, it was not brought on by it in the same sense that an infection might be produced by a gunshot wound

Money Laundering

The profits from organized crime and illegal activities amount to billions of dollars a year. These profits come from illegal drug transactions and, to a lesser extent, from racketeering, prostitution, and gambling. Under federal law, banks, savings and loan associations, and other financial institutions are required to report currency transactions involving more than $10,000. Consequently, those who engage in illegal activities face difficulties in depositing their cash profits from illegal transactions. As an alternative to storing cash from illegal transactions in a safe-deposit box, wrongdoers and racketeers launder "dirty" money through legitimate business to make it "clean." Money laundering is engaging in financial transactions to conceal the identity, source, or destination of illegally gained funds.

Criminal Sanctions

The sanctions imposed on criminal wrongdoers are also harsher than those applied in civil cases. As we will see in Chapter 12, the purpose of tort law is to enable a person harmed by a wrongful act to obtain compensation from the wrongdoer, rather than to punish the wrongdoer. In contrast, criminal sanctions are designed to punish those who commit crimes and to deter others from committing similar acts in the future. Criminal sanctions include fines as well as the much harsher penalty of the loss of one's liberty by incarceration in a jail or prison. Most criminal sanctions also involve probation and sometimes require performance of community service, completion of an educational or treatment program, or payment of restitution. The harshest criminal sanction is, of course, the death penalty.

Probable Cause

To obtain a search warrant, law enforcement officers must convince a judge that they have reasonable grounds, or probable cause, to believe a search will reveal a specific iIllegality. Probable cause requires the officers to have trustworthy evidence that would convince a reasonable person that the proposed search or seizure is more likely justified than not.

What is "in pro. per."?

To prosecute one's case by oneself. From "in propria persona," meaning "in his or her own person" (rather than by an agent or attorney).

Service-Based Hacking

Today, many companies offer "software as a service." Instead of buying software to install on a computer, the user connects to Web-based software. The user can write e-mails, edit spreadsheets, or perform other tasks using his or her Web browser. Cyber criminals have adapted this distribution method to provide "crimeware as a service." A would-be thief no longer has to be a computer hacker to create a botnet or steal banking information and credit-card numbers. He or she can rent the online services of cyber criminals to do the work for a small price. Fake security software (also known as scareware) is a common example. The thief can even target individual groups, such as U.S. physicians or British attorneys.

How Can Legal Causes be Distinguished From The Causes That May Have Produce The Result In Question But That May Not Provide The Basis For a Criminal Prosecution?

Too Complex Too Indistinguishable From Other Causes Not Knowable Not Provable in a Court of Law

24. DERIVATIVE LIABILITY OR ITS ABSENCE

Traditional Common Law (Minority) Rule: Accomplice's liability derives from the principal. Accomplice cannot be convicted if principal has not been convicted, nor can accomplice be convicted of a higher offense than the principal. - If the principal gets off, then prosecution cannot reach the accomplice unless the prosecution can demonstrate that the accomplice is a principal in the first degree by establishing the accomplice's (1) actus reus, (2) mens rea, and (3) causation (accomplice's aid caused principal's conduct). New Common Law (Majority) Rule: Accomplice is liable if he has mens rea for the doing of the actus reus, and the other actor commits the necessary actus reus. Liability does not depend on whether the other actor possessed requisite mens rea. MPC Rule: MPC expressly rejects accomplice liability's dependence on principal's conviction. - If a defendant attempts to aid another, doing all conduct necessary for complicity, but the other actor never actually commits or attempts the crime, the defendant is still guilty of an attempt to commit the crime if he satisfies the substantial step analysis under attempt doctrine. Regina v. Cogan & Leak: Husband convinced his friend to rape his wife. His friend believed the wife was consenting and so found innocent of rape. - Rule: An accomplice can be guilty of an offense, even though the other actor is not convicted, if the accomplice has the necessary mens rea for the conduct of the other actor. - Contrast with MPC: Even though husband cannot legally rape his wife because they are married, MPC holds husband guilty of rape because the husband is legally accountable for the rapist's conduct via "innocent" agency, conspiracy, or complicity. Regina v. Richards: Defendant convinces other men to assault her husband badly enough to put him in the hospital, but they only succeed in wounding him superficially. - Traditional Rule: An accomplice cannot be convicted of a higher offense than the principal actor when that accomplice is not present at the crime and the accomplice's intent is remote to the crime. Dusenberry v. Virgina: Defendant physically assisted in commission of other actor's consensual sex. - Traditional Rule: Before a defendant can be convicted as an accomplice, prosecution must show that the other actor committed the crime. If a principal and a girl are consenting, the principal does not have the conduct necessary for rape, so the defendant cannot be an accomplice. EXCEPTIONS A defendant is not an accomplice if: 1. The defendant is the victim of the offense. 2. The offense is defined such that the defendant's conduct is inevitably incident to its commission (e.g. buying drugs). 3. The defendant abandons his complicity and: (1) wholly deprives the effectiveness of offense's commission, or (2) gives timely warning to law enforcement or otherwise makes proper efforts to prevent the offense's commission.

12. MURDER (COMMON LAW)

Traditional Common Law Murder - Four Categories (1) Intentional Homicide - Homicides in which the defendant intended to kill another or knew that another's death would result (2) Grievous Bodily Injury - Homicides in which the defendant intended to inflict grievous bodily injury or knew that such injury would result (3) Extreme Recklessness or Depraved Heart - Homicides in which the defendant manifested extreme recklessness with respect to the life of another, thereby displaying a "depraved heart" (a) Limited to "human life" and not to grievous bodily injury (4) Felony Murder - Homicides resulting while the defendant was committing a felony ____________________________________________ Subjectively, (a) Actual intent or knowledge to kill, (b) intention or knowledge to inflict grievous bodily injury (GBI), or (c) extreme recklessness demonstrating a depraved heart ____________________________________________ Objectively, knowledge of such facts such that from which a reasonable person would infer a high risk of grievous bodily injury or death (Smith) ____________________________________________ OLD PA/TN LAW: First Degree: Subjective intent to kill (I/K re: death) ____________________________________________ Second Degree: All other CL murder (Brown) ____________________________________________ NEW TN/CA LAW: First Degree: Deliberation and premeditation (actual advanced planning) ____________________________________________ Second Degree: All other CL murder (Brown)

Burglary

Traditionally, burglary was defined as breaking and entering the dwelling of another at night with the intent to commit a felony. This definition was aimed at protecting an individual's home and its occupants. Most state statutes have eliminated some of the requirements found in the common law definition. The time of day at which the breaking and entering occurs, for example, is usually immaterial. State statutes frequently omit the element of breaking, and some states do not require that the building be a dwelling. When a deadly weapon is used in a burglary, the perpetrator can be charged with aggravated burglary and punished more severely.

The Exclusionary Rule

Under what is known as the exclusionary rule, any evidence obtained in violation of the constitutional rights spelled out in the Fourth, Fifth, and Sixth Amendments generally is not admissible at trial. All evidence derived from the illegally obtained evidence is known as the "fruit of the poisonous tree," and such evidence normally must also be excluded from the trial proceedings. For instance, if a confession is obtained after an illegal arrest, the arrest is the "poisonous tree," and the confession, if "tainted" by the arrest, is the "fruit." The purpose of the exclusionary rule is to deter police from conducting warrantless searches and engaging in other misconduct. The rule can sometimes lead to injustice, however. If the evidence of a defendant's guilt was obtained improperly (without a valid search warrant, for instance), it normally cannot be used against the defendant in court.

PC 20810: Blade weapons: Undetectable knife

Undetectable knife or other instrument, with or without a hand-guard, that satisfies all of the following requirements: • It is capable of ready use as a stabbing weapon that may inflict great bodily injury or death • It is commercially manufactured to be used as a weapon • It is not detectable by a metal detector or magnetometer, either handheld or otherwise, which is set at standard calibration

What are the two main sovereign jurisdictions in the United States?

United States federal government Each individual state

Types of Crimes

Violent Crime: --Definition—Crime that causes others to suffer harm or death. --Examples—Murder, assault and battery, sexual assault (rape), and robbery. Property Crime: --Definition—Crime in which the goal of the offender is some form of economic gain or the damaging of property; the most common form of crime. --Examples—Burglary, larceny, arson, receiving stolen goods, forgery, and obtaining goods by false pretenses. Public Order Crime: --Definition—Crime that is contrary to public values and morals. --Examples—Public drunkenness, prostitution, gambling, and illegal drug use. White-Collar Crime: --Definition—An illegal act or series of acts committed by an individual or business entity using some nonviolent means to obtain a personal or business advantage; usually committed in the course of a legitimate occupation. --Examples—Embezzlement, mail and wire fraud, bribery, bankruptcy fraud, insider trading, and the theft of intellectual property. Organized Crime: --Definition—A form of crime conducted by groups operating illegitimately to satisfy the public's demand for illegal goods and services (such as gambling and illegal narcotics). Money laundering—Passing "dirty" money (obtained through criminal activities, such as illegal drug trafficking) through legitimate enterprises so as to "launder" it (make it appear to be legitimate income). RICO—The Racketeer Influenced and Corrupt Organizations Act (RICO) makes it a federal crime to use income obtained from racketeering activity to purchase any interest in an enterprise, acquire or maintain an interest in an enterprise through racketeering activity, conduct or participate in the affairs of an enterprise through racketeering activity, or conspire to do any of the preceding activities. RICO provides for both civil and criminal liability.

You must prove the defendant did the act. You must prove the defendant was capable enough to understand his actions would cause harm.

What must be proven in a general intent case?

You must prove the defendant did the act. You must prove the defendant did the act with a specific goal, purpose, or objective in mind.

What must be proven in a specific intent case?

You must prove the defendant did the act.

What must be proven in strict liability offenses?

How does a Bill become a Statute?

When a Bill is approved by both houses and signed into law by the President (federal bill) or the governor (state bill).

Embezzlement

When a person who is entrusted with another person's property fraudulently appropriates it, embezzlement occurs. Typically, embezzlement is carried out by an employee who steals funds. Banks are particularly prone to this problem, but embezzlement can occur in any firm. Embezzlement is not larceny because the wrongdoer does not physically take the property from the possession of another, and it is not robbery because no force or fear is used. The intent to return the embezzled property—or its actual return—is not a defense to the crime of embezzlement. Embezzlement occurs whether the embezzler takes the funds directly from the victim or from a third person. If the financial officer of a large corporation pockets checks from third parties that were given to her to deposit into the corporate account, she is embezzling. Frequently, an embezzler takes a relatively small amount at one time but does so repeatedly over a long period. This might be done by underreporting income or deposits and embezzling the remaining amount or by creating fictitious persons or accounts and writing checks to them from the corporate account. Even an employer's failure to remit state withholding taxes that were collected from employee wages can constitute embezzlement.

reasonableness requirement

Whether the defendant subjectively believed that the use of force was necessary

Seven Years Old to Age of Responsibility (Varies by state)

Would be charged as a minor. The violators in this case are referred to as Juvenile Offender. There is a possible defense of infancy and in the conviction is adjudicated delinquent. If it is a serious or habitual offense, the person is charged as an adult and is considered a criminal, there is not a possibility for a defense of infancy (regardless of mentality) and the person is usually convicted as Found Guilty

Range of punishment for law:

a fine to a death penalty, community service, rehab, restitution,

PC 26100(b): Permitting a person to discharge a firearm from a vehicle (Felony)

any driver or owner of a vehicle who knowingly permits any person to discharge any firearm from the vehicle

PC 26100(a): Permitting any person to bring a firearm into a vehicle (Misdemeanor)

any driver or owner of any motor vehicle who permits any person to carry into, or bring into the vehicle a firearm in violation of PC 25850(a) or Fish and Game Code Section 2006

PC 25850(a): Carrying a loaded firearm in a public place or in a vehicle (Misdemeanor)

any person who carries a loaded firearm on one's person or in a vehicle in: a public place, on any public street in an incorporated city, within a prohibited area of an unincorporated area

PC 21310: Dirks and daggers (Felony)

any person who carries concealed upon his or her person any dirk or dagger is guilty of a felony A dirk or dagger is: a knife or other instrument with or without a hand guard capable of ready use as a stabbing weapon that may inflict great bodily injury or death

PC 19100: Concealed explosives (Felony)

any person who carries concealed upon his or her person any explosive substance (other than fixed ammunition) The mere possession of an explosive substance alone is not a crime under Penal Code Section 19100. It must also be shown that the explosive substance was carried concealed on the individual's person.

PC 417(a)(1): Drawing, exhibiting, or unlawful use of a deadly weapon other than a firearm (Misdemeanor)

any person who except in self-defense in the presence of any other person draws or exhibits any deadly weapon in a rude, angry, or threatening manner, unlawfully uses a deadly weapon in any fight or quarrel Actual injury or assault is not required for the crime to be complete

PC 29800(a)(1): Possession of a firearm by a convicted felon narcotic addict or other restricted person (Felony)

any person who has been convicted of a felony or is addicted to any narcotic drug (PC 29800(a)(1)) or two convictions of (PC 29800(a)(2)) or offenses enumerated in PC 23515

PC 21510: Switchblade knife (Misdemeanor)

any person who possesses on one's person in the passenger's or driver's area of any motor vehicle in any public place or place open to the public sells, offers for sale, exposes for sale loans, transfers, or gives away a switchblade knife with a blade two or more inches in length The passenger's or driver's area of a motor vehicle includes an area designed to carry the driver and passengers and any interior compartment spaces (i.e., glove compartment, door compartments, etc.).

PC 26100(c): Discharging a firearm from a motor vehicle at a person other than an occupant of a motor vehicle (Felony)

any person who willfully and maliciously discharges a firearm from a motor vehicle at any person other than an occupant of a motor vehicle

A statute that enforces private right and liabilities, as differentiated from criminal law is called a

cicil law

homicide in self defense the burden is on who

prosecution to prove beyond a reasonable doubt that the homicide was unlawful, not justifiable

Contempt

disregard of court order or disrespect toward court

what is threatening a public officer? (PC71)

intent to cause or attempt to cause a public officer or educational employee from doing or not doing something that is part of their job, by threat communicated directly to that person or their property and the threat must be credible.(felony)

voluntary intoxication

intoxication that is the result of willful personal choice It can be liquor, drugs or other drugs --- is a defense when relevant to specific intent

Unauthorized disclosure of information regarding any patient to a third party may result in.

invasion of privacy

proximate cause of death: guy gets drunk and drove his boat, flip it lost his buddy. buddy found dead. what type of crime is this

invol manslaughter due to negligence causation if proximate cause of death

Self-Incrimination

is guaranteed by a clause in the Fifth Amendment to the U.S. Constitution. The clause reads "nor shall [any person] be compelled in any criminal case to be a witness against himself."

One who aids and abets in the commission of a crime

is not the only guilty of that crime, but is also guilty of any other crime committed by a principle which is a natural and probable consequence of the crime originally aided and abetted.

presumption of consciousness

just like innocence, presumed consciousness until proven otherwise.

An impartial panel established to listen to and investigate patient's complaints about medical care or excessive fees is called a _______ commitlee.

medical grievance

Conceal Define

more than a simple withholding of knowledge possessed by a party that a felony has been committed.

Syndrome-Based Defense Enhancements

personal abnormalities, generally supplement defenses based on excuses rather than those claiming justifications. According to mental health law expert Stephen Morse, "[A]bnormalities are usually excusing conditions that bear on the accused's responsibility, rather than objectively justifying conditions that make otherwise wrongful conduct right under the circumstances." If it can be demonstrated that a person charged with a crime was "suffering" from a known syndrome at the time that the crime was committed, such a showing may lower or eliminate criminal liability in at least three ways: (1) It may help support the applicability of a traditional defense (including a justification); (2) it may expand the applicability of traditional defenses (including justifications) to novel or unusual situations or to situations in which such defenses might not otherwise apply; or (3) it may negate the mens rea needed to prove the offense, leading the jury or trial judge to conclude that a crucial element necessary to prove the crime is missing. Some suggest a fourth possibility, however, and that is "the creation of new affirmative defenses,"42 in which novel and unique defenses, beyond those now recognized by law, would take their place alongside other more traditional defenses, such as duress, mistake, and self-defense.

Tort

private or civil wrong.

Information given by a patient to medical personnel which cannot be disclosed without consent of the person who gave it is

privileged communication

Adjective law

procedural law

abandonment of attempt

reason of voluntarily abandoning his purpose or because he was prevented or interfered with in completion the crime

The health worker is protected by law if it can be determined that he or she acted reasonable as compared with fellow workers this is called

reasonable care

Insanity As a mens rea Defense

refers to the defendant's state of mind at the time the act occurred. Insanity at the time of trial is a different matter

Define: False reporting of a crime (PC148.5)

reporting a crime to law enforcement knowingly is false (misdemeanor)

Define: falsly reporting a destructive device bomb. (148.1)

reporting a destructive device to anyone knowing the report is false (felony)

What must the effective use of the duress defense be based on?

showing that the defendant feared for his or her life or was in danger of great bodily harm—or that the defendant was acting to prevent the death or bodily harm of another.9 Likewise, the threat under which the defendant acted must have been immediate, clear, and inescapable and must not have arisen from some illegal or immoral activity of the defendant. The MPC states that the defense of duress is "unavailable if the actor recklessly placed himself in a situation in which it was probable that he would be subjected to duress.

The time established for filing law suits is

statute of limitations

A writ that commands a witness to apear at a trial or other proceeding and give testimony is a(n)

subpoena

To interrupt or discontinue a suit temporarily with the intention or resumption at a later date, or to ask for a continuance is called a _____.

suspension

PC 245400: Felony Exceptions

the person previously has been convicted of any felony or of any crime made punishable by the provisions listed in Penal Code Section 16850 (PC 25400(c)) • the firearm is stolen and the person knew or had reasonable cause to believe it was stolen (PC 25400(c)(2)) • the person is an active participant in a criminal street gang as defined in (PC 25400(c)(3)) • the person is not in lawful possession of the firearm (PC 25400(c)(4)) • the person has been convicted of a crime against a person or property or of a narcotic or dangerous drug violation (PC 25400(c)(5)) • the concealable firearm and the unexpended ammunition are either in the immediate possession of the person or readily accessible to that person or the firearm is loaded as defined in PC 25850 and the person is not listed with the Department of Justice as the registered owner of the firearm. (PC (25400(c)(6)(A)(B))

3 events causing the great depression

the stock market crash, internal weaknesses in the american economy, the european economy.

Principles:

those who actually commit the crime, AND those who aid and abet the commission of the crime

Accessories:

those who assist AFTER the commission of the crime

One can be a principal to a crime merely by advising and encouraging its commission by another.

true

The test as to whether one is an accessory is whether the latter could be prosecuted as a principal.

true

To be guilty of being an accessory, the accused must have actual knowledge that the principal has committed a felony

true

Define: Rout (PC406)

two or more assembled and acting together, who make an attempt or advance toward the commission of an act that would be a riot if actually committed (misdemeanor) - Attempted Riot

Define: Unlawful assembly? (PC407)

two or more gathered to do an unlawful act or LAWFUL act in a violent, boisterous manner (misdemeaner)

Define: refusal to disperse upon lawful command (PC416(a))

two or more people assemble for the purpose of disturbing the public peace or committing an unlawful act and who do not leave when told to do so by a public officer. (misdemeanor)

Define: Riot (PC404)

two or more persons acting with a common intent to use force or violence or threaten to do so with the immediate power to execute such a threat and the acts disturb the public peace (misdemeanor)

It is _______ to deny treatment to an HIV infected parients.

unethical

Battery

unlawful use of force on another.

• Duty to Retreat

• Theres is no duty to retreat • CALJIC 5.50 • a person threatened with an attack that justifies the exercise of the righ of self-defense a person may stand his ground and defend himself by the use of all forces and means which would appear to be necessary to a reasonable... • Caljic 5.55 • The right to self-defense is not avilable to a person who seeks a quarrel....

Battery:

• Bodily Injury • Caljic 5.30 • It is lawful for a person who is being assaulted to defend himself from attack if, as a reasonable person, e has grounds for believing and does believe that bodily injury is about to be inflicted upon him. In doing so, that person may use all force and means which he believes to be reasonably necessary and which would appear to a reasonable person, in the same or similar circumstances, to be necessary to prevent the injury which appears to be imminent. • Caljic 5.31 • An assault with the fists does not justify the person being assaulted in using a deadly weapon in self-defense unless that person believes and a reasonable person in the same or similar circumstance would believe that the assault is likely to inflict great bodily injury upon him. • Caljic 5.52 • The right of self-defense exists only as long as the real or apparent threatened danger continues to exist. When the danger ceases to appear to exist, the right to use force in self-defense ends.

• Necessity

• CALJIC 4.43 • a person is not guilty of a crime when he engages in an act, otherwise criminal, through necessity, The defendant has the burden of proving by a preponderance of the evidence all of the facts necessary to establish the elements of this defense namely 1. The act charged as a criminal was done to prevent a significant and imminent evil 2. there was no reasonable legal alternative to the commission of the act 3. the reasonably foreseeable harm likely to be cause by the act was not disproportionate to the harm avoided. 4. The Defendant entertained a good-faith beleif that his act was necessary to prevent the greater harm 5. that belief was objectively reasonable under all the circumstances 6. the defendant did not substantially contribute to the creation of the emergency

• Defense of Home

• CALJIC 5.42 • A person may defend his home or dwelling agains anyone who manifestly intend or endeavors in a violent or riotus... • Penal Code Sextion 198.5 • Any person using force intended or likely to cause death or great bodly injury within his ot her residence shall be presumed to have held a reasonable fear of imminent peril of death or great bodily injury to self, family, or a member of the household when that force is used against another person, not a member of the family or household, who unlawfully and forcibly enters or has unlawfully and forcibly entered the residence and the person using the force knw or had reason ... • EG: someone break into your house


Ensembles d'études connexes

Cardiac/Urinary NCLEX review questions

View Set

Chapter 4 Treatment Settings and Rehabilitation

View Set

Assignment 1.1: Terms for Body Structure

View Set

Back of the Book Questions: Theory

View Set

Water Quality: 2019-2020 Scioly but better

View Set

Chapter 9 : Cellular Respiration and Fermanation

View Set